CONTRACTS MBE

Lakukan tugas rumah & ujian kamu dengan baik sekarang menggunakan Quizwiz!

Pursuant to its current contract with a boutique retailer of bath products, a soap maker was to supply 100 bars of soap each month. The contract called for payment upon delivery, and it specified that failure to pay would trigger an additional interest charge on the amount due. Under the current contract, the retailer failed to timely pay for the first delivery, and the soap maker imposed an additional interest charge of 10 percent of the amount due, which the retailer paid without objection. Under the previous contract between the retailer and the soap maker, which contained a similar clause, the soap maker had imposed an additional interest charge of five percent each of the three times that the retailer had failed to make timely payments. In its dealings with other customers, the soap maker has imposed an interest charge of six percent on the amount due. There is no consistent practice among manufacturers of goods in general or soap in particular as to the rate of interest imposed on the unpaid amount due. This is true regardless of the type of retailer—boutique or otherwise. If a court is called upon to determine the rate of interest that the soap maker may impose on additional late payments by the retailer under the terms of the current contract, which of the following rates should it adopt? (a) Ten percent, as the interest rate that reflects the course of performance under the current contract. (b) Six percent, as the interest rate that reflects the course of dealings between the soap maker and its other customers. (c) Five percent, as the interest rate that reflects the most frequent course of dealings between the soap maker and the retailer. (d) A reasonable rate of interest, as there is no common trade practice upon which the court can rely.

Answer choice A is correct. The Uniform Commercial Code (UCC) applies to a contract such as the one in this case because it is a contract for the sale of soap (a moveable item) and thus a contract for the sale of goods. Under the UCC, a party may explain or supplement the terms of the contract—even if the terms are unambiguous—by evidence of trade usage, course of dealing, or course of performance. Evidence of the parties' conduct with respect to the current contract at issue is accorded priority over evidence of the past course of dealings or trade usage. Consequently, because the retailer and the soap maker have treated the 10 percent interest rate as the rate to be charged when the retailer fails to timely pay for the monthly delivery of soap under the current contract, this is the rate of interest that the court should find is required by the contract. Answer choice B is incorrect because the UCC does not have a specific provision for the relevance of course of dealing by a party with third parties in ascertaining the meaning of a contract term. Answer choice C is incorrect because, although the court may consider how the parties have interpreted a similar provision in a previous contract between the same parties, this evidence of the course of dealing is subordinate to the evidence of how the parties have interpreted the provision in the current contract at issue (i.e., course of performance). Answer choice D is incorrect. Although a court may supply a missing term when the parties fail to include such a term in their contract, and the touchstone for such a term is reasonableness, when the parties have effectively defined a term through their course of performance or dealing, the court is not free to select a reasonable term when there is no trade usage upon which to rely in fixing the term.

A fashion designer wanted to start designing and selling sneakers, so she contacted a clothing manufacturer to make the sneakers in accordance with her designs. After negotiating the price and other terms, the two parties entered into a written contract on January 1. Under the contract, the manufacturer promised to make the sneakers on the condition that the designer send the design specifications to the manufacturer by February 15. The manufacturer made clear to the designer that the first batch of sneakers would take at least two months to manufacture after receipt of the design specifications. The manufacturer also promised to deliver the first batch of sneakers to the designer by May 15.The designer did not send the design specifications to the manufacturer until February 27, never notifying the manufacturer of the delay. Despite the delay, the manufacturer proceeded to manufacture the first batch of sneakers according to the specifications, and they were completed by May 1. However, due to a paperwork error by the manufacturer, it failed to deliver the sneakers to the designer until June 1. Which most accurately describes the legal obligations of the two parties under the contract on June 1? (a) The manufacturer was in breach of the contract for failing to deliver the sneakers by May 15. (b) The designer's failure to submit specifications by February 15 excused the manufacturer's duty to deliver the sneakers by May 15. (c)The designer's failure to submit specifications by February 15 waived the condition that the manufacturer would deliver the sneakers by May 15. (d) There was an implied condition that the designer give the manufacturer notice of any delay in submitting the specifications.

Answer choice A is correct. A party's duty to perform under a contract can be made contingent upon the occurrence of a condition precedent. In that case, the party's performance will not become due until the condition occurs or the nonoccurrence is excused. One excuse for nonoccurrence is waiver, which arises when the party whose duty is subject to the condition indicates by words or conduct that the condition need not be met. Answer choice B is incorrect. Here, the manufacturer's duty to make the sneakers was contingent upon the designer's sending the specifications by February 15 (condition precedent). Although the designer did not send the specifications until February 27 (nonoccurrence), the manufacturer made the sneakers anyway (waiver). As a result of its waiver, the manufacturer was required to perform as promised under the contract. And since the manufacturer promised to deliver the first batch of sneakers by May 15, its failure to do so was a breach. Answer choice C is incorrect. The manufacturer's promise to deliver the sneakers by May 15 was an obligation to deliver them by that date—not a condition that the designer could waive. Answer choice D is incorrect. A court could imply a condition that the designer give the manufacturer notice of any delay in submitting the specifications. But even if a court did so, it would likely hold that the manufacturer waived the condition by its conduct.

A collector of rare cars sent a letter to a well-known car enthusiast on January 2 regarding the possible sale by the collector of a limited-edition car. On January 3, the car enthusiast sent a letter to the collector, offering to purchase the limited-edition car for a price of $35,000. The letter also stated that the car enthusiast reserved the right to cancel the contract before January 10. Upon receiving the car enthusiast's offer on January 5, the collector immediately mailed an acceptance of the offer to the car enthusiast. On January 6, the car enthusiast sent a fax to the collector cancelling the contract. The car enthusiast received the collector's acceptance on January 7. The collector sued the car enthusiast for breach of contract. Which party is likely to prevail? (a) The car enthusiast, because his letter on January 3 was not a valid offer. (b) The car enthusiast, because the collector's acceptance arrived after he canceled the contract. (c) The collector, because the car enthusiast's offer was irrevocable. (d) The collector, because his acceptance was effective on January 5.

Answer choice A is correct. A statement is an offer only if the person to whom it is communicated could reasonably interpret it as an offer. It must express the present intent of a person to be legally bound to a contract. The primary test of whether a communication is an offer is whether an individual receiving the communication would believe that he could enter into an enforceable deal by satisfying the condition. In his letter to the collector offering to purchase the car, the car enthusiast reserved the right to cancel the contract before January 10. Accordingly, it would not be reasonable for the collector to believe that he could form an enforceable deal with the car enthusiast by accepting the offer prior to January 10. Therefore, the collector's attempted acceptance was not effective because there was not a valid offer. Answer choice B is incorrect. Under the mailbox rule, even if the offer had been valid, the collector's acceptance would have been effective upon mailing, not receipt. Here, the fact that the car enthusiast cancelled the contract prior to receiving the collector's acceptance has no effect on the outcome because there was no valid offer to accept. Answer choice C is incorrect because there was no consideration to make the offer irrevocable. Moreover, the car enthusiast specifically reserved the right to revoke the offer (cancel the contract). Answer choice D is incorrect. Although the collector's acceptance would have been effective when it was mailed on January 5 under the mailbox rule, there was not a valid offer for the collector to accept at that time.

An elderly man owned a run-down diner that was in financial trouble. The man wanted to retire, but he could not find a buyer for the diner. The man's son decided to help his father. The son agreed to pay the man $5,000 per month to cover all expenses associated with the man's cost of living. In addition, the son would also cover the costs associated with keeping the diner running. In exchange, the man would relinquish total control of the diner to the son's girlfriend. The man and his son executed a valid, written contract, which stated that the agreement would take effect 30 days after the contract was executed. The son then told his girlfriend the good news. The girlfriend was excited about the opportunity to revamp the diner. She immediately ordered new dinnerware, created and printed out new menus, and paid $3,000 upfront to an interior decorator to help redesign the diner's interior. In total, she spent $5,000. The girlfriend also told the man and the son what she had done to help revamp the diner's image. Two weeks before the man was to relinquish total control of the diner to the girlfriend, the son and the girlfriend broke off their relationship. When the time came to relinquish control to the girlfriend, the man and his son orally agreed that the man should hand control to the man's daughter. If the girlfriend sues the son for breach of contract, will she succeed? (a) Yes, because she spent $5,000 in detrimental reliance on the contract. (b) Yes, because the modification of the contract to give control to the daughter was not in writing. (c) No, because the girlfriend's benefit was only incidental to the benefit conferred on the man. (d) No, because the girlfriend was only a donee beneficiary of the contract.

Answer choice A is correct. An intended beneficiary is one to whom the promisee wishes to make a gift of the promised performance. The promisee must have an intention (explicit or implicit) to benefit the third party, or the beneficiary is incidental. Here, the son, as the promisee, agreed to have the man give control of the restaurant to the girlfriend, and his intentions were explicitly stated in the contract. Accordingly, she is an intended beneficiary. Generally, an intended beneficiary of a "gift promise" (i.e., a donee beneficiary) may sue only the promisor because the promisee is not under an obligation to the intended beneficiary. However, there is an exception when the promisee causes detrimental reliance. Here, the son told the girlfriend about the contract, her reliance on the promise was reasonably foreseeable, and the girlfriend did rely on the promise to her detriment. Thus, the girlfriend can sue the son for breach even though he is a promisee in this arrangement. Answer choice B is incorrect. The original contract and any modification of it does not fall within any of the provisions of the Statute of Frauds. Accordingly, the modification to give control to the man's daughter did not need to be in writing. However, the modification was not permitted because the girlfriend's rights as an intended beneficiary had already vested. Answer choice C is incorrect. An incidental beneficiary is one who benefits from a contract even though there is no contractual intent to benefit that person. Here, there was contractual intent to benefit the girlfriend, thus her benefit was not incidental to that of the man. Answer choice D is incorrect. If the promisee tells the intended beneficiary about the contract and should reasonably foresee reliance, and the beneficiary does so rely to his detriment, then the intended beneficiary may also sue the promisee even if she is an intended donee beneficiary.

On June 1, a cereal manufacturer hired an engineer to repair its only cereal-packaging machine by June 15. On June 5, a granola company contacted the cereal manufacturer to offer them a contract to produce 10,000 boxes of the granola company's most popular recipe to fill an order by a chain of grocery stores. The granola company needed to produce 10,000 boxes by June 20 to meet its contractual duties to the chain of grocery stores and to avoid high damages from lost sales. Believing it would have time to fill the order if the packaging machine was repaired by June 15, the cereal manufacturer agreed to a liquidated damages clause for $1,000 for each day's delay in delivery of the boxes after June 20. The engineer was inexcusably three days late repairing the packaging machine, and as a result, the cereal manufacturer was three days late in delivering the 10,000 boxes to the granola company. The cereal manufacturer paid the granola company $3,000 under the terms of their contract. The cereal manufacturer and the engineer knew on June 1 that under ordinary circumstances, the cereal manufacturer would suffer virtually no damages from a three-day delay in the repair of the packaging machine. If the cereal manufacturer sues the engineer for breach of contract and demands $3,000 in consequential damages, is the cereal manufacturer's request for damages likely to succeed? (a) No, because the damages were unforeseeable on June 1. (b) No, because time was not of the essence in the contract between the cereal manufacturer and the engineer. (c)Yes, because the engineer's inexcusably late performance was the but-for cause of the cereal manufacturer's damages. (d) Yes, because the liquidated damages clause was reasonably related to the sales lost by the granola company.

Answer choice A is correct. Damages are recoverable if they were the natural and probable consequences of breach, or if they were in the contemplation of the parties at the time the contract was made, or if they were otherwise foreseeable. Here, neither the cereal manufacturer nor the engineer had any reason to expect on June 1 that late performance on their contract could result in such high damages. Therefore, the cereal manufacturer cannot recover these consequential damages from the engineer. Answer choice B is incorrect because, regardless of whether time was of the essence on the engineer's contract with the cereal manufacturer, the cereal manufacturer cannot recover unforeseeable consequential damages from the engineer. Answer choice C is incorrect because the fact that the engineer's delay was the but-for cause of the cereal manufacturer's damages is insufficient to allow the cereal manufacturer to recover these unforeseeable consequential damages from the engineer; it can only recover consequential damages that were reasonably foreseeable at the time the contract was made. Answer choice D is incorrect. Even if the liquidated damages clause in the contract between the cereal and granola companies was reasonable and enforceable, that is not sufficient to make the damages suffered by the cereal manufacturer due to the engineer's delay reasonably foreseeable on June 1. Therefore, this fact is not relevant to the cereal manufacturer's ability to recover from the engineer.

A renowned chef entered into a written agreement with a restaurant to provide services for a period of three months. The restaurant selected the renowned chef based on his culinary taste and unique skill for testing new types of science-based cooking techniques in the kitchen. The restaurant believed its patrons would be amazed by the renowned chef's unique culinary talents. A week before the renowned chef was scheduled to begin performance under the contract, the renowned chef informed the restaurant that he found another chef to perform in his place. The other chef was familiar with and competent in utilizing the science-based cooking techniques used by the renowned chef. The restaurant immediately notified that renowned chef that it would not accept performance under the contract by the other chef. Was the agreement properly delegated? (a) No, because the restaurant had a substantial interest in having the renowned chef perform. (b) No, because there was no consideration provided for the delegation. (c) Yes, because the other chef was competent in using the same techniques as the renowned chef. (d) Yes, because the written agreement did not prohibit delegation.

Answer choice A is correct. Generally, obligations under a contract can be delegated. However, delegation is not permitted when the other party to the contract has a substantial interest in having the delegating individual perform (for example, in a personal services contract involving taste or a special skill), or the delegation is prohibited by the contract. In this case, the restaurant selected the renowned chef based on his unique skill for testing new types of science-based cooking techniques, as well as based on its belief that its patrons would be amazed by the renowned chef's unique talents. Thus, the restaurant had a substantial interest in having the renowned chef personally perform as agreed under the written contract. As a result, delegation to the other chef was not permitted here. Answer choice B is incorrect because no consideration is necessary for a delegation to be effective. Answer choice C is incorrect. Although the other chef was competent in the techniques, it was the renowned chef's unique talents that caused the restaurant to select him for the services. Therefore, delegation was not permitted, no matter how competent the other chef was. Answer choice D is incorrect because the mere fact that the written agreement did not specifically prohibit delegation does mean delegation is permitted in this instance. Even if the writing is silent on the matter of delegation, delegation is not permitted in a personal services contract involving taste or a special skill.

A bicycle vendor contracted with a supplier to purchase 60 identical helmets to be delivered in six monthly shipments of 10 helmets each. The first four shipments arrived in perfect condition. Upon receiving each shipment, the vendor accepted the helmets and made a payment to the supplier for one-sixth of the total price under the contract. The fifth shipment arrived with only four helmets and a note from the supplier that read as follows: "Six of the helmets in this shipment were damaged when we received them from the manufacturer. We will deliver six replacement helmets within two business days." The four helmets received were in perfect condition, and the vendor has an adequate supply of helmets remaining to prevent any loss of sales in the next two business days. Which of the following is true regarding the vendor's rights under the contract? (a) The vendor must accept the four helmets and is not entitled to cancel the rest of the contract. (b) The vendor must accept the four helmets, but is entitled to cancel the rest of the contract based on the breach. (c) The vendor may accept or reject the helmets, but is not entitled to cancel the rest of the contract based on the breach. (d) The vendor may accept or reject the helmets, and is entitled to cancel the rest of the contract.

Answer choice A is correct. If the seller makes a nonconforming tender or tenders nonconforming goods under one segment of an installment contract, the buyer can reject only if the nonconformity substantially impairs the value of that shipment to the buyer and cannot be cured. If the seller makes adequate assurances that he can cure the nonconformity, then the buyer must accept the shipment. Here, the nonconforming shipment does not substantially impair the value of that shipment to the vendor, and the supplier has given adequate assurances that the nonconformity will be promptly cured. Therefore, the vendor must accept the shipment, and answer choices C and D are incorrect. When there is a nonconforming tender or a tender of nonconforming goods under one segment of an installment contract, the buyer may cancel the contract only if the nonconformity substantially impairs the value of the entire contract to the buyer. Because the vendor will likely suffer no damage from this delayed shipment of six helmets, the nonconformity does not substantially impair the value of the entire contract, and the vendor cannot cancel the contract based on this nonconformity. Therefore, answer choices B and D are incorrect.

A college student wanted to purchase a car so he could visit friends at other nearby colleges. After looking at a few different used cars, the college student found one he liked and entered into negotiations with a salesman for the purchase of the car. The college student and the salesman negotiated the terms and financing for the sale of the car, including monthly payments, but the salesman informed the college student that he would not sell the car to the student unless the student had someone who could guarantee the loan that would be provided to purchase the car. The college student contacted his uncle who agreed to guarantee the loan for the student. The uncle accompanied the college student to the car dealership and met with the salesman. The uncle stated, "I'll guarantee the loan so that my nephew can buy this car and enjoy it." The salesman and the uncle shook hands. The salesman provided the college student with the loan necessary to purchase the car and the student took possession of the car. The college student subsequently failed to make any payments on the loan and the entire amount has become due. Can the salesman recover the entire amount of the loan from the uncle? (a) No, because the agreement to guarantee the loan was not in writing. (b) No, because there was no consideration for the uncle's promise to pay the debt. (c)Yes, because the uncle agreed to guarantee the loan. (d)Yes, because the uncle is primarily liable on the loan.

Answer choice A is correct. The Statute of Frauds applies to suretyship agreements. Suretyship is a three-party contract, wherein one party (the surety) promises a second party (the obligee) that the surety will be responsible for any debt or other obligation of a third party (the principal) resulting from the principal's failure to pay as agreed. A promise to answer for the debt of another must generally be in writing to be enforceable. Here, the uncle agreed to act as a surety for the college student. However, the agreement was not in writing. Therefore, because the uncle's main purpose was not his own economic advantage and this is not an indemnity contract, the salesman cannot enforce the agreement against the uncle without a writing. Answer choice B is incorrect because the uncle did not have to receive a benefit for there to be consideration under the contract. The salesman merely had to have a detriment. Therefore, the contract is not unenforceable due to lack of consideration. Answer choice C is incorrect. Although the uncle agreed to guarantee the loan to the college student, the agreement was not enforceable because it was not in writing. Answer choice D is incorrect. The uncle was secondarily liable on the loan as a surety—he would only have to pay if the college student failed to pay. Moreover, even if the student never pays, the salesman cannot recover the amount of the loan from the uncle because the agreement cannot be enforced against him without a writing.

A car collector entered into an agreement with a car restorer to completely restore an exotic convertible. The car collector sought substantial work on the interior and exterior of the car, as well as the engine. The car restorer began working on the interior of the car and was able to complete a substantial portion of the interior restoration in one day. After the car restorer left his shop for the day, a severe storm occurred that contained highly damaging winds. A large tree located next to the shop was uprooted and landed directly on the exotic convertible, completely destroying it. The car restorer sought to recover the value of the work performed on the car's interior from the car collector. Will the car restorer's claim to recover the value of the work performed succeed? (a) No, because no benefit was conferred by the car restorer upon the car collector. (b) No, because the duties of both parties to the contract were discharged by impossibility. (c) Yes, because the car collector's duty under the contract is not discharged. (d) Yes, because the car restorer can recover the value of his service in quasi-contract.

Answer choice A is correct. The defense of impracticability (impossibility) is available if the specific subject matter of the contract is destroyed. In this case, the exotic convertible was destroyed through no fault of either party. As a result, the parties were no longer required to perform under the contract. The car restorer can still recover in quasi-contract for any benefit that was conferred prior to impracticability, but the court would only award restitutionary recovery in order to prevent the unjust enrichment. When a defendant is unjustly enriched by the plaintiff, restitution generally allows the plaintiff to recover on the benefit conferred by the plaintiff upon the defendant, rather than on the harm suffered by the plaintiff. Here, no benefit has been conferred, and thus the car restorer cannot recover the value of his service. Answer choice B is incorrect. Although the duties of both parties were discharged, this fact alone would not bar the car restorer's recovery in quasi-contract for the work performed. Therefore, answer choice A is the better answer. Answer choice C is incorrect because the duties of both parties are discharged due to impracticability. Answer choice D is incorrect. The fact that the exotic convertible was destroyed does not preclude the car restorer from recovering the value of the work performed prior to the convertible's destruction, but because no benefit was actually conferred upon the car collector, the car restorer cannot recover the value of his work.

On August 1, a buyer and a seller contracted in writing for the sale of the seller's duck farm. The contract provided that the closing would occur on September 15. On September 1, the buyer sent the seller an email containing the following: "After discussing our August 1 contract with my financial advisors and some experts in the field, I am increasingly concerned that entering the duck market at this time is a grave mistake. As such, I do not intend to buy your duck farm unless I am legally obligated to do so." If the seller sues the buyer for breach of contract on September 1, is the seller likely to succeed? (a) No, because the buyer's statement is neither a present breach nor a repudiation of the August 1 contract. (b) No, because the seller has not yet accepted or relied upon the buyer's repudiation. (c) Yes, because the buyer has committed a breach by anticipatory repudiation. (d) Yes, because the buyer's statement created reasonable grounds for the seller's insecurity with respect to the buyer's performance.

Answer choice A is correct. The doctrine of anticipatory repudiation is applicable when a promisor repudiates a promise before the time for performance is due. The repudiation must be clear and unequivocal, as opposed to merely creating insecurity. Here, the buyer stated that he did not intend to purchase the farm unless he was legally obligated to do so. Because the buyer is still bound by the contract, he is legally obligated to purchase the farm; his concern about entering the duck market does not amount to a clear and unequivocal statement that he will not perform. Answer choice B is incorrect. Repudiation may be retracted until the promisee acts in reliance on the repudiation, signifies acceptance of the repudiation, or commences an action for breach of contract. However, there is no requirement that the promisee accept or rely upon the repudiation to bring an action for breach. Answer choice C is incorrect because, as discussed above, the buyer has not clearly and unequivocally indicated that he will not perform under the contract. Answer choice D is incorrect because statements causing mere insecurity about a party's performance are insufficient to create a cause of action against the party making the statement.

A borrower owed a lender $50,000 due on March 1. On January 10, the lender telephoned the borrower and said that he would discharge the debt if the borrower would promise to pay the lender $45,000 by January 15. The borrower responded, "I will attempt to get the money together." On January 11, the lender again telephoned the borrower and said that he had changed his mind and would expect the borrower to make full payment on March 1. On January 15, the borrower tendered $45,000 as full payment, which the lender refused to accept. On March 1, the borrower refused the lender's demand for $50,000, and the lender sued for that amount. Which of the following statements best supports the lender's position? (a) The borrower's January 10 statement was not a return promise, and therefore the lender effectively revoked his offer on January 11. (b) The January 10 telephone conversation between the lender and the borrower created an executory accord and therefore did not operate as a discharge of the $50,000 debt. (c) The lender's offer to discharge the debt was a gift promise and therefore was not binding on the lender. (d) The lender's promise to discharge the $50,000 debt was not enforceable because it was not in writing.

Answer choice A is correct. The lender's offer requested that the borrower accept by making a return promise. However, the borrower's response to the lender's offer was a statement of intention, which was not sufficiently promissory to constitute acceptance of an offer and create a binding contract. Therefore, the lender effectively revoked his offer on January 11. Answer choice B is incorrect because the mutual assent required for an enforceable executory accord did not arise from the January 10 conversation between the lender and the borrower. Answer choice C is incorrect because an offer in which a creditor promises to accept a payment for less than the amount owed prior to the due date is sufficient consideration to support a return promise by a borrower accepting the offer. Moreover, the dispositive issue is not whether there was consideration but whether the borrower's response to the lender's offer was an acceptance. Answer choice D is incorrect because the purported contract between the lender and the borrower was not a transaction that would fall within a statute of frauds.

A private high school was in the market for new desks and chairs for its classrooms. It had inquired into the cost of acquiring 1,000 new desks and chairs from a particular vendor. On June 15, the vendor sent a signed letter to the private high school offering to sell 1,000 desks and chairs for $30,000. The letter stated that an acceptance of the offer by the high school would be effective so long as it was received by June 25. On June 23, the private high school mailed its signed, written acceptance of the vendor's offer. On June 24, the vendor, after concluding that the price it originally requested was too low, directly notified the private high school that it was revoking its offer. The vendor received the private high school's acceptance on June 26.Was an enforceable contract between the private high school and the vendor formed? (a) No, because the private high school's acceptance was received on June 26. (b) No, because the vendor revoked the offer on June 24. (c) Yes, because the private high school mailed its acceptance on June 23. (d) Yes, because the vendor's offer was irrevocable.

Answer choice A is correct. Under the mailbox rule, an acceptance that is mailed within the allotted response time is effective when sent (not upon receipt), unless the offer provides otherwise. Here, the vendor's offer stated that an acceptance would only be effective if it was received by June 25. Thus, the mailbox rule does not apply. The vendor received the private high school's acceptance on June 26. The acceptance is ineffective because it was not received by June 25. Therefore, the offer was no longer available when the acceptance was received. As a result, no contract was formed. Answer choice B is incorrect. Under the UCC, an offer to buy or sell goods is irrevocable if the offeror is a merchant, there is assurance that the offer is to remain open, and the assurance is contained in a signed writing by the merchant. Here, the vendor was a merchant and he informed the private high school in a signed letter that it could accept his offer by June 25. Accordingly, the vendor's offer was irrevocable until June 25. Thus, the vendor's attempted revocation of the offer on June 24 was ineffective. Answer choice C is incorrect because the acceptance had to be received by June 25—it was not effective upon mailing. Answer choice D is incorrect. Although the vendor's offer was irrevocable under the UCC firm offer rule, the offer was only irrevocable until June 25. The offer was no longer available on June 26 when the vendor received the private high school's acceptance.

Before putting her home up for sale, a homeowner painted the living room ceiling to conceal major water damage caused by a leaking roof that had not yet been repaired. On the first day the home was offered for sale, the homeowner gave a buyer a personal tour. The homeowner made no statements at all regarding the water damage or the roof. Without discovering the water damage or the leaking roof and without consulting a lawyer, the buyer immediately agreed in writing to buy the home for $200,000. Before the closing date, the buyer discovered the water damage and the leaking roof. The cost of repair was estimated at $22,000. The buyer has refused to go through with the purchase. If the homeowner sues the buyer for breach of contract, is the homeowner likely to prevail? (a) No, because no contract was formed since the buyer did not have a real opportunity to understand the essential terms of the contract. (b) No, because the homeowner concealed evidence of the water damage and of the leaking roof. (c) Yes, because the homeowner made no affirmative statements of fact about the water damage or the leaking roof. (d) Yes, because the buyer acted unreasonably by failing to employ an inspector to conduct an independent inspection of the home.

Answer choice B is correct. A misrepresentation of fact that induces assent provides a basis for avoidance of a contract. A misrepresentation is an untrue assertion of fact. Affirmative conduct that is intended or known likely to prevent another from learning a fact is equivalent to an assertion that the fact does not exist and constitutes a misrepresentation. The homeowner's painting over the water damage constituted affirmative conduct intended to prevent the buyer from learning of the water damage and the leaking roof. Therefore, the buyer's assent was induced by a misrepresentation, and the buyer can avoid the contract. Answer choice A is incorrect because the facts fail to support the conclusion that the buyer did not understand the essential terms of the contract. Moreover, an assenting party's lack of an opportunity to understand the essential terms of a contract will not necessarily invalidate the contract. In addition, a party's failure to use care in reading a writing will not necessarily preclude that party from avoiding a contract induced by fraud. Answer choice C is incorrect because an affirmative statement is not required to avoid a contract on the basis of misrepresentation. Answer choice D is incorrect because the homeowner's painting over the water damage was affirmative conduct intended to prevent the buyer from learning of the water damage and the leaking roof. The buyer's not having employed an independent inspector will not prevent the buyer from avoiding the contract based on the homeowner's concealment.

A junior associate at a law firm, prone to compulsive gambling and riddled with law school debt, threatened to file a civil suit against his law firm for discrimination after being passed over for a promotion. In actuality, the associate knew that he was never up for the promotion due to poor work performance, but the firm did not want the case to go to trial. The firm offered to settle with the associate for $10,000 in exchange for a promise not to sue. The associate agreed. However, the firm's senior partner changed her mind and the firm later repudiated the settlement agreement. The associate then filed suit against the firm for breach of contract. At trial, it was established that the firm did not discriminate against the associate, that the associate knew he was not up for a promotion, that the recipient of the promotion and several other associates had demonstrably superior job performance to the junior associate, and that the associate's suit was motivated by his need to pay his debts. Is the associate entitled to recover under the settlement agreement? (a) No, because the consideration given for the settlement agreement was inadequate. (b) No, because there was no bargained-for exchange to support the settlement agreement. (c) Yes, because public policy favors the settlement of litigation. (d) Yes, because there was a legal detriment bargained for in exchange for a promise.

Answer choice B is correct. A settlement agreement is a contract, so it can only be enforced if it is supported by consideration. For consideration to exist, (1) there must be a bargained-for exchange of promises or performance and (2) the promised or provided performance must have legal value. A party's promise not to bring suit to enforce a claim or assert a defense has legal value—and constitutes consideration for a settlement agreement—only if the claim or defense is valid or subject to a good-faith dispute or the party honestly believes that the claim or defense may be valid. Here, the associate promised not to sue the firm for discrimination in exchange for $10,000. But since the associate knew that he was not up for the promotion, he did not honestly believe in the validity of his discrimination claim. Accordingly, there was no bargained-for exchange to support the associate's settlement agreement with the firm, and he is not entitled to recover under that agreement. Answer choice A is incorrect. Parties generally cannot challenge contracts on the basis that the consideration was inadequate—eg, due to a difference in economic value between items exchanged. And here, instead of giving inadequate consideration, the associate gave no consideration at all since he did not honestly believe that his claim was valid. Answer choice C is incorrect. Public policy favors the settlement of disputed claims to reduce the volume of litigation. However, in the absence of consideration, settlement agreements (like other contracts) are not enforceable. Answer choice D is incorrect. An exchange has legal value if there is a detriment to both sides. Here, there was a detriment to the firm (payment of $10,000 to the associate) but not to the associate, who knew that the claim he promised to give up was invalid.

A landowner and a conservationist executed a contract to transfer the landowner's undeveloped tract of land to the conservationist's wildlife rescue organization to be used to raise an endangered species of elk as part of a repopulation project. Because the landowner lived on a tract of land adjacent to the undeveloped tract and did not want the elk wandering onto his property, the landowner included a clause in the land sale contract providing that the conservationist must build an 8-foot fence around the undeveloped tract. The conservationist agreed to this term, and both parties signed the written contract of sale. The conservationist built an 8-foot fence between the landowner's tract and the purchased tract, but never completely enclosed the tract with a fence. As a result, the elk began to wander onto the property of the landowner's neighbor, causing property damage. Satisfied that the elk were staying off his property, the landowner never brought an action against the conservationist. Can the neighbor prevail against the conservationist in a breach of contract action? (a) No, because the landowner has waived the condition requiring the fence. (b) No, because the neighbor is an incidental beneficiary to the contract. (c) Yes, because the elk have caused property damage on the neighbor's land. (d) Yes, but only if the jurisdiction has adopted the Second Restatement approach to third-party beneficiaries.

Answer choice B is correct. An incidental beneficiary has no rights to enforce a contract. Here, the contract between the landowner and the conservationist was not made with any intent to benefit the neighbor. At most, the neighbor is an incidental beneficiary of the contract. Thus, the neighbor cannot bring an action based on the contract. Answer choice A is incorrect because even if the landowner had also tried to bring an action based on the conservationist's failure to meet this condition, the neighbor would remain an incidental beneficiary and would be unable to bring an action on the contract. Answer choice C is incorrect. Although property damage may expose the conservationist to tort liability, it is not sufficient to create a cause of action based on the contract. Answer choice D is incorrect because, under both the First and Second Restatements, an incidental beneficiary does not have a right to enforce the contract.

A buyer and a homeowner signed a writing on July 1 providing that the homeowner would sell his home to the buyer on July 20 for $250,000. However, the buyer's obligation to purchase the home was conditioned on the buyer being able to obtain financing before the date of closing. On July 15, the homeowner received another offer on his home for $275,000. Because the homeowner had not yet heard from the buyer regarding whether he had obtained financing for the purchase, the homeowner accepted the offer and delivered title to the home to the second buyer. On July 20, the buyer, having obtained the necessary financing, visited the homeowner to close on their contract. Because the homeowner could no longer deliver the title of the home, the buyer sued the homeowner for breach of contract. If the buyer succeeds in his action, which of the following will the buyer be entitled to recover? (a) Nominal damages only. (b) The amount, if any, by which the fair market value of the home exceeded $250,000 on the date of the homeowner's breach. (c) $25,000, plus the amount, if any, by which the fair market value of the home exceeded $250,000 on the date of the homeowner's breach. (d) The full fair market value of the home on the date of the homeowner's breach.

Answer choice B is correct. Damages for failing to perform a real estate sales contract are measured by the difference between the contract price and the market value on the date of the breach. Answer choice A is incorrect because the buyer is entitled to more than nominal damages for the homeowner's breach. Answer choice C is incorrect because the buyer is not entitled to the difference between the contract price he offered and the price offered by the second buyer. Answer choice D is incorrect because the buyer is only entitled to the amount by which the fair market value of the home exceeded $250,000 on the date of the homeowner's breach. Allowing him to recover the full market value while allowing him to keep the amount he would have paid for the home would result in the buyer's unjust enrichment.

A youth community organization and a well-known local graffiti artist, who was popular with youths in the area, entered into a written contract for the graffiti artist to create a mural on the wall of a new center for the organization. The artist was to be paid $1,500 upon completion of the mural in one month. The contract did not address the issue of delegation. Two weeks later, when the graffiti artist had completed approximately one-third of the mural, he was approached by an art gallery that wanted him to create a mural on one of their walls for an upcoming modern art show in two weeks. The graffiti artist contacted his long-time friend, a talented artist in his own right, and asked him to take over the community center mural for him. The friend agreed, promising to finish the mural in the same style as the graffiti artist. The graffiti artist had not worked from plans or preliminary drawings but instead had relied on his own creative inspiration each day in the painting the mural. When the graffiti artist notified the community organization, the community organization objected. Can the community center refuse to permit the graffiti artist's friend to complete the mural without breaching the contract? (a)Yes, because the graffiti artist only completed one-third of the mural before delegating his duty to his friend. (b) Yes, because the community center had a substantial interest in having the graffiti artist create the mural. (c) No, because the artist's friend agreed to complete the mural in the same style as that of the graffiti artist. (d) No, because the contract did not prohibit delegation of the graffiti artist's duty to create the mural.

Answer choice B is correct. Generally, obligations under a contract can be delegated. However, delegation is not permitted when the other party to the contract has a substantial interest in having the delegating individual perform, such as in a personal services contract involving taste or a special skill. Consequently, when an artist is engaged to create an original work, delegation is not permitted. Answer choice A is incorrect because it is irrelevant how much of the mural the graffiti artist had completed. Even if the graffiti artist had completed 90% of the mural, he would still be in breach of contract because he was not permitted to delegate the contractual duties to his friend. Answer choice C is incorrect. Answer choice C is incorrect. When delegation is permitted, the other party must accept the conforming performance of a delegate or be in breach of the contract. However, delegation is not permitted when the other party to the contract has a substantial interest in having the delegating individual perform, as was the case here. With regard to the creation of a work of art, the other party has a substantial interest in having the artist himself complete the work of art, even though the delegate would do so in the same style as the artist. Therefore, the community center need not accept the delegate's performance. Answer choice D is incorrect because, even though the contract did not prohibit delegation, the graffiti artist was still not permitted to delegate his duties because the community organization had a substantial interest in having the graffiti artist, rather than his friend, create the mural.

A line cook at a restaurant was inspired to become a chef, but he lacked the necessary training. The line cook asked the chef at the restaurant if he could borrow $2,400 to take a professional cooking class. The chef agreed in a writing signed by both parties to loan him the money, provided that the line cook repaid her $200 each month for a year, beginning when the six-month class was over. The line cook made the first payment, but then missed the next two payments. The chef brought an action against the line cook prior to the time the fourth payment was due. How much is the chef entitled to in a judgment against the line cook? (a) Nothing, because the entire loan amount was not yet due. (b) $400 (c) $600 (d) $2,400

Answer choice B is correct. If at the time of a breach the only remaining duties of performance are those of the party in breach, and those duties are for the payment of money in installments not related to one another, then breach by non-performance as to less than the whole, whether or not accompanied or followed by a repudiation, does not give rise to a claim for damages for total breach and is a partial breach of contract only. A claim for damages for partial breach is one for damages based on only part of the injured party's remaining rights to performance. Here, the chef is entitled to judgment only for the amount due for two months of payment that the line cook missed. Answer choice A is incorrect. As stated above, when the payments are in installments not related to one another, the nonbreaching party can recover for the partial breach. Answer choice C is incorrect because the fourth payment is not yet due, so the chef is only owed $400, not $600. Answer choice D is incorrect, as this is a partial breach, and the chef is only entitled to recover the payments missed for the second and third month.

A motorcycle enthusiast purchased a custom-made motorcycle from a boutique motorcycle shop. The enthusiast paid $5,000 for the motorcycle, which was to be delivered to him in one month. One week after he purchased the motorcycle, the enthusiast decided that he no longer wanted the motorcycle. He called up his best friend and told him that he would give him the motorcycle when it was finished. The enthusiast also instructed the shop to deliver the motorcycle to his friend. Three weeks later, the shop delivered the motorcycle to the enthusiast. The enthusiast accepted the motorcycle without protest. When the friend found out, he sued the shop for its failure to deliver the motorcycle to him. Will the friend succeed? (a) No, because an assignee cannot force an obligor to comply with an assignment by the obligee-assignor. (b) No, because there was no consideration to support the enthusiast's assignment of the motorcycle to his friend. (c) Yes, because the friend detrimentally relied upon the enthusiast's assignment. (d) Yes, because the enthusiast made a valid assignment of the motorcycle to the friend.

Answer choice B is correct. If no consideration supports an assignment of contractual rights (a gratuitous assignment), the assignment is generally revocable, unless the obligor has already performed or promissory estoppel applies. Here, the enthusiast accepted the motorcycle without protest. His acceptance effectively revoked the gratuitous assignment. In addition, there is no indication that the friend relied on this assignment to his detriment. Answer choice A is incorrect. The shop, as obligor, may be compelled to comply if the assignment is valid and enforceable by the assignee. Here, however, the assignment was gratuitous and was revoked by the enthusiast's acceptance of the motorcycle. Therefore, the friend cannot enforce the assignment. Answer choice C is incorrect. Promissory estoppel is referred to as a "consideration substitute." The doctrine of promissory estoppel (detrimental reliance) can be used under certain circumstances to enforce a promise that is not supported by consideration. However, as stated above, there are no facts to indicate that the friend detrimentally relied upon the enthusiast's gift of the motorcycle. Answer choice D is incorrect. The valid assignment to the friend could be revoked by the enthusiast because it was a gratuitous assignment.

A film student asked a film director if he would loan her $3,000 to make a short film. She promised to pay him back by the time she graduated from film school in a year and a half. The film student's fiancé told the director that if the film student did not repay the loan before the film student graduated from film school, the fiancé would repay the loan. The director agreed, and lent the film student $3,000. When the film student graduated from film school, she had not yet repaid the loan and did not have the money to do so. The fiancé, who was no longer engaged to the film student, refused to pay as well. What is the fiancé's best defense if the director brings an action against him to recover the $3,000? (a) The fiancé's promise was not in writing even though it was made in consideration of marriage. (b) The Statute of Frauds applies to a suretyship promise. (c) The fiancé did not receive consideration for his promise to pay the film student's debt. (d) The fiancé's oral promise is not enforceable because repayment was not due for a year and a half.

Answer choice B is correct. Suretyship is a three-party contract, wherein one party (the surety) promises a second party (the obligee) that the surety will be responsible for any debt or other obligation of a third party (the principal) resulting from the principal's failure to pay as agreed. Here, there was a suretyship contract, which the Statute of Frauds generally requires to be in writing to be enforceable. There is an exception to this rule if the main purpose of the surety in agreeing to pay the debt of the principal is the surety's own economic advantage, rather than for the principal's benefit. Here, the facts suggest that the fiancé's motivation was personal—to help his fiancée—rather than for his own economic benefit. Thus, the fiancé can raise the Statute of Frauds as a defense to repayment. Answer choice A is incorrect. Any agreement in consideration of marriage is within the Statute of Frauds. In this case, the fiancé's promise to the director, while presumably motivated by his future marriage to the film student, was not made in exchange for a promise to marry. Answer choice C is incorrect because the fiancé did receive consideration. Consideration requires that some legal detriment be bargained for in order for a contract to exist. The exchange must be of some benefit to the promisor or detriment to the promisee. Here, the payment of $3,000 to the film student was a detriment to the director as promisee. Thus, the promise was supported by consideration. Answer choice D is incorrect. In order for the Statute of Frauds to apply, the terms of the contract must make it impossible for performance to be completed within one year. Here, it was possible for performance to be completed within a year because the film student was not prohibited from repaying the loan before the expiration of the one-year period.

A manufacturer entered into a written contract with a pet store chain whereby the manufacturer promised to design and create 1,000 collars to discourage dogs under 10 pounds from barking. In exchange, the pet store agreed to pay $50,000 upon delivery of these "bark prevention" collars. After doing some research and development, the manufacturer realized that the current device used in such collars for larger dogs would not work for dogs under 10 pounds. Upon determining the need for a new type of vibration sensor, which would add an additional $10,000 to the cost of the collars, the manufacturer informed the pet store of the issue. The pet store chain signed a document agreeing to the increased purchase price. After the collars were delivered and accepted, the pet store paid the manufacturer $50,000 only. If the manufacturer files an action against the pet store to recover the additional $10,000, will it succeed? (a) Yes, because the unforeseen difficulties in designing a collar for small dogs made performance impracticable. (b) Yes, because consideration is not required for a modification of this contract. (c) No, because the manufacturer did not sign the price increase document. (d) No, because there was no bargained for exchange for the increased price.

Answer choice B is correct. The UCC does not require consideration to modify a contract, although the modified contract must satisfy the Statute of Frauds if the sale of goods is for at least $500. Here, the UCC applies to the sale of dog collars. Since the modification was in writing, it is enforceable even though the buyer did not receive additional consideration for the modification. Answer choice A is incorrect. Although impracticability may constitute a reason for permitting modification of a contract without additional consideration under the common law, a mere increase in the cost to a party in order to be able to perform does not generally constitute impracticability. Moreover, the manufacturer likely assumed the risk of having to create a new sensor. Consequently, the doctrine of impracticability does not apply here. Answer choice C is incorrect. The Statute of Frauds only requires that the party against whom a writing is to be enforced sign the writing (i.e., the party to be charged). The chain's signature on the document is sufficient because the manufacturer is seeking to enforce the pet store chain's promise to pay an additional $10,000. Answer choice D is incorrect because there does not need to be a bargained for exchange to support a modification of a contract governed by the UCC.

A farmer and an innkeeper have dealt with each other on a weekly basis for the last five years. Every Sunday, the innkeeper sends a request for his produce requirements for the following week, and the farmer delivers the requested produce to the inn the following Saturday evening. Upon delivery, the innkeeper signs an invoice indicating the agreed-upon price for that week's delivery and setting the deadline for timely payment as six weeks after delivery. Each invoice signed by the innkeeper in the last five years has noted that the farmer does not offer a discount for pre-delivery payments, but the invoices are silent regarding discounts for early payments. The custom of the industry in the jurisdiction is to allow a five percent discount from the invoice price for any payments made at least a month before their due date, and an additional five percent discount for early payments made before delivery. However, in all of their prior transactions, the innkeeper took up to five weeks to pay after delivery and always deducted five percent from each invoice price. The innkeeper has never paid before delivery, and the farmer has never objected to the five percent discount. The present contract called for a single delivery of produce for $1,000, and the same delivery procedures and invoice were used in the present contract as had been used previously. The innkeeper paid the farmer $900 the day before the delivery, taking a 10 percent discount. The farmer delivered the produce, and the innkeeper signed the invoice. However, the next day, the innkeeper received the following letter from the farmer: "Your payment of $900 is insufficient under our contract. Payment in full in accordance with your signed invoice is due within six weeks of delivery. Recall that no discounts are permitted under our contract. "Which of the following statements concerning these facts is most accurate? (a) The custom of the trade controls, and the innkeeper does not owe the farmer any additional payment. (b) The parties' course of dealing controls, and the innkeeper owes the farmer $50 as long as payment is made before the due date. (c) The farmer's retraction of his prior waiver controls, and the innkeeper owes the farmer $100. (d) The written contract controls, and the innkeeper owes the farmer $100 regardless of when he pays the farmer.

Answer choice B is correct. The Uniform Commercial Code (UCC) controls this fact pattern. UCC § 2-202 provides that, although a final written expression of an agreement may not be contradicted by any prior agreement, it may be explained or supplemented by course of dealing, trade usage, or course of performance. Here, the custom of the trade would allow a five percent discount for any payment that is at least a month early, and an additional five percent if the payment is made before delivery. Here, the innkeeper tried to apply this discount to his pre-delivery payment. However, a course of dealing, when inconsistent with a trade usage, controls. The parties' course of dealings in this case has always allowed a five percent discount for any payment made before its due date. Therefore, the agreement in this case should be interpreted to embody the course of dealing of the parties, and as long as the farmer completes the payment before the due date. Answer choice A is incorrect because, as explained above, a course of dealing between two parties with an established pattern of transactions controls over the trade usage. Answer choice C is incorrect because this question doesn't raise an issue of a waiver or retraction of a waiver. Instead, the question goes to the meaning of the agreement made by the parties. Answer choice D is incorrect because, as explained above, the UCC allows the course of dealing to control.

An engineer entered into a written contract with an owner to serve in the essential position of on-site supervisor for construction of an office building. The day after signing the contract, the engineer was injured while bicycling and was rendered physically incapable of performing as the on-site supervisor. The engineer offered to serve as an off-site consultant for the same pay as originally agreed to by the parties. Is the owner likely to prevail in an action against the engineer for damages resulting from his failure to perform under the contract? (a) No, because the engineer offered a reasonable substitute by offering to serve as an off-site consultant. (b) No, because the engineer's physical ability to perform as on-site supervisor was a basic assumption of the contract. (c) Yes, because the engineer breached the contract by disappointing the owner's expectations. (d) Yes, because the engineer's duty to perform was personal and absolute.

Answer choice B is correct. The engineer is not in breach. The incapacity of a particular person to perform his or her duty under a contract renders the performance impracticable and operates as an excuse for nonperformance. The parties contracted for the engineer to personally provide on-site services. Therefore, the engineer's continued ability to perform those services was a basic assumption of the contract, and his nonperformance is excused. Answer choice A is incorrect. The parties contracted for the engineer to perform on-site services. An offer by a party to perform less than what was contracted for, even if the substituted performance is reasonable, will not excuse a breach. More importantly, his nonperformance is excused for the reasons set forth above. Answer choice C is incorrect because the engineer is not in breach. Answer choice D is incorrect. Although the engineer's duty to perform was personal and definite, he is not in breach.

An amateur bodybuilder entered into a contract with a personal trainer to help the bodybuilder prepare for his first bodybuilding competition. The written agreement stated that the bodybuilder would pay the personal trainer $4,000 to do daily weight-lifting sessions, and teach him about competition musculature and proper nutrition. Per the contract, the training would occur over a period of eight weeks leading up to the competition, and the bodybuilder would pay the personal trainer at the end of the eight weeks. Three weeks before the competition, the bodybuilder was lifting weights with the guidance of the personal trainer, when suddenly a beam fell from the gym's ceiling and onto the bodybuilder while he was lifting weights. The bodybuilder suffered severe head trauma, and was put into a medically induced coma for more than a month. With regard to the contractual duties of each party, which of the following is a correct statement? (a) Only the personal trainer's duty to perform is dismissed, but he can recover reliance damages from the bodybuilder for the weeks that he did train the bodybuilder. (b) Each party's duty to perform is dismissed, and the personal trainer cannot recover anything from the bodybuilder. (c) You Selected: Each party's duty to perform is dismissed, but the personal trainer can recover under a theory of quasi-contract for the weeks that he did train the bodybuilder. (d) Only the personal trainer's duty to perform is dismissed, but the bodybuilder is still obligated under the contract because the amount that he owed the personal trainer can be determined.

Answer choice C is correct. A party's duty to perform can be discharged by impracticability. Here, the parties executed a valid contract, but performance on both sides became impracticable because of the occurrence of an unforeseeable event (the falling beam injuring the bodybuilder), the nonoccurrence of which was a basic assumption of the contract. At that point, both parties' contractual duties ceased—the trainer did not have to provide training services, and the bodybuilder did not have to pay the $4,000 due under the contract. Although there would be no legal recovery, the equitable doctrine of restitution (quasi-contract) would apply to prevent unjust enrichment. A party may recover in quasi-contract for any benefit that was conferred prior to impracticability. In this case, the bodybuilder would have to pay the trainer $2,500 for the services already rendered (i.e., the five weeks of training). Answer choice A is incorrect because both parties' duties are discharged, not merely the personal trainer's. Answer choice B is incorrect, as quasi-contractual recovery is permitted for any benefit conferred prior to impracticability. Answer choice D is incorrect because both parties' duties are discharged, not merely the personal trainer's.

A buyer contracted with an owner of commercial property located in a strip mall to purchase the property for $750,000. The contract called for closing and delivery of possession to occur on March 1. At the time of contracting, the owner informed the buyer that the current tenants were wrongfully refusing to vacate the premises, and would not do so until March 31. The owner notified the current tenants, who ran a call center on the premises, that they would need to vacate the premises before April 1. Although the current tenants stopped operating the call center before April 1, they were not able to empty the space completely because they had attached numerous cubicles to the floor, and the cubicles occupied the entire space of the property. Shortly after entering the contract, the buyer, who unbeknownst to the owner, planned on using the property as a gymnastics studio, ordered gymnastic equipment that was to be delivered on March 2. Due to the delay, the buyer was forced to rent a storage unit for this equipment for $1,000. By April 1, the fair market value of the property had risen to $755,000. In addition, the monthly fair market rental value of the property was $3,000. If the buyer files an action against the owner for damages, what will she likely recover? (a) Consequential damages of $1,000 in storage unit costs, and expectation damages of $3,000 for the fair rental value of the property for the month of March. (b) Expectation damages of $3,000 for the fair rental value of the property for the month of March. (c) Nothing, because the property increased in value during the month of March. (d) Nothing, because the delay was attributable to the tenants, not the owner.

Answer choice B is correct. The owner breached the contract by failing to deliver possession of the property on March 1. As a result, the owner is liable to the buyer for damages that the buyer suffered because of the breach. The buyer was denied possession of the property for the month of March. Therefore, the buyer is entitled to the fair market rental value of the property for a month, which is $3,000. Answer choice A is incorrect. The owner was unaware of the buyer's intended use of the property. Therefore, the buyer is not entitled to consequential damages for the cost of storing the gymnastic equipment for the month. Nor would the buyer be entitled to the storage fee as incidental damages. The buyer had notice that the previous tenants were wrongfully refusing to vacate and that the property would not be vacant until March 31. In light of this, it wasn't commercially reasonable to schedule the delivery for March 2. Answer choice C is incorrect because the buyer would have paid the same price, and received the same benefit, regardless of the owner's breach, which makes the increased value of the property insufficient compensation for the loss the buyer suffered. Answer choice D is incorrect. Although the owner may seek damages from the tenants for wrongfully holding over, the owner is nevertheless liable to the buyer for his own failure to deliver the property to the buyer on March 1, as contractually promised.

A lawn service company agreed in writing to purchase from a supplier all of its requirements for lawn care products during the next calendar year. In the writing, the supplier agreed to fulfill those requirements and to give the company a 10% discount off its published prices, but it reserved the right to increase the published prices during the year. After the parties had performed under the agreement for three months, the supplier notified the company that it would no longer give the company the 10% discount off the published prices. Does the company have a viable claim against the supplier for breach of contract? (a) Yes, because part performance of the agreement by both parties made it enforceable for the full year. (b) Yes, because the company's agreement to buy all of its lawn care products from the supplier made the agreement enforceable. (c) No, because the supplier could, and did, revoke its offer with respect to future deliveries. (d) No, because the absence of a minimum quantity term rendered the company's promise illusory.

Answer choice B is correct. The parties entered into an enforceable requirements contract under UCC § 2-306. Although the terms of the parties' agreement granted the supplier the discretion to increase the published prices during the year, the contract did not grant the supplier the right to discontinue the promised 10% discount off the published prices. The supplier's refusal to give the company the 10% discount was a breach of the contract's agreed-upon terms. Answer choice A is incorrect because, pursuant to the terms of their contract, the parties agreed to a one-year term. Therefore, part performance is not required in order for the contract to be enforceable for a full year. Answer choice C is incorrect because revocation relates to the contract formation process. Given that mutual assent has occurred, revocation is not a relevant issue. Answer choice D is incorrect. UCC § 2-306(1) validates the enforceability of requirements contracts even if the parties do not include a minimum quantity term in their agreement. Therefore, the absence of a minimum quantity term in the parties' requirements contract did not render it illusory, and the contract is enforceable.

An actor straight out of drama school and an agent entered into a one-year written contract that described the services the agent would provide. Because he was eager for work, the actor agreed, in the contract, to pay the agent 15% of his yearly earnings. At the end of the year, the actor was so pleased with his many roles that he gave the agent 20% of his earnings. After the first contract had expired, the actor and the agent decided to continue working together. They photocopied their old contract, changed the date, and signed it. At the end of the year, a dispute arose as to what percentage of earnings the actor owed. It is a trade practice in the acting profession for actors to pay their agents 10% of their yearly earnings, payable at the end of the year. What percentage of the actor's earnings is a court most likely to award the agent? (a) 20%, because course of dealing is given greater weight than trade usage. (b) 15%, because it was an express term of the contract. (c) 10%, because trade usage is the applicable default rule. (d) Nothing, because the contract is too indefinite.

Answer choice B is correct. The parties' new contract adopted the terms of their previous contract, which included the 15% fee. Evidence of express terms is given greater weight than evidence of trade usage. Therefore the express term, the 15% fee, is controlling because a court would give it greater weight than the evidence of trade usage, the 10% fee. Answer choice A is incorrect because the generally applicable standard, which provides that evidence of course of dealing is given greater weight than evidence of trade usage, is inapplicable here because a "course of dealing" was never established and because the written contract was explicit. One instance of previous conduct is insufficient to establish a prior course of dealing. Answer choice C is incorrect because, as previously explained, evidence of an express term is given greater weight than evidence of trade usage. Answer choice D is incorrect because the contract contains the essential terms, including the duration, a description of the agent's services, and the method for calculating the agent's fee, 15% of the actor's earnings in this case.

A computer retail outlet contracted to service a bank's computer equipment for one year at a fixed monthly fee under a contract that was silent as to assignment or delegation by either party. Three months later, the retail outlet sold the service portion of its business to an experienced and well-financed computer service company. The only provision in the agreement between the retail outlet and the computer service company relating to the outlet's contract with the bank stated that the outlet "hereby assigns all of its computer service contracts to [the computer service company]." The computer service company performed the monthly maintenance required under the service contract. Its performance was defective, however, and caused damage to the bank's operations. Whom can the bank sue for damages arising from the computer service company's defective performance? (a) The retail outlet only, because the computer service company made no promises to the bank. (b) Either the retail outlet or the computer service company, because the bank has not released the outlet and the bank is an intended beneficiary of the outlet's agreement with the computer service company. (c) Either the retail outlet or the computer service company, because since each has the right to enforce the bank's performance of its contract with the retail outlet, mutuality of remedy renders either potentially liable for the defective performance. (d) The computer service company only, because it is a qualified and a financially responsible supplier of computer services.

Answer choice B is correct. The retail outlet effectively delegated to the computer service company the outlet's duty of performance owed to the bank. However, absent consent by the obligee (the bank) or performance by the delegate (the computer service company), an effective delegation does not relieve the delegating party (the retail outlet) of its duty to the obligee. When the computer service company purchased the service contracts, the bank also became an intended beneficiary of the computer service company's promise to the retail outlet. The computer service company expressly agreed with the retail outlet to perform the outlet's obligation to the bank, making the bank an intended beneficiary of that obligation. Thus, both the retail outlet and the computer service company remain liable to the bank. Answer choice A is incorrect because the computer service company expressly agreed with the retail outlet to perform the outlet's obligation to the bank, thus imposing a duty on the computer service company of which the bank was an intended beneficiary. Therefore, the bank is also entitled to sue the computer service company for its defective performance. Answer choice C is incorrect because, assuming an effective assignment, the right of the retail outlet to the bank's performance was extinguished. It is true that the bank can sue either the retail outlet or the computer service company, but it can do so (a) because the bank has not released the outlet and (b) because the bank is an intended beneficiary of the outlet's agreement with the computer service company. Answer choice D is incorrect because the competence and financial responsibility of the retail outlet's delegate, the computer service company, does not discharge the retail outlet's obligation to the bank.

A wholesaler contracted in a signed writing to sell to a bakery 10,000 pounds of flour each week for 10 weeks, the flour to be delivered to the bakery on Mondays and payment to be made on Wednesdays of each week. The bakery did all of its weekly bread baking on Tuesdays. On Monday morning of the first week, the wholesaler tendered delivery of 8,000 pounds of flour to the bakery, and the bakery accepted it on the wholesaler's assurance that the remaining 2,000 pounds would be delivered later that evening, which it was. The bakery paid for both deliveries on Wednesday. On Monday of the second week, the wholesaler tendered delivery of 5,000 pounds of flour to the bakery and said that the remaining 5,000 pounds could not be delivered on Monday but would be delivered by Wednesday. The bakery rejected the tender. Was the bakery legally justified in rejecting the tender of the 5,000 pounds of flour? (a)Yes, because the bakery was legally entitled to reject any tender that did not conform perfectly to the contract. (b) Yes, because the tender was a substantial impairment of that installment and could not be cured. (c) No, because the tender was not a substantial impairment of the entire contract, and the wholesaler had given assurance of a cure. (d) No, because by accepting the first 8,000 pounds on Monday of the first week, the bakery had waived the condition of perfect tender and had not reinstated it.

Answer choice B is correct. UCC § 2-612(2) provides that a buyer may reject any installment that is nonconforming if the nonconformity substantially impairs the value of that installment and it cannot be cured. The delivery of less than the contracted-for amount constituted a nonconformity that substantially impaired the value of the installment since the wholesaler could not deliver the remaining 5,000 pounds until the day after the bakery needed the flour to fulfill its weekly baking needs, and the bakery was entitled to reject the tender. Answer choice A is incorrect because, although UCC Article 2 adopts the perfect tender rule, that rule is inapplicable to assess the conformance of a seller's performance under an installment contract. Because this is an installment contract, the substantial impairment standard governs the buyer's right to reject, and the bakery is entitled to reject under that standard. Answer choice C is incorrect. A buyer's right to reject a particular installment is not dependent on a substantial impairment of the entire contract. Moreover, in this case, the wholesaler's assurance of cure was inadequate since the wholesaler could not deliver the remaining 5,000 pounds until the day after the bakery needed the flour to fulfill its weekly baking needs. Answer choice D is incorrect because a buyer's previous acceptance of a nonconforming tender does not constitute a waiver of its right to reject a later nonconforming tender if the other grounds for rejection are satisfied.

A carpenter and a homeowner entered into a valid written contract for the carpenter to design, build, and install kitchen cabinets in the homeowner's house at a cost of $25,000; the price includes the cost of materials used, but mostly reflects the cost of the carpenter's services. After execution of the contract, but before beginning performance, the carpenter realized that his initial calculations were incorrect. The carpenter phoned the homeowner, explained the matter, and demanded an additional $5,000 in order to perform the job. The homeowner orally agreed, after securing the carpenter's consent to change the style of handles and hinges to be used on the cabinets, a change that, unaware to the homeowner, resulted in a cost savings to the carpenter. The carpenter constructed and installed the cabinets. When the homeowner tendered a check for $25,000 to the carpenter, the carpenter demanded the agreed-upon contract price of $30,000. In a common-law jurisdiction, is the carpenter likely to prevail in his demand for the full $30,000? (a) Yes, because the homeowner agreed to pay the higher amount. (b) Yes, because the carpenter's duties under the contract were modified. (c) No, because the carpenter did not suffer a financial detriment. (d) No, because the oral modification violated the Statute of Frauds.

Answer choice B is correct. Unlike a contract for goods, modification of a contract for services must be supported by consideration. Where a contract has both services and goods elements, the predominant purpose test applies. Since the cost of the carpenter's services exceed those of the materials used to make the cabinets, this contract is likely to be classified as a contract for services. Consideration can be found when there is a change in a party's duties, even where that change is financially beneficial to the party. Answer choice A is incorrect because, although the homeowner did agree to pay the higher amount, this promise is unenforceable under the pre-existing duty rule unless there is variation in the carpenter's duties. In this case, the carpenter agreed to use different hardware on the cabinets. Answer choice C is incorrect because, although the carpenter did not suffer a financial detriment as a result of the change in the type of hardware to be used on the cabinets, the carpenter's duties were varied. Generally, a court will not evaluate the adequacy of consideration where the consideration is given as part of a bargained-for exchange. Answer choice D is incorrect because, although the Statute of Frauds applies to a contract that, as modified, falls with its provisions, this contract, being one for services, does not violate the Statute of Frauds. Even if the value of the materials used by the carpenter is $500 or more, the UCC Statute of Frauds provision does not apply because the contract will be classified as one for services, not goods.

A man and a woman entered into a valid contract with each other to provide transportation services for their neighbor, who recently had major surgery on her hip. The contract provided that the man and woman would each pay $100 each month for a period of one year to a local black car service. The man and woman selected the particular black car service based on recommendations that indicated it was very professional and always punctual. After one month of performance under the contract, the man refused to pay the black car service under the contract and the black car service stopped providing service to the neighbor. The woman brought an action seeking specific performance by the man pursuant to the terms of the contract. Will the woman be successful? (a) No, because contracts for personal services cannot be enforced by specific performance. (b) No, because monetary damages would be sufficient and are calculable. (c) Yes, because her damages cannot be calculated with reasonable certainty. (d) Yes, because the neighbor was an intended beneficiary of the contract.

Answer choice B is correct. When damages are an inadequate remedy, the nonbreaching party may pursue the equitable remedy of specific performance. And, when a promisor fails to pay a third-party beneficiary, the promisee, on behalf of the third-party beneficiary, can sue the promisor for specific performance of the promise. However, to do so, the remedy of specific performance must be appropriate. In determining whether the legal remedy is adequate, the court will consider a variety of factors, including the difficulty of proving damages with reasonable certainty, hardship to the defendant, balance of the equities, the wishes and understandings of the parties practicality of enforcement, and mutuality of the agreement. Here, the man and woman entered into a valid contract requiring the payment of $100 per month for a period of one year. Accordingly, the contract damages can be calculated with reasonable certainty and are an adequate remedy in this instance. Answer choice A is incorrect. Courts rarely grant specific enforcement of contracts for personal services. Although the contract was made for the provision of transportation services, the contract did not require the personal services of either the man or the woman. Therefore, specific performance would have been an available remedy if damages were inadequate. Answer choice C is incorrect. Damages can be calculated with reasonable certainty because the contract called for the payment of $100 by the man for a period of one year. Answer choice D is incorrect because, while it is true that the neighbor was an intended beneficiary of the contract, the remedy of specific performance is still inappropriate, given that money damages are an adequate remedy for the man's breach in this case.

A homeowner contracted with a landscaper to tend the extensive gardens around the homeowner's home for $1,000 per month for one year. The landscaper completed the work to the homeowner's satisfaction for three months. Two days before the fourth month, the landscaper told the homeowner that he had to leave town unexpectedly. The landscaper asked for the homeowner's permission to assign his rights and delegate his duties under the contract to another gardener for the next two months. The homeowner, frustrated by being given such short notice, reluctantly agreed to the assignment. Although the gardener consented to the delegation, he performed very little work for the next two months, and most of the gardens died. Can the homeowner recover damages against the landscaper under the contract? (a) Yes, because the landscaper did not give the homeowner adequate notice of the delegation. (b) Yes, because the homeowner's consent to the delegation did not create a novation. (c) No, because the landscaper was released from liability under the contract for two months. (d) No, because the homeowner must first seek recovery against the gardener.

Answer choice B is correct. When obligations are delegated, the delegator is not released from liability, and recovery can be had against the delegator if the delegate does not perform, unless the other party to the contract agrees to release that party and substitute a new one. Merely consenting to a delegation does not create a novation. Here, the homeowner's consent did not form a novation, and the landscaper is still liable on the contract. Answer choice A is incorrect because there is no notice requirement for an otherwise acceptable delegation of duty. Answer choice C is incorrect because the landscaper's temporary delegation of his duty did not release him from liability under the contract. Answer choice D is incorrect because there is no need for the homeowner to seek damages against the delegate before the delegator.

A driver purchased car insurance from an insurance company. The policy provided collision and comprehensive coverage, but failed to include uninsured motorist protection, as required by state law. When the driver's car suffered damage covered by the terms of the policy, the insurance company claimed that the absence of the uninsured motorist protection made the policy unenforceable. The insurance company refused to pay. Will the driver likely succeed in an action against the insurance company to recover for the damage suffered to the driver's car? (a) No, because the insurance policy violated the statute. (b) No, because it is against public policy to enter insurance contracts with insufficient coverage. (c) Yes, because it would be against public policy to allow the insurance company to avoid the contract. (d) Yes, because the policy will be strictly construed against the insurance company.

Answer choice C is correct. A contract that violates a state statute may be declared unenforceable on grounds of public policy. However, when the contract violates a policy that was intended for the benefit of a contracting party seeking relief, the contract may be enforceable in order to avoid frustrating the policy behind the statute. Here, the statute is intended to protect consumers from contractual terms they do not understand by providing them with definitions for complex legal terms. Consequently, although the contract violates the state statute, the contract is enforceable by a consumer that the statute was intended to protect. Answer choices A and B are incorrect because the general rule in answer choice A is subject to an exception; contracts will not be unenforceable when those intended to be protected by the statute would be harmed by a finding of unenforceability, as is the case here. Answer choice D is incorrect because, although it is a true rule of law, it applies to disputes involving the interpretation of contract language, which is not at issue in this question.

An experienced rancher contracted to harvest his neighbor's wheat crop for $1,000 "when the crop [was] ripe." In early September, the neighbor told the rancher that the crop was ripe. The rancher delayed because he had other customers to attend to. The neighbor was concerned that the delay might cause the crop to be lost, for hailstorms were common in that part of the country in the fall. In fact, in early October, before the crop was harvested, it was destroyed by a hailstorm. Is the rancher liable for the loss? (a) No, because no time for performance was established in the contract. (b) No, because the neighbor failed to tell the rancher that the crop might be destroyed by a hailstorm. (c) Yes, because at the time the contract was made, the rancher had reason to foresee the loss as a probable result of his breach. (d) Yes, because a party who undertakes a contractual obligation is liable for all the consequences that flow from his breach.

Answer choice C is correct. According to the Restatement (Second) of Contracts, a nonbreaching party is entitled to recover damages that the party in breach "had reason to foresee as a probable result of the breach" when the parties entered into the contract. The rancher's experience and the frequency of hailstorms in the fall combined to make the loss resulting from the rancher's breach foreseeable. Answer choice A is incorrect because the language in the contract tying the rancher's performance to when the crop was ripe was sufficient to establish the time of the rancher's performance. The rancher will be liable for the loss because such a loss was foreseeable. Answer choice B is incorrect. According to the Restatement (Second) of Contracts, reason to foresee can arise from circumstances that result in the breaching party having had actual or constructive knowledge of the loss that might result from the breach. Therefore, the neighbor's failure to specifically inform the rancher that the crop might be destroyed by a hailstorm does not determine whether the loss was foreseeable. The rancher's experience and the frequency of hailstorms in the fall combined to make the loss resulting from the rancher's breach foreseeable. Answer choice D is incorrect because the rules of contract law limit the recoverability of damages for breach. In this case, the rancher will be liable for the loss because, as previously explained, such a loss was foreseeable.

A professional basketball team had a five-year contract with a sports arena in which they played their home games. Per the contract, the team paid a monthly rental fee to rent the arena during the professional basketball season, which lasts from November through April. A sports bar that was located next door to the arena was the unofficial gathering spot for the team's fans both before and after the games. On game nights, the bar owner would decorate the entire bar with decorations bearing the team's logo, and would play only the team's game on all ten of its televisions. The bar owner also offered a 10% discount to any patron wearing a t-shirt, hat or other item of clothing bearing the team's logo. Prior to the beginning of the basketball season during the fifth year of the team's contract with the arena, the bar owner refurbished all of his booths and bar stools, and covered them with material bearing the team's colors and logo. He also purchased 10 brand new HD flat screen televisions to replace his old ones. In total, he spent $20,000, all of which he put on his credit card. Although he did not have the money to pay off the credit card, he knew he could recoup the costs once the basketball season started. Unfortunately, before the season started, the team breached its contract with the arena, and signed a contract with another sports arena approximately 20 miles away from the sports bar. As a result, the bar owner earned $30,000 less that season than he had in the preceding four seasons. If the bar owner sues the team for its breach of contract with the arena, in whose favor is the court most likely to rule? (a) For the bar owner, because he was an intended beneficiary. (b) For the bar owner, because he had an implied-in-law contract with the team. (c) For the team, because the bar owner was only an incidental beneficiary. (d) For the team, because the owner negligently spent $20,000 using his credit card.

Answer choice C is correct. An incidental beneficiary is one who benefits from a contract even though there is no contractual intent to benefit that person. An incidental beneficiary has no rights to enforce the contract. Here, the sports bar owner was an incidental beneficiary because he benefited from the contract between the basketball team and the sports arena, but neither party had any contractual intent to benefit the bar owner. Answer choice A is incorrect. In order to be an intended beneficiary of a promised performance, the promisee must have an intention (explicit or implicit) to benefit the third party, or the beneficiary is incidental. Here, there was no explicit or implicit intention to benefit the bar owner, thus the bar owner was only an incidental beneficiary. Answer choice B is incorrect. When a plaintiff confers a benefit on a defendant and the plaintiff has a reasonable expectation of compensation, allowing the defendant to retain the benefit without compensating the plaintiff would be unjust. Here, the bar owner may have conferred a benefit on the basketball team by decorating the bar with the team's colors and logo, offering discounts to fans, and only showing their games on all of his television. However, the owner could not have a reasonable expectation of compensation for doing so. There is no evidence that the team contacted him or asked him to do any of these acts, which he did of his own volition. Answer choice D is incorrect. Even if the bar owner was negligent in putting $20,000 on his credit card, if he was an intended beneficiary, he may have still had a claim for damages against the basketball team.

A plastics manufacturer saw an advertisement for a plastic extruding machine. The manufacturer contacted the merchant seller and made arrangements to inspect the machine at the seller's place of business. The manufacturer walked around the machine once and stated: "Yes, this looks like what I need." When the manufacturer asked the price, the seller stated a price that was less than half the amount a similar, functioning, used machine commanded on the market. The manufacturer was surprised at the low price, but did not inquire as to the reason. The seller encouraged the manufacturer to perform a closer inspection before finalizing the purchase and offered to open the motor housing so that the motor could be examined, but the manufacturer declined. The parties completed the sale. The manufacturer transported the extruding machine to his factory. When it arrived, he first learned that the motor was burned out and required complete replacement, as was readily apparent upon visual inspection of it. Replacement of the motor would cost roughly the amount the manufacturer had paid for the machine. The manufacturer contacted the seller to return the machine, but the seller refused. The manufacturer filed suit against the seller. Will the manufacturer prevail? (a) Yes, because the seller violated the implied warranty of merchantability by selling a machine with a burned-out motor. (b) Yes, because the manufacturer's unilateral mistake regarding the condition of the machine was caused by the seller. (c) No, because the seller made no claims regarding the operability of the machine. (d) No, because the manufacturer waived any implied warranties by failing to inspect the machine.

Answer choice D is correct. Under UCC Article 2, a warranty of merchantability is implied whenever the seller of goods is a merchant who deals in goods of the kind sold. To be merchantable, goods must be fit for their ordinary purpose and pass without objection in the trade. A breach of this warranty must have been present at the time of the sale. However, if the buyer, before entering into the contract, has examined the goods as fully as the buyer desires, or has refused to examine the goods, there is no implied warranty with respect to defects that an examination would have revealed to the buyer. Here, the manufacturer declined to closely inspect the machine, even after learning of the unusually low sales price. Had he done so, he would have discovered the damaged motor when he opened the motor housing, which the seller had offered to do. Because the manufacturer refused to examine the goods, he waived any implied warranty of merchantability that would have otherwise attached to the sale. Answer choice A is incorrect because the manufacturer waived the implied warranty of merchantability when he declined to inspect the machine. Answer choice B is incorrect because the seller did not cause the manufacturer's unilateral mistake; rather, the seller offered to open the motor housing, and the manufacturer declined the offer. Answer choice C is incorrect because the warranty of merchantability is implied in all sales of goods by merchants who deal in goods of the kind sold, and need not be expressly stated by the seller. If the defect could not have been uncovered by a reasonable inspection, the manufacturer would have had a valid claim against the seller for violation of the implied warranty of merchantability, even though the seller had not expressly made any such claims.

At the beginning of an auction, which consists of 25 lots of goods owned by the same person, the auctioneer announced that the right of the seller to withdraw goods was not reserved. After the sales of two lots were completed but prior to the auctioneer's call for bids on the third lot, the owner sought to withdraw the third lot from the auction. Can the owner properly withdraw the third lot? (a) No, because the auctioneer had announced that the auction was without reserve. (b) No, because sales of other lots owned by the same person had been completed. (c) Yes, because the auctioneer has not called for bids on the third lot. (d) Yes, because the auctioneer has not announced completion of the sale of the third lot.

Answer choice C is correct. Answer choice C is correct. In an auction, each lot is considered to be a separate sale. In a no-reserve auction, after the auctioneer calls for bids on the goods in a lot, the goods cannot be withdrawn unless no bid is received within a reasonable time. With regard to the third lot, because the auctioneer had not called for bids on that lot, the goods can be withdrawn. Answer choice A is incorrect. Even though the right of the seller to withdraw goods is not reserved, a seller possesses the right to withdraw the goods until the auctioneer has called for bids on the goods. Answer choice B is incorrect. When goods are auctioned in lots, each lot is a separate sale. Consequently, the fact that other lots owned by the same person have been sold in the same auction does not affect the right of the owner of a lot to withdraw that lot from the auction. Answer choice D is incorrect because it states the rule for withdrawal of goods from an auction where the auction is with reserve.

A manufacturer of disposable foam products entered into a written contract with a take-out restaurant to sell them 5,000 disposable foam food containers per month at 5 cents per food container for a two-year period. Per the contract, payment of $250 was due once a month. The restaurant was also responsible for paying shipping charges, which were payable on a quarterly basis. Finally, the contract stated that if any of the containers were discovered to be damaged, they could be returned for a full refund. However, the contract was silent with regard to the return of any containers that were not damaged. In the last month of its contract with the manufacturer, the restaurant decided to switch to biodegradable, recycled cardboard food containers because its take-out customers complained about the harmful effects of disposable foam products on the environment. The restaurant asked the manufacturer not to ship the final shipment, but the manufacturer refused. The restaurant had not yet made the $250 payment for the final shipment of containers, but it did owe the manufacturer delivery charges for its final quarter. The restaurant returned all of the containers to the manufacturer. In addition, the restaurant sent an envelope with cash for the final shipping charges. In the envelope, the restaurant included a brief memo stating that the cash included was for payment in full, including all food containers and shipping charges. The manufacturer's accounting department received the envelope, read the memo, and deposited the cash. The manufacturer then sued the restaurant for the final payment of $250. Which of the following legal concepts would be the strongest argument against payment of $250? (a) Novation (b) Good-faith modification (c) Estoppel (d) Accord and satisfaction

Answer choice D is correct. Under an accord agreement, a party to a contract agrees to accept a performance from the other party that differs from the performance that was promised in the existing contract, in satisfaction of the other party's existing duty. When a party agrees to accept a lesser amount in full satisfaction of its monetary claim, there must be consideration or a consideration substitute for the party's promise to accept the lesser amount. Consideration can exist if the other party honestly disputes the claim or agrees to forego an asserted defense. Here, there was an "accord and satisfaction" that discharged both the original contract and the accord contract. There was a dispute about whether the undamaged food containers could be returned, and the manufacturer's deposit of the cash (i.e., satisfaction) sent by the restaurant with the memo indicating payment in full discharges any duties of the restaurant. Answer choice A is incorrect. A novation is the substitution of a new contract for an old one when the original obligor is released from his promises under the original agreement. This is not the case here. Answer choice B is incorrect. Under the UCC, no consideration is necessary to modify a contract; however, good faith is required. Here, the good-faith dispute did not suspend the duty of the restaurant. The accord and satisfaction discharged both the original contract and the accord contract. Answer choice C is incorrect. A party who indicates that a condition will not be enforced may be estopped from using that condition as a defense if the other party reasonably relied on the impression that the condition had been waived. There is no estoppel issue here, but rather an accord and satisfaction, which discharges any contractual duties of the restaurant.

Due to recent financial difficulties, a man asked his brother if he would be willing to loan him $3,000. The brother agreed to make the $3,000 loan to the man. Under the terms of their agreement entered into on December 31, the man would be responsible for making monthly payments of $125 plus interest for the next two years at the beginning of each month, starting January 1. Pursuant to the agreement with his brother, the man made payments each month for the first six months. However, he failed to make the agreed-upon payments for July and August. On August 30, the man told his brother that he could no longer afford to repay him. The brother filed suit against the man on August 31 for breach of contract. What amount, if any, will the brother be able to recover on August 31? (a) $2,250 plus interest, the amount outstanding on the loan because the man repudiated the contract. (b) $375 plus interest, the amount owed for July, August, and September. (c) $250 plus interest, the amount owed for July and August. (d) Nothing, because the agreement did not contain an acceleration clause.

Answer choice C is correct. If at the time of a breach the only remaining duties of performance are (i) those of the party in breach and (ii) for the payment of money in installments not related to one another, then breach by nonperformance as to less than the whole, whether or not accompanied or followed by a repudiation, does not give rise to a claim for damages for total breach and is a partial breach of contract only. In this case, on August 31, the brother is only entitled to recover the amount due for July and August, as the other installments are not due yet. Answer choice A is incorrect. Even though the man repudiated the entire contract by telling his brother he could no longer afford to repay him, the brother cannot recover the entire amount owed under the agreement and is limited to the installments that are currently outstanding as of August 31. Answer choice B is incorrect because the payment for September is not due yet. Answer choice D is incorrect. Although an acceleration clause would be required for the brother to recover the entire amount of the loan, the absence of an acceleration clause does not preclude the brother from recovering the amount currently due and outstanding under the agreement.

A woman contracted with a tennis instructor to provide tennis lessons for her friend as a gift. The woman agreed to pay the instructor's fee of $100 per lesson for a total of 10 lessons over the course of a month. The next day, the woman informed the friend of the gift. The friend thanked the woman and told her that she would never have tried tennis on her own. The friend immediately went to the sporting goods store and purchased a racket, tennis shoes, and various articles of tennis clothing, at a total cost of $500. She called the woman to tell her about the items she had purchased and also called the tennis instructor to make sure she had all the items she would need for the lessons. A day before the first lesson was scheduled, the woman and the tennis instructor rescinded their agreement. The friend sued the tennis instructor for breach of contract. Will the friend likely prevail against the tennis instructor? (a) No, because the friend, as the intended recipient of a gift, cannot enforce the contract. (b) No, because the woman and the tennis instructor could validly rescind the contract. (c) Yes, because the friend's rights in the contract vested when she purchased $500 of tennis equipment in reliance on the contract. (d) Yes, because the friend was an incidental beneficiary of the contract between the woman and the tennis instructor.

Answer choice C is correct. In general, an intended beneficiary is one to whom the promise of the performance will satisfy the obligation of the promisee to pay money to the beneficiary or the circumstances indicate that the promisee intends to give the beneficiary the benefit of the promised performance. An intended beneficiary of a "gift promise" (a donee beneficiary) may sue only the promisor. First, the promisee must have an intention (explicit or implicit) to benefit the third party, or the beneficiary is incidental. Here, the woman (promisee) intended that the friend benefit from the contract with the tennis instructor (promisor). However, the right of an intended beneficiary will not prevent the parties to the contract from modifying or rescinding the agreement unless the intended beneficiary's right vests. The interest vests when the beneficiary materially and justifiably relies on the rights created, manifests assent to the contract at one of the party's request, or files a lawsuit to enforce the contract. Once the beneficiary's rights have vested, the original parties to the contract are both bound to perform the contract. Any efforts by the promisor or the promisee to rescind or modify the contract after vesting are void, unless the third party agrees to the rescission or modification. In this case, the friend was an intended beneficiary of the contract between the woman and the tennis instructor. Her rights as an intended beneficiary vested when she purchased $500 of tennis equipment in reliance on the contract. Accordingly, she can enforce the contract against the tennis instructor. Answer choice A is incorrect because an intended beneficiary of a gift is entitled to enforce the contract once her rights have vested. Answer choice B is incorrect. Once the friend's rights vested, the woman and the tennis instructor were bound to perform under the contract and could not rescind the contract unless the friend agreed to the rescission. Answer choice D is incorrect because the friend was an intended, not incidental, beneficiary of the contract as the woman entered into the contract with the intention of providing a benefit to the friend. Moreover, if the friend was an incidental beneficiary, she would not have the right to enforce the contract and thus would not prevail.

A grandson asked to borrow $2,000 from his grandmother. She told him that she did not have the money but that she would ask a friend to lend him the money. The friend agreed on the condition that the grandmother would guarantee the loan. After the grandmother sent an unsigned letter to her friend that outlined the terms of the loan and her agreement to guarantee its repayment, the friend made the loan. When the loan came due and the grandson was unable repay it, the friend demanded that the grandmother make good on her promise. When she refused to do so, the friend filed an action in small claims court against the grandmother. What is the grandmother's best defense to this action? (a) The friend did not give the grandmother notice that he had accepted her offer to guarantee the loan. (b) The Statute of Frauds prevents enforcement of the grandmother's promise because of the amount of the loan. (c) The grandmother did not sign the letter containing her promise to guarantee the loan. (d) The grandmother did not receive consideration in exchange for her promise to guarantee the loan.

Answer choice C is correct. Suretyship is a three-party contract, wherein one party (the surety) promises a second party (the obligee) that the surety will be responsible for any debt or other obligation of a third party (the principal) resulting from the principal's failure to pay as agreed. Under the Statute of Frauds, a suretyship contract, or contract to answer for the debt of another, must be signed by the party to be charged. Here, because the grandmother had not signed the letter that contained her promise to act as a surety of the loan, the Statute of Frauds would prevent enforcement of her promise. Answer choice A is incorrect. Although generally an acceptance must be communicated to the offeror, because the grandmother's offer did not restrict the manner of acceptance, the friend's acceptance by performance (i.e., lending the money to the grandson) was permissible. With respect to an offer of suretyship, the surety does not need to be notified of the obligee's performance unless the terms of the offer specifically require it. Answer B is incorrect. Although a contract for the sale of goods must be in writing if the purchase price is $500 or more, there is no monetary limitation on an oral contract that does not fall within the confines of the UCC, such as the loan agreement in this case. Answer choice D is incorrect because the grandmother did not have to receive a benefit in order for there to be consideration under the contract. Consideration requires that some legal detriment be bargained for in order for a contract to exist. The exchange must be of some benefit to the promisor or detriment to the promisee. Here, the payment of $2,000 to the grandson was a detriment to the friend as promisee.

A janitorial service contracted in writing with a hospital for a one-year term. Under the terms of the contract, the janitorial service agreed to clean the hospital daily in accordance with the hygiene standards of the city's health code. Because the janitorial service did not clean a patient's room in accordance with the required hygiene standards, the patient contracted an infection that required continued hospitalization. In addition to suing the hospital, the patient sued the janitorial service for breach of contract. Which of the following statements is most accurate with respect to the breach of contract claim against the janitorial service? (a) The janitorial service is liable to the patient as a matter of public policy, because it violated the city's health code. (b) The patient is an intended third-party beneficiary under the contract, because the janitorial service's promise was intended to benefit all hospital patients. (c) The patient has no claim for breach of contract against the janitorial service, because she is an incidental beneficiary. (d) The patient cannot sue on the contract, because she was not named in the contract.

Answer choice C is correct. The patient cannot recover because she is an incidental beneficiary rather than an intended third-party beneficiary. The circumstances fail to indicate that the hospital intended to give the patient the benefit of the promised performance. Answer choice A is incorrect because a violation of public policy may render a contract unenforceable. Here, however, the patient's ability to recover against the janitorial service will be governed by a third-party beneficiary theory, under which she would be an incidental beneficiary and therefore not eligible to recover damages. Answer choice B is incorrect because, as previously explained, the patient cannot recover because she is an incidental beneficiary rather than an intended third-party beneficiary. Answer choice D is incorrect because an intended third-party beneficiary acquires rights under a contract regardless of whether the party is identified as an intended beneficiary at the time the contract is made. In this case, however, the patient is an incidental beneficiary rather than an intended third-party beneficiary, and therefore she cannot recover on a breach of contract claim.

A bank agreed to lend a merchant $10,000 for one year at 8% interest. The loan proceeds were to be disbursed within two weeks. The merchant intended to use the loan proceeds to purchase a specific shipment of carpets for resale at an expected profit of $5,000 but said nothing about these plans to the bank. The bank failed to disburse the proceeds and refused to assure the merchant that it would do so. The merchant was able to secure a loan from another lender at 10% interest for one year. However, by the time the merchant started the application process for a substitute loan, it was too late to pursue the opportunity to buy the shipment of carpets. In an action against the bank for breach of contract, which of the following amounts is the merchant likely to recover? (a) Nothing, because damages lost opportunities are not recoverable. (b) Nothing, because the parties failed to tacitly agree that the merchant would be entitled to damages in the event of a breach by the bank. (c) You Selected: The difference in cost over time between a loan at 10% and a loan at 8%. (d) $5,000, the merchant's foreseeable loss.

Answer choice C is correct. The presumed wide availability of credit limits a borrower's recovery to the additional cost of obtaining a loan from another lender. In the typical case, a lender has no reason, at the time of contract formation, to foresee that the borrower will not be able to obtain a substitute loan. Furthermore, at the time of contract formation under these facts, the bank was not aware and had no reason to be aware of the merchant's opportunity to purchase the carpets. Therefore, the merchant's recovery is limited to the difference in cost over time between a loan at 10% and a loan at 8%. Accordingly, answer choice D is incorrect. Answer choice A is incorrect because damages for lost opportunities may be recoverable in some cases as consequential damages for a breach of contract. However, consequential damages for the lost opportunity to purchase the carpets are not appropriate under these facts because (1) of the presumed wide availability of loans and (2) the fact that the bank had no reason to be aware of the merchant's ability to purchase the carpets. The proper measure of damages, therefore, is the additional cost of obtaining a loan from another lender. Answer choice B is incorrect. The tacit agreement test has been rejected by courts and by the UCC.

A manufacturer of hardwood furniture entered into an enforceable written contract with a buyer for an oak dining room set. The contract price for the set was $5,000, and the manufacturer expected a $1,000 profit on the sale. A week before the buyer was to pay for and receive the dining room set, the buyer unjustifiably repudiated the contract. The manufacturer delivered the set to one of its local showrooms for resale, and two weeks later, the manufacturer sold the set to another purchaser for $5,000. The manufacturer has shown that this particular furniture set is very popular, and the manufacturer had a large inventory of the furniture ready for sale at the time of each relevant transaction. If the manufacturer sues the buyer for breach of contract, what is the most likely amount the manufacturer can recover? (a) Nothing, because the manufacturer did not notify the buyer of its intent to resell the furniture set. (b) Only incidental damages, because the resale price for the furniture set was equal to the contract price. (c) $1,000 plus any incidental damages, because the manufacturer is a volume seller. (d) $5,000 plus any incidental damages, because the buyer unjustifiably repudiated the contract.

Answer choice C is correct. To qualify as a "lost volume" seller, the seller needs to show only that it could have supplied both the breaching purchaser and the resale purchaser with the goods. Because the manufacturer had an adequate inventory of this furniture set to meet the demands of both purchases, the manufacturer is a "lost volume" seller and can recover the lost profits of the sale along with any incidental damages. Answer choice A is incorrect because such notice is only required if the seller elects to privately resell wrongfully rejected goods and sue for the contract price minus the resale price. Here, the manufacturer may instead sue for lost profits as a volume seller without providing such notice. Answer choice B is incorrect because the manufacturer qualifies as a lost volume seller and may recover its lost profits for the sale. Answer choice D is incorrect because the buyer's repudiation does not entitle the manufacturer to the entire contract price, as the manufacturer was able to resell the dining room set at the same price.

A developer contracted in writing to sell to a buyer a house on a one-acre lot for $100,000. The developer told the buyer that the lot abutted a national park and that the water for the house came from a natural artesian spring. The developer knew that both of these representations were important to the buyer and that both were false. The buyer moved into the house and eight months later learned that a private golf course was being constructed on the adjacent land and that the water for his house was piped in from the city reservoir. The buyer immediately sued the developer to avoid the contract. The construction of the golf course will probably increase the market value of the buyer's property, and the water from the city reservoir exceeds all established standards for drinking water. Is the buyer likely to prevail? (a) No, because eight months exceeds a reasonable time for contract avoidance. (b) No, because the developer's misstatements caused no economic harm to the buyer. (c) Yes, because the contract was void ab initio. (d) Yes, because the buyer retained the power to avoid the contract due to

Answer choice D is correct. A party may avoid a contract to which the party's assent was induced by fraud. The time period within which a party may avoid a contract due to misrepresentation does not begin to run until that party either knows or has reason to know of the misrepresentation. In this case, the buyer's eight-month delay will not preclude him from avoiding the contract because he did not learn of the misrepresentation until eight months after the parties entered into the contract; the buyer immediately sought to avoid the contract after learning of the misrepresentation. Answer choice A is incorrect because, as previously stated, the time period within which a party may avoid a contract due to a misrepresentation does not begin to run until that party either knows or has reason to know of the misrepresentation. Answer choice B is incorrect because a party may avoid a contract to which the party's assent was induced by fraud even if the resulting transaction is fair on its terms. Because the buyer immediately sought to avoid the contract after learning of the misrepresentation, he will likely prevail. Answer choice C is incorrect because "void ab initio" means void from the outset and an agreement to which assent is induced by a misrepresentation is not void. Rather, it is voidable, which permits the party whose assent was induced by the misrepresentation to avoid the agreement.

A seller entered into a contract to sell to a buyer a house for a price of $150,000. The contract contained the following clause: "This contract is conditional on the buyer's securing bank financing at an interest rate of 7% or below." The buyer did not make an application for bank financing and therefore did not secure it, and refused to proceed with the purchase. The seller sued the buyer for breach of contract. Is the seller likely to prevail? (a) No, because the buyer did not secure bank financing. (b) No, because the contract did not expressly impose on the buyer any obligation to apply for bank financing. (c) Yes, because a court will excuse the condition to avoid a disproportionate forfeiture. (d) Yes, because a court will imply a term imposing on the buyer a duty to use reasonable efforts to secure bank financing.

Answer choice D is correct. A performance that is subject to an express condition cannot become due unless the condition occurs or its nonoccurrence is excused. The duty of good faith imposed an obligation on the buyer to make reasonable efforts to secure bank financing. Accordingly, the failure of the condition (in this case, the bank financing commitment) to occur did not discharge the buyer's performance obligation. For these reasons, answer choices A and B are incorrect. Answer choice C is incorrect. Although the nonoccurrence of a condition may be excused in order to avoid disproportionate forfeiture, the excuse is inapplicable here because there was no forfeiture, which is defined as "the denial of compensation that results when the obligee loses his right to the agreed exchange after he has relied substantially, as by preparation or performance, on the expectation of that exchange." There is no evidence here of any such reliance.

A charity, seeking to raise funds, held a legally permitted raffle in which the prize is a new automobile. A week before the raffle, the organizer of the raffle contacted a friend who had purchased a raffle ticket. The organizer promised to ensure that the friend would win the raffle if the friend gave the organizer $1,000. The friend agreed and gave the organizer $1,000. On the day before the raffle, the friend began to feel guilty. He went to the organizer, renounced the scheme, and demanded his $1,000 back. The organizer refused. The next day at the raffle, the automobile was awarded to someone else. The applicable jurisdiction makes it a crime to fraudulently conduct a contest, lottery, or prize drawing. Can the friend successfully bring a legal action against the organizer? (a) No, because the agreement between the friend and the organizer was illegal. (b) No, because the friend failed to take any action to prevent the raffle from being held. (c) Yes, because there was a valid contract between the organizer and the friend. (d) Yes, because the friend is entitled to a return of the $1,000 paid to the organizer.

Answer choice D is correct. Although the friend is not entitled to enforce the illegal agreement between him and the organizer, the friend is entitled to restitution with regard to the $1,000 paid to the organizer since he withdrew from the transaction before the raffle was held and did not engage in serious misconduct. For this reason, answer choice A is incorrect. Answer choice B is incorrect because, even though the friend's failure to take action to prevent the raffle from being held may result in criminal liability, the friend's action in withdrawing from the transaction before the illegal purpose was accomplished does give rise to the right to restitution. Answer choice C is incorrect because, although there was a valid offer and acceptance and valuable consideration was given by both the friend and the organizer, the contract was made for an illegal purpose and therefore is unenforceable.

At the auction of a rare automobile that had been owned by an infamous criminal, two individuals, a museum owner and a private collector, became involved in a bidding war. After a protracted exchange of bids, the museum owner declined the auctioneer's invitation to bid a higher amount than the private collector had. However, as the auctioneer was bringing down his hammer to announce the sale of the automobile to the private collector, the museum owner called out a higher bid before the hammer struck the sounding block on the auctioneer's desk. The auctioneer acknowledged the museum owner's bid and called for other bids. A third party placed a bid, which unmatched by either the museum owner or the private collector, was subsequently accepted by the auctioneer as the winning bid. The museum owner and private collector challenged the third party's ownership of the automobile. Who is the rightful owner of the automobile? (a) The private collector, based on the private collector's last bid before the fall of the hammer. (b) The private collector, based on the museum owner's refusing the auctioneer's invitation to bid prior to the fall of the hammer. (c) The museum owner, based on the museum owner's bid made while the hammer was falling. (d) The third party.

Answer choice D is correct. An auction sale is complete when the auctioneer announces its end, such as by the fall of the auctioneer's hammer or in any other customary way. When a bid is made contemporaneously with the falling of the hammer, the auctioneer may, at his discretion, treat the bid as continuing the bidding process or declare the sale completed at the fall of the hammer. In this case, the museum owner made his bid contemporaneously with the falling of the hammer. The auctioneer chose, at his discretion, to treat the museum owner's bid as continuing the bidding process. The third party subsequently made a bid that was unmatched. Therefore, because the third party's bid was accepted by the auctioneer as the winning bid, the third party is entitled to the automobile. Answer choice A is incorrect. Although the auctioneer could have elected to award the automobile to the private collector based on the collector's last bid made before the fall of the auctioneer's hammer, the auctioneer was not required to do so. Answer choice B is incorrect. Although the museum owner did decline the auctioneer's invitation to bid prior to the fall of the hammer, this failure to bid at that time did not preclude the auctioneer from recognizing the museum owner's subsequent bid. Answer choice C is incorrect because, although the auctioneer had the option to recognize the museum owner's bid that was made while the hammer was falling, the auctioneer could not automatically accept this bid as the final bid. Instead, having recognized this bid, the auctioneer was required to treat it as a continuation of the bidding.

A woman was interested in creating custom wedding invitations for her upcoming wedding and entered into a written contract with a graphic designer for the production of 400 invitations. Three weeks after the contract was executed, the graphic designer informed the woman that he had sold his design shop and transferred all of his current contracts to another artist of equal skill and popularity. Reluctantly, the woman agreed to performance by the other artist. When she received the finished product, the woman discovered that the date of the wedding was incorrect and her fiancé's name was spelled incorrectly. The woman brought an action for breach of contract against the graphic designer. Is the woman likely to prevail in her action for breach of contract against the graphic designer? (a) No, because the graphic designer did not make the error on the invitations. (b) No, because the woman consented to a novation that released the graphic designer from liability. (c) Yes, because she did not receive consideration for the delegation to the other artist. (d) Yes, because the graphic designer remained liable under the contract.

Answer choice D is correct. Generally, obligations under a contract can be delegated. When obligations are delegated, the delegator is not released from liability, and recovery can be had against the delegator if the delegate does not perform, unless the other party to the contract agrees to release that party and substitute a new one (a novation). In this case, the graphic designer delegated his obligations under the contract with the woman to the other artist. When she received the finished invitations from the other artist, the date of the wedding was incorrect and her fiancé's name was spelled incorrectly. Because the other artist (the delegate) did not properly perform, the woman can seek recovery against the graphic designer (the delegator), who remained liable under the contract. Answer choice A is incorrect. The woman can seek recovery from the graphic designer, even though he did not make the error, because the graphic designer remained liable under the contract. Answer choice B is incorrect. A novation is the substitution of a new contract for an old one when the original obligor is released from his promises under the original agreement. A novation may be express or implied after delegation if (i) the original obligor repudiates liability to the original promisee and (ii) the obligee subsequently accepts performance of the original agreement from the delegate without reserving rights against the obligor. Merely consenting to a delegation does not create a novation. Here, the mere fact that the graphic designer delegated his obligations under the contract to the other designer and the woman consented did not effect a novation. Answer choice C is incorrect because no consideration is necessary for the delegation to be effective.

On June 1, a general contractor and a subcontractor entered into a contract under which the subcontractor agreed to deliver all of the steel joists that the general contractor required in the construction of a hospital building. The contract provided that delivery of the steel joists would begin on September 1. Although the general contractor had no reason to doubt the subcontractor's ability to perform, the general contractor wanted to be sure that the subcontractor was on track for delivery in September. He therefore wrote a letter on July 1 to the subcontractor demanding that the subcontractor provide assurance of its ability to meet the September 1 deadline. The subcontractor refused to provide such assurance. The general contractor then immediately obtained the steel joists from another supplier. If the subcontractor sues the general contractor for breach of contract, is the subcontractor likely to prevail? (a) No, because the subcontractor anticipatorily repudiated the contract when it failed to provide adequate assurance. (b) No, because the contract failed to specify a definite quantity. (c)Yes, because a demand for assurance constitutes a breach of contract when the contract does not expressly authorize a party to demand assurance. (d) Yes, because the subcontractor's failure to provide assurance was not a repudiation since there were no reasonable grounds for the general contractor's insecurity.

Answer choice D is correct. The adequate assurance doctrine requires that a party respond to a demand for adequate assurance only if the demand is reasonable and justified. A demand is justified if the demanding party has reasonable grounds for insecurity with respect to the other party's potential performance. The facts in this case state that the general contractor had no reason to doubt the subcontractor's ability to perform. Therefore, the general contractor was unjustified in demanding adequate assurance, and the subcontractor properly refused to respond to the demand. Answer choice A is incorrect because, as stated above, the contractor had no reason to doubt the subcontractor's performance, so the subcontractor was under no obligation to provide assurance. Answer choice B is incorrect because the parties entered into a requirements contract that under UCC § 2-306(1) will not fail for indefiniteness. Answer choice C is incorrect because the adequate assurance doctrine is a UCC implied term that arises by operation of law. Therefore, the doctrine's applicability is not dependent upon the express authority of the party seeking assurance.

A farmer contracted to sell 100,000 bushels of wheat to a buyer. When the wheat arrived at the destination, the buyer discovered that the farmer had delivered only 96,000 bushels. The buyer sued the farmer for breach of contract. At the trial of the case, the court found that the written contract was intended as a complete and exclusive statement of the terms of the agreement. The farmer offered to prove that in the wheat business, a promise to deliver a specified quantity is considered to be satisfied if the delivered quantity is within 5% of the specified quantity. The buyer objected to the offered evidence. Is the court likely to admit the evidence offered by the farmer? (a) No, because the offered evidence is inconsistent with the express language of the agreement. (b) No, because the written contract was totally integrated. (c) Yes, because the offered evidence demonstrates that the farmer substantially performed the contract. (d) Yes, because the offered evidence explains or supplements the agreement by usage of trade.

Answer choice D is correct. This transaction involves a sale of goods and is subject to UCC Article 2. Under Article 2, evidence of trade usage that can be construed as reasonably consistent with an agreement's express language is admissible to interpret or supplement an agreement. The majority rule provides that trade usage will be viewed as consistent with an agreement's express language unless the usage completely negates specific express language. The trade usage allowing for a variation of up to 5% does not completely negate but rather qualifies the express language calling for the delivery of 100,000 bushels of wheat. Accordingly, answer choice A is incorrect. Answer choice B is incorrect because evidence of trade usage is admissible to explain or supplement an agreement even if the parties intended the agreement to be completely integrated. Answer choice C is incorrect. Under UCC § 2-601, the applicable standard for determining breach of a non-installment transaction involving a sale of goods is the perfect tender rule and not the substantial performance standard. However, the offered evidence of trade usage would establish that there was no breach; according to the trade usage standards, the farmer's delivery of 96,000 bushels was performance that conformed to the agreement.

A new men's apparel company entered into a contract with a well-known former mayor to make appearances at different retail stores throughout the country to promote the company's line of men's suits. Knowing that the former mayor's sponsorship had helped other new companies in the area build a consumer base, the company agreed to pay the former mayor $10,000 per appearance. Prior to the former mayor's first appearance, a news story was released that alleged the company engaged in discriminatory practices in the workplace. Despite the company's truthful assertions that the news story was completely false, the former mayor refused to make any appearances under the contract. The company was unable to find another famous public figure to make the appearances and, as a result, its line of men's suits was a complete failure. The company subsequently sued the former mayor for breach of contract. At trial, the company's attorney presented reliable uncontested evidence calculating the lost profits sustained by the company due to the former mayor's failure to make the appearances. Will the company likely be able to recover damages for lost profits? (a) No, because the damages for lost profits were not foreseeable. (b) No, because the lost profits of a new business cannot be recovered. (c) Yes, because the former mayor anticipatorily repudiated the contract. (d) Yes, because the lost profits of a new business are recoverable if proved with reasonable certainty.

Answer choice D is correct. To recover damages, a plaintiff must prove the dollar amount of the damages with reasonable certainty. Courts are hesitant to award damages for lost profits, as they are difficult to prove. When lost profits are considered too speculative, such as with a new venture, courts often limit a party's recovery to reliance damages (i.e., reasonable expenditures made in connection with the contract). However, in this case, the facts provide that the company's attorney presented reliable and uncontested evidence regarding the company's lost profits due to the former mayor's breach. This evidence is sufficient to prove those damages with reasonable certainty, especially as the evidence was uncontested. Accordingly, although the company is a new business, on these facts, it can recover its lost profits. Answer choice A is incorrect. Damages are recoverable if they were the natural and probable consequences of breach. Here, it was foreseeable that the former mayor's failure to appear at the retail stores would result in significantly lower sales numbers for the company's new line of suits. Answer choice B is incorrect because lost profits of a new business can be recovered if proved with reasonably certainty. Answer choice C is incorrect. Although the former mayor's failure to appear constituted an anticipatory repudiation of the contract, that fact would entitle the company to damages, but not necessarily lost profits. The reason lost profits are recoverable here is because they were proved with reasonable certainty by the company's attorney. Therefore, answer choice D is the best answer choice.

A motorcyclist and his friend were discussing an old motorcycle that the motorcyclist was interested in selling. His friend demonstrated an interest in purchasing the motorcycle, and the two began to negotiate a possible purchase price. After a couple hours of negotiations, the motorcyclist and his friend orally agreed upon a price of $8,500. They also agreed that their oral agreement should be put into writing. The motorcyclist and his friend had their mutual friend, an attorney, draw up a simple contract for the sale of the motorcycle. Without reading the contract, the motorcyclist and his friend both signed it. Unbeknownst to both of them, the price quoted in the contract was $850 instead of $8,500 due to an error by the attorney. If the friend requests that the contract be reformed, is reformation of the contract an appropriate remedy in this case? (a) No, because both parties must agree to the reformation of a written contract. (b) No, because the formal written contract controls over the oral agreement. (c) Yes, because at least one party was mistaken as to the terms of the contract. (d) Yes, because both the motorcyclist and the friend were mistaken as to the price term in the written contract.

Answer choice D is correct. When a writing fails to express the agreement because of a mistake of both parties, the court may, at the request of a party, reform the writing to express the agreement. Reformation of a writing for mistake is available if there was a prior agreement (either oral or written) between the parties, there was an agreement by the parties to put that prior agreement into writing, and as a result of a mistake, there is a difference between the prior agreement and the writing. Here, the attorney's error caused the formal written contract to fail to express the parties' actual agreement, as evidenced by their prior oral agreement following negotiations. As both parties were mistaken as to the contents of the formal written agreement concerning the price, reformation would be an appropriate remedy. Answer choice A is incorrect. Reformation for mistake can be granted on the request of one party, as long as the requirements for reformation are met. Answer choice B is incorrect because the written contract can be reformed based on the mistake of both parties as to the contents of the contract. Answer choice C is incorrect because, although reformation only needs to be requested by one party, reformation is only available if both parties are mistaken as to the contents or effect of the writing.

A groom left his bride at the altar on the day of her wedding. The bride could not bear to keep any painful reminders of the occasion, so she offered to sell her wedding dress to one of her bridesmaids for $5,000. The bride stated that the offer would remain open for 30 days. The bridesmaid said she was interested but would have to think about it. A week later, the bridesmaid emailed the bride to ask if the price included a custom-made veil that the bride had worn. The bride did not respond to the bridesmaid's question. Within the 30-day period, the bridesmaid accepted the bride's initial offer of $5,000 for the wedding dress. In response, the bride stated that the bridesmaid could only buy the wedding dress for $6,000. Was a contract formed when the bridesmaid accepted the initial offer of $5,000? (a) Yes, because the bridesmaid's question did not constitute a counteroffer. (b) Yes, because the bride was required to keep the initial offer open for the 30-day period. (c) No, because the bridesmaid's question acted as a counteroffer and rejection of the $5,000 offering price. (d) No, because the bride raised the price of the dress to $6,000.

Answer choice A is correct. A counteroffer is an offer made by an offeree to the offeror relating to the same matter as the original offer and proposing a substituted bargain differing from that proposed by the original offer. An offeree's power of acceptance is terminated by the making of a counteroffer. A counteroffer acts as a rejection of the original offer and creates a new offer. However, mere suggestions or inquiries, including requests for clarification or statements of intent, made in a response by the offeree do not constitute a counteroffer. Here, the bridesmaid asked if the $5,000 price included the custom-made veil. This question was merely a request for clarification as to what was included in the offer; it was not a counteroffer. Therefore, the bridesmaid could accept the original offer of $5,000 for the wedding dress within the 30-day period. Answer choice B is incorrect because the bride was not required to keep the offer open for 30 days. In general, an offer can be revoked by the offeror at any time prior to acceptance. An offer is revoked when the offeror makes a manifestation of an intention not to enter into the proposed contract. In this case, the bride could have revoked the offer at any time prior to the expiration of 30 days, provided that she communicated the revocation to the bridesmaid. Importantly, the bridesmaid did not provide consideration to keep the offer open and the offer was not otherwise irrevocable. Answer choice C is incorrect because, as stated above, the bridesmaid's question about the custom-made veil was merely a request for clarification, not a counteroffer. Thus, it did not constitute a rejection. Answer choice D is incorrect because although the bride could have raised the price of the wedding dress from $5,000 to $6,000, she did not attempt to do so until after the bridesmaid accepted the original offer and created an enforceable contract.

On May 10, the coach of a youth league baseball team sent a letter to a supplier asking the supplier to promptly ship 20 red jerseys to him. On May 15, the supplier received this letter and sent the coach a reply letter accepting the offer. On May 16, the supplier realized that he had no red jerseys with which to fill the order, and sends the coach 20 blue jerseys with a note that the blue jerseys were tendered as an accommodation. The coach received the jerseys and accommodation note on May 18, and received the supplier's acceptance letter on May 19. On May 20, which of the following is a correct statement of the parties' legal rights and duties? (a) The coach can either accept or reject the blue jerseys and, in either event, recover damages, if any, for breach of contract. (b) The coach can either accept or reject the blue jerseys, but if he rejects them, he will thereby waive any remedy for breach of contract. (c) The supplier's shipment of nonconforming goods constituted an acceptance of the coach's offer, thereby creating a contract for the sale of the blue jerseys. (d) The supplier's shipment of the blue jerseys constituted a counteroffer.

Answer choice A is correct. A seller's shipment of nonconforming goods with a notice of accommodation does not constitute an acceptance and breach, but rather a counteroffer, which the buyer is free to either accept or reject. However, an offer calling for prompt shipment can be accepted either by a prompt promise to ship or by the prompt shipment of goods. Under the mailbox rule, an acceptance is effective when mailed. Consequently, on May 15 when the supplier mailed his acceptance to the coach, a contract for 20 red jerseys was formed, even though the coach did not receive this acceptance until May 19. With regard to the May 16 shipment of blue jerseys by the supplier that the coach received on May 18, the coach may accept or reject these jerseys as nonconforming goods and, in either event, recover damages, if any. Answer choice B is incorrect. Although a seller's shipment of nonconforming goods with a notice of accommodation does not constitute an acceptance and breach, but rather a counteroffer, which the buyer is free to either accept or reject, this is only true if the seller has not already accepted the buyer's offer. Here, the supplier had earlier mailed its acceptance of the coach's offer to supply red jerseys, so the supplier's shipment of the blue jersey constitutes a breach of contract. Consequently, the coach may reject the blue jerseys without waiving any remedy for breach of contract. Answer choice C is incorrect because the supplier's shipment of the blue jerseys did not constitute an acceptance of the coach's offer, but a breach of the contract that had already been formed for the shipment of red jerseys. Answer choice D is incorrect because, although typically an accommodation shipment constitutes a counteroffer rather than an acceptance and breach of the contract, here the shipment, despite being designated as an accommodation, cannot be a counteroffer because the supplier had already accepted the coach's offer.

At the beginning of the week, a homeowner met with a contractor regarding remodeling a bathroom in her home. At the conclusion of their meeting, the contractor told the homeowner that he would charge her $5,000-$6,000 for the work, but that he would get back to her with a final price. When he arrived at his office later that day, the contractor opened an email from the homeowner that she had sent earlier. In the email, she stated that she would pay the contractor $5,000 for the job. The next day, the contractor responded by email that he could not complete the work for less than $5,500. The homeowner replied by email that she couldn't pay $5,500, but that, if the contractor changed his mind, he could begin work before the end of the week. The contractor received the email, but did not respond. The contractor appeared the next day at the homeowner's house and began remodeling the bathroom. Which of the following statements regarding the relationship between the parties is most accurate? (a) A contract was formed at the price of $5,000. (b) A contract was formed at the price of $5,500. (c) A contract was formed at a reasonable price. (d) No contract was formed.

Answer choice A is correct. An offer is terminated by rejection. A counteroffer acts as a rejection of the original offer and creates a new offer. A terminated offer may be revived by the offeror, however. As with any open offer, the revived offer can be accepted by the offeree. In this case, the contractor rejected the homeowner's original offer of $5,000 and made a counteroffer of $5,500. The homeowner rejected the counteroffer and revived her offer of $5,000, and invited the contractor to accept the offer by beginning performance by saying, "If you change your mind [i.e., if you want to accept my revived $5,000 offer], you can begin performance." The contractor accepted the revived offer by appearing at the homeowner's house and beginning to remodel the bathroom. Answer choice B is incorrect because, although the contractor made a counteroffer of $5,500, the homeowner did not accept it. Answer choice C is incorrect because, as noted with respect to answer choice A, the parties entered into a contract at a price of $5,000. Also, as a construction contract, the contract is controlled by the common law, which, unlike the UCC which governs the sale of goods, generally requires that the parties agree on a price. Answer choice D is incorrect because the homeowner revived her original offer, and the contractor accepted the offer by appearing at the homeowner's house to remodel the bathroom.

In January, a local farmer contracted with a chef to sell the chef a specified amount of local organic tomatoes to be delivered on August 1. On June 15, the farmer called the chef to tell him that part of his crop was infested with tomato fruitworms, and he was unsure that he would be able to deliver the full amount requested by August 1. The chef told the farmer that it was absolutely essential that he receive those tomatoes on time to make organic tomato sauce for a restaurant scheduled to open in August. The farmer assured him that he would do his very best to save the crop and deliver by August 1. Does the chef have valid legal grounds to cancel the contract and order tomatoes from another source? (a) No, because the farmer did not state unequivocally that he could not deliver the tomatoes on time. (b) No, because the farmer still had more than 30 days in which to deliver the tomatoes. (c) Yes, because the farmer committed an anticipatory repudiation of the contract by causing the chef to feel insecure about the performance. (d) Yes, because the farmer failed to provide adequate assurances to the chef.

Answer choice A is correct. Anticipatory repudiation occurs when there has been an unequivocal refusal of the buyer or seller to perform, or when reasonable grounds for insecurity arise with respect to the performance of either party, and the other party fails to provide adequate assurances within a reasonable time (not to exceed 30 days under the UCC). Mere expressions of doubt as to a party's ability to perform do not constitute an anticipatory repudiation. Therefore, the farmer has not repudiated, and the chef is still bound to the contract. Answer choice B is incorrect because the 30-day time period is only relevant if the chef had asked the farmer for assurances, which he did not. Answer choice C is incorrect because, while expressions of doubt by a party as to its ability to perform may give the other party the right to demand adequate assurances, they do not amount to the affirmative manifestation of intent required to constitute an anticipatory repudiation. Answer choice D is incorrect because, although anticipatory repudiation occurs under the UCC when a party creating reasonable grounds for insecurity fails to provide adequate assurances within 30 days of demand for assurances, this requirement for assurances only applies where demand has been made. Here, the chef reiterated that he needed the tomatoes by the agreed-upon date, but he never demanded assurances from the farmer.

A builder ordered 100 squares of shingles from a home-supply store for installation on the roofs of homes that he was building. The builder agreed to a price of $120 per square. Delivery was set for no later than noon on the following Monday to the construction site. The truck from the store with the ordered shingles arrived at 1:00 p.m. the following Monday. The builder rejected the shipment due to its failure to arrive on time. The store, which regularly sold 600 squares of shingles per week, resold the squares that had been rejected by the builder, at a price of $110 per square. The store would have made a profit of $3,000 had the builder accepted the shingles. If the store sues the builder for breach of contract, how much can the store recover from the builder? (a) Nothing (b) $1,000, the contract price minus the resale price (c) $3,000, the store's lost profit on the initial sale (d) $4,000, to recover the store's total expectation damages

Answer choice A is correct. Because the builder did not breach the contract, the builder is not liable to the store for damages. The sale of the shingles is a sale of goods and therefore is governed by the Uniform Commercial Code (UCC), even though the builder planned to incorporate the shingles into homes being constructed. Under the UCC, a buyer may reject the goods if the seller fails to make a perfect tender, which includes a timely delivery of the goods. Unlike under the common law, substantial performance of a contractual obligation is not sufficient. Answer choice B is incorrect. Although this is one measure of expectation damages of a seller of goods, the store is not entitled to damages because the builder as buyer was entitled to a perfect tender of the goods, which the store failed to make by its late delivery of the shingles. Answer choice C is incorrect. Lost profits are a measure of a seller's expectation damages if the seller is a lost-volume seller, as the store in this case is with regard to the shingles. This remedy would be available to the store had the builder breached the contract, but the builder did not. Answer choice D is incorrect because, even if the store had been entitled to expectation damages, it could not recoup both measures of its expectation damages.

A manufacturer of t-shirts contracted with a new clothing store to sell the store 1,000 t-shirts per month for a period of two years. The clothing store's signature color for their clothing was an orange-tinted red color, called coquelicot, which is very difficult to replicate on a consistent basis. The contract specified that any t-shirts that were not coquelicot could be returned, but it was silent with regard to the return of any t-shirts for other reasons. One year into the contract, the store decided to switch to coquelicot-colored baseball hats instead of t-shirts. The store returned the most recent shipment of t-shirts to the manufacturer and demanded a refund. The manufacturer refused to grant the refund, and the store sued the manufacturer for damages. At trial, the manufacturer introduced the contract, which clearly stated that t-shirts that were not coquelicot could be returned. The store then attempted to introduce evidence that it had returned t-shirts for other reasons to the manufacturer in the past and received a refund. Is this evidence admissible? (a)Yes, because the evidence is relevant to show that the manufacturer had accepted the return of coquelicot-colored t-shirts in the past. (b) Yes, because the evidence can reasonably establish the parties' course of dealing on this issue. (c) No, because the evidence regarding the return of the t-shirts violates the parol evidence rule. (d) No, because the express term in the contract regarding the return of t-shirts takes precedence over the course-of-performance.

Answer choice A is correct. Course-of-performance evidence is admissible under the UCC to explain or supplement a contract. A course of performance is a sequence of conduct that is relevant to understanding an agreement between the parties if: (i) the agreement involves repeated occasions for performance by a party, and (ii) the other party accepts performance without objection and with knowledge of the course of performance. Here, there were monthly purchases of t-shirts and evidence that the manufacturer had accepted returns of coquelicot-colored t-shirts in the past. Therefore, this course-of-performance evidence is admissible. Answer choice B is incorrect because course of dealing refers to conduct between the parties based on another contractual relationship. Here, because there is only the one contract between the two parties and the evidence relates to the performance of that contract, rather than another contract, the evidence does not relate to their course of dealing, but rather their course of performance. Answer choice C is incorrect because the parol evidence rule does not prevent the introduction of evidence regarding conduct by the parties after the written contract was executed. Answer choice D is incorrect because the agreement was silent as to the return of coquelicot-colored t-shirts. Thus, course-of-performance evidence is admissible to explain how the parties viewed the right of the store to return such t-shirts.

The owner of a coffee shop saw the work of an eccentric local artist at an art show. The owner discovered that the artist operated a small interior decorating business and, wanting the artist's unique style reflected in her own business, hired the artist to decorate her coffee shop. A week before the artist was scheduled to decorate the coffee shop, the artist sold her decorating business to a young art school graduate and delegated all of her outstanding contracts to him. The graduate took over all financial and creative management of the business. If the coffee shop owner refuses to accept performance by the art school graduate, is the owner liable for breach of contract? (a) No, because the artist's duty under the contract involved her taste and skill. (b) No, because the delegation created reasonable grounds for insecurity. (c) Yes, because the art school graduate is completely capable of performing the contract. (d) Yes, because the contract did not prohibit delegation of duty.

Answer choice A is correct. Delegation is not permitted when the other party to the contract has a substantial interest in having the delegating individual perform (for example, in a personal services contract involving taste or a special skill). Answer choice B is incorrect. Any delegation of performance under a contract for the sale of goods may be treated by the other party as creating reasonable grounds for insecurity. When this occurs, the other party may demand assurances from the delegate without prejudice to his rights against the delegator. Here, the predominant purpose of the contract was to provide interior decorating services based on the artist's unique style. As a result, this contract would not be treated as a sale of goods contract and therefore the UCC rules concerning reasonable grounds for insecurity and the right to seek assurances would not apply. Answer choice C is incorrect. When delegation is permitted, the other party must accept the conforming performance of a delegate or be in breach of the contract. However, even if the art school graduate is capable of decorating the coffee shop, the owner sought out the original contract with the artist because she was impressed by the artist's unique style. Therefore, this contractual duty cannot be delegated, and the owner need not accept performance by the art school graduate. Answer choice D is incorrect because a contract does not need to prohibit delegation explicitly when a party to the contract has a substantial interest in having the delegating individual perform. In such cases, delegation is prohibited even if the contract is silent.

A recent college graduate offered to buy all of the computers from a struggling online retailer for which he had been an intern during college. The terms of the written agreement were such that the graduate would pay $10,000 for a "reasonable number of computers, as the closing retailer no longer needed them." Due to the graduate's internship with the retailer, he knew that there were 50 computers in the office and knew that nearly all of them were unused, so he believed he would receive all 50 computers once the retailer closed. He gave the retailer a check for $10,000 and in return, took ten computers from the office that day. With the help of the $10,000 and a sudden upswing in sales in the online retail market, the retailer became profitable. When the graduate demanded the remaining 40 computers, the retailer refused. Instead, the retailer returned the $10,000 to the graduate and demanded the return of the ten computers in the graduate's possession. The graduate sued the retailer for breach of contract. The retailer moved to dismiss and argued that no valid contract existed. How should the court rule? (a) Grant the motion, because there was no agreement as to quantity. (b) Grant the motion, because the retailer's increased profitability constituted a supervening event. (c) Deny the motion, because the court may supply missing terms in a contract. (d) Deny the motion, because the parties both had the objective intent to form a contract.

Answer choice A is correct. For a contract to exist, the terms of the contract must be certain and definite, or the contract fails for indefiniteness. Under common law, all essential terms - the parties, subject matter, price, and quantity - must be specified in the agreement or the contract fails. Under the UCC, a court may supply missing terms, but a contract must still specify the parties, subject matter, and quantity. Here, the contract is governed by the UCC, and because it is missing the key term of quantity, it is therefore invalid. Answer choice B is incorrect because the increased profitability would not discharge the retailer's duty to perform if performance had been required. Answer choice C is incorrect because under the UCC, the court may not supply a missing quantity term. Answer choice D is incorrect because although the parties intended to form a contract, they failed to adequately define the terms of the contract.

A father hoping to build a new playground for his children had a friend whose hobby was woodworking. One day over lunch, the two men discussed an arrangement in which the woodworker would build and deliver a swing set to the father for $2,000 within two weeks. After lunch, the woodworker sent an email to the father restating what had been discussed. The father immediately responded in a signed email stating "We have a deal. But please deliver the set within one week instead." The woodworker did not respond but began working on the swing set that day. Eight days later, the father called the woodworker to ask why the swing set had not been delivered. The woodworker stated that he intended to deliver the swing set within the two-week period originally discussed. He began to work more quickly to complete the swing set sooner and delivered the swing set two days early, but the father refused to pay him for it. Under the UCC, is the woodworker entitled to recover the $2,000? (a) Yes, because he delivered the swing set within two weeks. (b) Yes, because the goods were specially manufactured. (c) No, because the parties did not agree to all essential terms. (d) No, because the woodworker did not deliver the swing set within one week.

Answer choice A is correct. For a sale of goods where at least one party is a nonmerchant, an acceptance that contains a modification to the terms in the offer is nevertheless an acceptance. There is an exception if the acceptance is expressly conditioned on assent to the modified terms, in which case the acceptance would be a counteroffer, but that does not seem to have occurred here. In this case, the father did not say that he accepted IF the woodworker delivered the set in a week, but rather accepted and added a different term. Here, there is a sale of a goods contract between nonmerchants (or at most, between a merchant and a nonmerchant), so the father's acceptance was valid against the original offer. The new, week-long time period would be treated as a proposal for an additional or different term that must be separately accepted to become a part of the contract. Here, it was not accepted, so the terms are in accordance with the original contract. Answer choice B is incorrect because the exception for specially manufactured goods is applicable when there is a dispute about whether a contract is enforceable even though it is not in writing. In this case, not only was the contract in writing, but the terms of the contract are reflected in the woodworker's original offer. Answer choice C is incorrect because the parties did agree to all terms. The father's attempted modification did not create a lack of understanding. Answer choice D is incorrect because the delivery needed to be made within two weeks, as the one-week modification was not part of the contract.

While attending a rodeo on August 20, a hat maker entered into a valid, written agreement with the rodeo manager to make 500 leather cowboy hats for an upcoming rodeo event at a price of $75 per hat. Per the agreement, the rodeo manager agreed to pay one-fourth of the total purchase price to a tannery owner to whom the hat maker owed a debt for a previous leather order. On August 25, the hat maker changed his mind about paying one-fourth of the purchase price to the tannery owner. The hat maker and rodeo manager subsequently executed a valid modification of the original agreement. The rodeo manager's brother was also present on August 20 when the original agreement was executed, but he did not know about the August 25 modification of the agreement to no longer pay the tannery owner. On August 30, the brother, who was friends with the tannery owner, called and told him that his debt from the hat maker would finally be paid off. However, the rodeo manager refused to pay one-fourth of the purchase price to the tannery owner. If the tannery owner sues the rodeo manager for one-fourth of the purchase price, will he recover? (a) No, because the tannery owner did not rely on the August 20 agreement between the hat maker and the rodeo manager. (b) No, because there was no consideration for the promise to pay the tannery owner by the hat maker and the rodeo manager. (c) Yes, because the tannery owner had the right to sue the rodeo manager to enforce the contract between the rodeo manager and the hat maker. (d) Yes, because the rodeo manager agreed to pay one-fourth of the purchase price to the tannery owner on August 20.

Answer choice A is correct. If performance of a promise would satisfy an actual, supposed, or asserted duty of the promisee to a third party, and the promisee did not intend to make a gift to the third party, then the third party is an intended beneficiary who is a creditor beneficiary. A creditor beneficiary has the right to sue either the promisor or the promisee to enforce the contract. Here, the tannery owner was a creditor beneficiary as of August 20. However, the hat maker and the rodeo manager agreed on August 25 to not pay the tannery owner. The tannery owner did not know about the August 20 agreement until August 30, after the two parties had agreed to not pay the tannery owner. Thus, he did not rely upon the original agreement that was later modified by both parties. For this reason, answer choices C and D are incorrect. Answer choice B is incorrect because an agreement to benefit a third party does not need to be supported by separate consideration. In addition, the tannery owner cannot recover because the two parties to the original agreement eliminated the third-party beneficiary agreement on August 25. Because the original agreement involved the sale of goods, no consideration is required to amend it, as long as the amendment is done in good faith.

A gardener and a carpenter contracted in writing for the carpenter to repair the gardener's four identical beehives for $500 each. The contract was signed by both parties and provided that the gardener would pay the carpenter $2,000 upon delivery of the fourth repaired beehive. The gardener immediately delivered all four beehives to the carpenter for repair. The carpenter repaired and delivered the first two beehives without any problems and without demanding payment. However, upon delivery of the third repaired beehive, the carpenter demanded a payment of $1,500. Is the gardener required to make the demanded payment at this time? (a) No, because she has no duty to pay the carpenter anything until the last beehive is repaired and delivered. (b) No, because the course of performance between the parties has established that payment is not due upon the delivery of each repaired beehive. (c) Yes, because the carpenter is entitled to the fair value conferred on the gardener. (d) Yes, because the contract is divisible with respect to the repair and delivery of each beehive.

Answer choice A is correct. The express and unambiguous terms of the contract establish that the gardener is not required to tender payment for the beehives until the last one is repaired and delivered. Therefore, the gardener is not required to pay the carpenter anything at this time. Answer choice B is incorrect. A course of performance is relevant to show a waiver or modification of any term inconsistent with the course of performance. Because the course of performance here has been consistent with the terms of the contract, the course of performance is irrelevant and the express terms of the contract regarding the gardener's duty to pay will prevail. Answer choice C is incorrect because, although a party who commits a material breach of his contractual obligations would ordinarily be entitled to the fair value of any benefit conferred on the nonbreaching party, the facts do not establish that the carpenter has breached or clearly repudiated this contract. Therefore, the carpenter is not entitled to payment at this time. Answer choice D is incorrect. A divisible or installment contract is one in which the various units of performance are divisible into distinct parts. Recovery under divisible contracts is limited to the amount promised for the segment of the contract performed. However, this does not allow the performing party to demand payment for the units of performance already performed when there has been no breach and the contract explicitly provides for payment upon completion of the entire contract. Thus, the divisible nature of this contract will not entitle the carpenter to payment at this time.

On March 1, the owner of a ferry boat that operated only during daylight hours during the summer months of June, July, and August entered into a written agreement with a man to serve as the captain of the boat for the upcoming season. On May 1, the owner contracted with a woman to serve as the captain of the boat. On May 30, the man was diagnosed with an illness, and the treatment for this illness prevented him from being employed until the following year. On May 31, the owner, learning of the man's illness, told him not to worry about their contract, as he had found someone else to serve as captain of the ferry boat. The woman served as captain of the ferry boat for the summer months of June, July, and August that year. On September 1, the man sued the owner for damages based on a breach of their contract. Will his suit succeed? (a) No, because the man was unable to serve as the captain of the boat during the summer months. (b) No, because the owner informed the man about the owner's contract with the woman prior to June 1. (c) Yes, because the owner's contract with the woman constituted an anticipatory breach of the owner's contract with the man. (d) Yes, because the owner did not inform the man of the owner's contract with the woman until after the man learned he had an illness.

Answer choice A is correct. The owner's contract with the woman to serve as the captain of the ferry boat constituted an anticipatory breach of the owner's contract with the man, because the owner would not have been able to employ both the man and the woman as the captain of the ferry boat. Despite this breach, however, the owner's duty to pay damages to the man was discharged by the man's inability to have served as the captain of the ferry boat. Answer choice B is incorrect because, although the owner did tell the man about the owner's contract with the woman prior to the date on which the man was to begin serving as captain, this information constituted an anticipatory breach of the owner's contract with the man rather than a reason for excusing the owner from honoring that contract. Instead, it is the man's inability to serve as captain of the ferry boat during the summer months that discharges the owner's duty to pay damages. Answer choice C is incorrect because, although the owner's contract with the woman to serve as captain of the ferry boat did constitute an anticipatory breach of the owner's contract with the man, the owner's duty to pay damages to the man as a consequence of this breach was discharged by the inability of the man to have served as captain of the ferry boat during the summer months. Answer choice D is incorrect because, while the owner's delay in telling the man about the other contract was almost certainly not in good faith, there is no indication that this delay caused the man to forego other employment. Moreover, even had the man been able to secure alternative employment for the summer months, the man's illness would have prevented him from being employed during that time.

A comic-book collector entered into a written contract with a man who had just inherited a large collection of comic books. They agreed that the man would sell to the collector numerous rare comic books for $4,000. Before they signed the agreement, the collector e-mailed the man to ask if he would include a particularly rare issue of Batman in the sale. The man agreed by e-mail that he would. The contract contained a list of each comic book that was supposed to be part of the sale, but the list did not include the Batman issue. The contract also stated that it was "the complete and final agreement" between the man and the collector. When the collector received the comic books, he discovered that the shipment did not include the Batman comic book. He sued the man, stating that even though the written contract did not mention the particular Batman issue, the collector had relied on the promise of the issue when he signed the contract. At trial, the collector seeks to introduce the e-mail he sent to the man prior to the execution of the agreement that referenced the Batman comic book. Is the e-mail admissible? (a) No, because of the parol-evidence rule. (b) No, because the Uniform Commercial Code does not apply to this transaction. (c) Yes, because the agreement was only partially integrated. (d) Yes, because the parol-evidence rule applies only to oral communications.

Answer choice A is correct. The parol-evidence rule generally prevents a party to a written contract from presenting prior extrinsic evidence that contradicts the terms of the contract as written. The rule is concerned with whether any of the earlier oral or written terms are part of the parties' contract, even though they are absent from the parties' written agreement. Generally, the parol-evidence rule prevents a party to a written contract from presenting other evidence that contradicts the terms of the contract as written. If a document is determined not to be "integrated," then the parol-evidence rule may not apply. When documents are only partially integrated, the parties are permitted to present extrinsic evidence as long as the evidence is consistent with the writing. The UCC (applicable to this contract) in essence presumes that a contract is a partial integration. However, if the writing completely expresses all of the terms of the parties' agreement, then it is a total integration, and the parties cannot introduce any extrinsic evidence (oral or written) of prior or contemporaneous understandings or negotiations. Here, the contract stated that it was the complete and final agreement and included a list of each comic book included in the sale, but it did not list the Batman issue in the list. Thus, the writing was a total integration, and the e-mail is inadmissible as extrinsic evidence of the agreement to include the Batman issue. Answer choice B is incorrect because the Uniform Commercial Code (UCC) does apply to this contract because it involves the sale of goods. Regardless, even if the UCC did not apply, the parol-evidence rule would still be applicable to the contract. Answer choice C is incorrect because the writing was a total integration, but even in a partially integrated agreement, the parol evidence may not contradict the writing. Answer choice D is incorrect because the parol-evidence rule applies to any previous or contemporaneous written or oral evidence prior to integration, not just oral communications.

A math major at a local college agreed to tutor a high school student who was having trouble in his math class. The tutor promised to meet with the student for five hours a week for the remaining 2 months of the term, and the student's mother agreed to pay the tutor $20 per hour. In addition, the parties agreed that, if the student received a B or better in his math class, the mother would pay the tutor a $500 bonus. The parties did not reduce their agreement to writing. The mother paid the tutor weekly for the tutoring. At the end of the term, the student received a B+ in his math class. The mother, not having the funds to pay the tutor's bonus, contacted the tutor and offered her a bike worth $300 in lieu of the bonus payment. The tutor accepted the bike. Is the tutor entitled to recover the remaining $200? (a) No, because the tutor accepted the bike instead of the bonus. (b) No, because the tutor agreed to accept a lesser amount. (c) Yes, because the modification was not in writing. (d) Yes, because there was no dispute as to the amount owed.

Answer choice A is correct. Under an accord agreement, a party to a contract agrees to accept a performance from the other party that differs from the performance that was promised in the existing contract, in satisfaction of the other party's existing duty. Although the original debt is not generally discharged immediately upon entering into an accord, once the creditor accepts the lesser amount offered by the accord, the original contract is discharged. Note, however, that agreeing to accept a different type of performance does not discharge the original contract. Here, by agreeing to accept the bike as substitute performance (and by actually accepting the bike), she forfeited her right to recover the remaining $200. Answer choice B is incorrect because simply agreeing to an accord does not automatically discharge the original obligation. Until satisfaction occurs, the creditor can still recover under the original contract, and the original debt is not discharged unless there is a dispute as to the validity of the debt or when the payment is of a different type than called for under the original agreement. Here, even though the original agreement was discharged (both because satisfaction occurred and because payment was of a different form than originally contemplated), the tutor's agreement to a lesser amount is not what discharged that original agreement. Answer choice C is incorrect because there is no requirement that contract modifications or accords be in writing, unless they fall within the Statute of Frauds. Answer choice D is incorrect. Whether there is a dispute as to the amount owed can be relevant to whether an original debt is discharged. Here, the fact that there was no dispute as to the amount owed would allow the tutor to recover the difference if the mother had tried to make a partial payment of $300 instead of offering payment of a different kind. However, the tutor is not entitled to recover the remaining $200 because she already accepted the bike in lieu of the $500 bonus payment.

A wheat farmer contacted an agricultural services company in May to inquire about hiring workers for a five-day period toward the beginning of the summer-long harvest season to assist the farmer in harvesting his wheat crop. After some negotiations, the farmer entered into a written contract with the company to provide five workers for a five-day period starting in the first week of June for a cost of $5,000. On June 5, the company's workers went on strike. On June 9, the strike ended and the company's workers began harvesting wheat on the farmer's farm, and did so for five days. The farmer subsequently refused to pay the company, claiming the company's delay in performance excused his obligation to pay. Is the farmer's obligation to pay excused? (a) No, because the contract did not contain a "time is of the essence" clause. (b) Yes, because substantially performing does not excuse a breach for commercial contracts. (c) Yes, because starting in the first week of June was an express condition of the contract. (d) Yes, because the delay was a material breach as the harvesting season had already begun.

Answer choice A is correct. Under common law, a material breach of contract allows the nonbreaching party to withhold any promised performance and to pursue remedies for the breach, including damages. A breach is considered minor when the breaching party has substantially performed. Here, both parties agreed time was not of the essence (because the completion date was not an express condition, as explained more fully below) and the delay does not appear to have been consequential (a somewhat nebulous start date combined with a summer-long harvesting season further supports this conclusion). For this reason, answer choice D is incorrect. Answer choice B is incorrect because it misstates the law, as there is no such rule that substantial performance does not apply to commercial contracts. Substantial performance does not apply to the sale of goods under the UCC, but does apply to contracts for services, such as the contract here. Answer choice C is incorrect because the "first week of June" start date was not an express condition. Express conditions generally contain language such as, "on condition that" or "provided that."

A caterer contracted with a local farmer for the delivery of three dozen fresh local eggs. The contract provided that because the caterer used the sterilized egg shells to serve one of her signature dessert recipes, the eggs needed to be a uniform color. The farmer delivered the caterer 20 white eggs and 16 speckled eggs. The caterer immediately e-mailed the farmer and informed him that she was rejecting the eggs because she could not use the inconsistent shells to serve her desserts. She also told him she could either sell them for him or save them for him to retrieve, and that she would wait for his instructions. The caterer saved the eggs for a week while she waited for the farmer's instructions, but he never responded. Finally, concerned that the eggs would spoil soon, she gave them to a friend. If the farmer brings a contract claim against the caterer for the contract price of the eggs she gave to her friend, will he succeed? (a) No, because the caterer did not accept the eggs. (b) No, because the caterer had no obligations regarding the nonconforming eggs. (c) Yes, because the caterer had a duty to retain the eggs until the farmer retrieved them. (d) Yes, because the caterer was required to sell the eggs on the farmer's behalf.

Answer choice A is correct. Under the UCC, if either the tender or the goods are nonconforming, then the buyer has the right to accept or reject all or part of the goods. Upon tender of nonconforming goods, a buyer must retain possession of the rejected goods for a reasonable period of time to allow the seller to reclaim them. However, a buyer's attempts in good faith to dispose of defective goods when the seller fails to give instructions within a reasonable time are not to be regarded as an acceptance. Here, the caterer properly rejected the eggs and tried to keep them until the farmer told her what to do with them, but she never received a response. Therefore, she did not accept the eggs, and the farmer cannot recover the contract price in a contract action. Answer choice B is incorrect because the caterer was obligated to notify the farmer that she was rejecting the eggs and to retain the eggs for a reasonable period of time for the farmer to reclaim them, which she properly did. Answer choice C is incorrect because the caterer was only required to retain the eggs for a reasonable amount of time. Based on the fact that she was concerned the eggs would spoil, she had already waited a reasonable amount of time with no response from the farmer when she gave the eggs to her friend. Answer choice D is incorrect. If rejected goods are perishable and the seller has no local agent to whom they can be returned, in the absence of other instructions from the seller, a merchant buyer is required to sell the goods on the seller's behalf. However, a non-merchant buyer is not required to sell the goods on the seller's behalf. Because the caterer is not in the business of selling eggs, she had no duty to sell these perishable goods on the farmer's behalf.

A general contractor learned that a company was accepting bids for a lucrative construction project involving a high-rise building. The general contractor contacted a number of subcontractors and informed them that he would be accepting bids for the electrical work on the project for the next week. After receiving a number of bids from subcontractors, the general contractor selected a bid from a young subcontractor, which was the lowest bid (although within a reasonable range of the other bids), and used that bid in calculating his overall bid on the construction project. Soon after submitting his bid to the general contractor but after the general contractor had submitted his overall bid, the subcontractor realized that he could have charged more for his services based on their market value. The company ended up choosing the general contractor's bid for the project. When the general contractor called the subcontractor to tell him that his bid was accepted and inform him of the starting date of the project, the subcontractor told the general contractor that he revoked his bid. As a result, the general contractor had to use a different subcontractor to perform the work at a cost $3,000 higher than the first subcontractor's bid. In a suit to recover the $3,000 from the subcontractor, is the general contractor likely to prevail? (a) Yes, because the general contractor detrimentally relied on the subcontractor's bid. (b) Yes, because an enforceable contract was formed when the general contractor used the subcontractor's bid in his overall bid on the project. (c) No, because the subcontractor's bid was lower than the market value of similar services. (d) No, because the subcontractor's bid was an offer that could be freely revoked

Answer choice A is correct. Under the doctrine of promissory estoppel, a promise that the promisor should reasonably expect to induce action or forbearance of a definite and substantial character on the part of the promisee, and that does induce such action or forbearance, is binding if injustice can be avoided only by enforcement of the promise. In the construction industry, it would be unjust to permit a subcontractor to revoke a bid after inducing justifiable and detrimental reliance in the general contractor. Accordingly, a subcontractor's bid can be enforced by the contractor under the theory of promissory estoppel. In this case, the general contractor will prevail because he detrimentally relied on the subcontractor's bid and ended up having to spend $3,000 more due to the subcontractor's unjust revocation of the bid. Answer choice B is incorrect because an enforceable contract was not formed when the general contractor used the subcontractor's bid. Because the sub-bid is only an outstanding offer, the general contractor is not bound to accept it upon becoming the successful bidder for the general contract. The use of the subcontractor's bid did not constitute an acceptance of the offer. Rather, the subcontractor's bid is enforceable here under the doctrine of promissory estoppel. Answer choice C is incorrect because the mere fact that the bid was lower than market value does not cause the general contractor's reliance on that bid to be unjustifiable or otherwise unforeseeable. Answer choice D is incorrect. The subcontractor's bid was an offer, but it cannot be revoked after the general contractor detrimentally relied on the bid.

A restaurant placed an order for 100 pounds of USDA prime beef from a meat packer. The order was to be shipped to the restaurant immediately. The meat packer erroneously shipped 100 pounds of USDA choice, rather than prime beef. When the choice beef arrived at the restaurant, which of the following is an accurate description of the restaurant's legal obligations? (a) When the meat packer shipped the beef, a contract was formed, but the restaurant is not required to accept the choice beef. (b) The shipment constitutes a counteroffer, which the restaurant can accept or reject. (c) Since the shipment does not mirror the terms of the offer, there is no contract. (d) The choice beef cannot be rejected because a contract was formed when the meat packer shipped the beef.

Answer choice A is correct. Unless an offer specifies otherwise, the offer may be accepted by either a promise or by performance. If the buyer requests that the goods be shipped, then the buyer's request will be construed as inviting acceptance by the seller either by a promise to ship or by prompt shipment of conforming or nonconforming goods. Here, the meat packer, by shipping the beef, accepted the offer. However, because the beef shipped did not conform to the terms of the offer, the shipment of choice rather than prime beef constitutes a breach of the contract. Consequently, the restaurant may accept the shipment of choice beef or reject it. Answer choice B is incorrect because the UCC generally treats the shipment of nonconforming goods as both an acceptance of the offer and a breach of the contract. Answer choice C is incorrect because it states the mirror-image rule for an acceptance under the common law, but the UCC controls in this case. Answer choice D is incorrect. Although a contract has been formed, the restaurant is still entitled to reject the beef as nonconforming.

At the auction of construction equipment owned by a contractor, several lots were offered for bidding and the highest bids for each were accepted by the auctioneer. The auctioneer then announced that a lot that consisted of a backhoe was being auctioned off. Several bids for the backhoe were acknowledged by the auctioneer. Just before the auctioneer brought down her gavel, she glanced at the contractor. The contractor gave the auctioneer a prearranged signal. Acting in accord with the signal, the auctioneer stated that the backhoe was being removed from the auction. There had been no indication as to whether the auction was being held with or without reserve. The highest bidder on the backhoe, contending that he is now its owner, has brought suit against the contractor. How should the court rule? (a) For the contractor, because the auctioneer had not brought down the gavel, announcing the completion of the sale of the backhoe. (b) For the contractor, because the backhoe constituted equipment. (c) For the highest bidder, because unless the seller reserves the right to withdraw an item from an auction, the seller may not do so once the auction has begun. (d) For the highest bidder, because the seller forfeited his right to withdraw the backhoe by prearranging a signal with the auctioneer.

Answer choice A is correct. Unless specifically announced otherwise, an auction is with reserve, meaning that the seller has the right to withdraw an item from sale at any time before the auctioneer announces the completion of the sale. Here, because the backhoe was withdrawn from the sale before the auctioneer announced the completion of the sale by bringing down the gavel, the contractor remains the owner of the backhoe. Answer choice B is incorrect because the classification of the goods being auctioned is irrelevant to determining whether the goods can be withdrawn from the auction. Answer choice C is incorrect. As noted with respect to answer choice A, the default rule is that an auction is with reserve unless there is a specific announcement to the contrary. In addition, if goods in an auction are offered in lots, each lot represents a separate sale. Consequently, even though the auction had begun, the contractor retained the right to withdraw the backhoe, which was being auctioned as a separate lot, from sale until the auctioneer announced the completion of its sale. Answer choice D is incorrect, as there is no prohibition on this type of behavior.

As part of a divorce settlement, an ex-husband purchased from an insurance company an annuity to be paid at a fixed amount quarterly over the life of his ex-wife. (An annuity contract requires the payment of a fixed amount periodically for the duration of a person's life.) Within a week after the purchase, the ex-wife learned that she had a fatal illness, which had not previously manifested itself, but had existed for some time. She died two months later, prior to receiving any payments from the annuity. The ex-husband filed suit to rescind the annuity contract. Will he be successful? (a) No, because the annuity contract was a third-party beneficiary contract. (b) No, because the ex-husband assumed the risk of his ex-wife's death. (c) Yes, because the ex-wife's death frustrated the purpose of the annuity. (d) Yes, because the ex-husband and the insurance company made a mutual mistake as to ex-wife's health.

Answer choice B is correct. A mistake is a belief that is not in accord with the facts as to a basic assumption on which the contract was made that materially affects performance. Here, the ex-husband and ex-wife both entered into this agreement without knowledge of the wife's illness, both presumably believing that the ex-wife would live for some number of years. However, not every contract based on a mistaken belief will be rescinded because of that mistake. If the adversely affected party assumed the risk of mistake, he will be unable to rescind. The purchaser of an annuity contract assumes the risk that the person on whose life the annuity is based will die before the price paid for the annuity is recouped, and the issuer of the contract assumes the risk that that person will live beyond the recoupment date. This assumption of the risk is inherent in the nature of the annuity contract. Answer choice A is incorrect because the husband, as one of the parties to the annuity contract, did not surrender his contract rights merely because the contract was entered into by the ex-husband for the benefit of his ex-wife. Answer choice C is incorrect because, although the ex-wife's death did prevent her from receiving any benefit from the annuity, her death, while unanticipated, could not be characterized as unexpected. Instead, the occurrence of the ex-wife's death was a basic assumption on which the contract was based. The termination of the insurance company's obligation was specifically linked to her death. Answer choice D is incorrect because, in order for a contract to be avoided due to a mutual mistake, this risk of the mistake cannot have been assumed by the party seeking to avoid the contract. The ex-husband, as purchaser of the annuity, assumed the risk his ex-wife would die before the purchase price of the annuity could be recouped by her.

A woman emailed her friend, stating that she would like to buy the friend's teacup collection when "times aren't so tough. I would pay $1,000 for them." The friend responded with an email agreeing to the deal. The women did not exchange money or the teacups and did not see each other until a year later. When they did see each other, the friend apologized for forgetting about their discussion and told the woman she would deliver the teacups the next weekend and would accept a check at that time. The woman said she did not remember the discussion but would pay $750 for the teacups. The friend responded, "Haven't we already discussed this? Sold." The next day, the friend turned the teacups over to the woman, who provided her with a check for $750. The friend immediately responded that she needed the check for the remaining $250. The woman kept the teacups. Is the woman liable for the remaining $250? (a) No, because a contract had not been formed. (b) No, because a contract was not formed until the day the women spoke in person. (c) Yes, because the original contract was for $1,000. (d) Yes, because she kept the teacups.

Answer choice B is correct. A statement is an offer only if the person to whom it is communicated could reasonably interpret it as an offer. It must express the present intent of a person to be legally bound to a contract. The primary test of whether a communication is an offer is based on the objective theory of contracts; i.e., whether an individual receiving the communication would believe that he could enter into an enforceable deal by manifesting agreement to it. Here, the woman's email at the very most reflected a potential interest in buying the teacups at a future, unknown date. It does not reflect a present intent to be legally bound to buy the teacups. On the day of the oral discussion, however, she expressed a present intent to buy the teacups, and her friend would reasonably believe they could enter into an enforceable deal on that day. Answer choice A is incorrect because on the day of the oral discussion, the women did form an agreement regarding the teacups. Answer choice C is incorrect because as indicated above, the original letter indicated merely a possible interest in buying the teacups at a later date but no present intent to be bound by such an agreement. Answer choice D is incorrect because she accepted the teacups based on the agreement entered into on the day of the oral discussion.

A farmer owned a tractor and offered his brother the chance to purchase it. He stated that his brother had to decide whether he wanted to purchase the tractor within "six months of today's date." The brother paid him $200 on that day to keep the option open. The agreement was reduced to writing, signed by both men, and dated May 15. The farmer died on July 1. On August 15, the brother notified the executor of the farmer's estate that he wanted to accept the offer to buy the tractor. The executor refused to sell, and the brother filed suit for the enforcement of the contract. Is the brother likely to prevail? (a) Yes, because the brother made an enforceable contract to buy the tractor on May 15. (b) Yes, because the brother paid $200 to keep the option open. (c) No, because the offer terminated on July 1. (d) No, because at the time of the farmer's death, the tractor went to his estate.

Answer choice B is correct. Although the UCC generally applies to the sale of goods, the firm offer rule only applies to a promise not to revoke an offer when there is no consideration to support the promise. Here, the brother paid consideration to keep the offer open, so the UCC firm offer rule does not apply. While an offer generally terminates upon the death or mental incapacity of the offeror, an exception exists for an offer that is an option. Valid options do not terminate upon death or mental incapacity of the offeror because consideration was paid to keep the offer open during the option period; the offer is therefore made irrevocable during that period. Here, the offer would not terminate upon the farmer's death because it was an option for which consideration was paid, and it was still open at the time of the farmer's death. Answer choice A is incorrect because on May 15, the parties made an option contract; the actual contract to buy the tractor was made on August 15. Answer choice C is incorrect because the offer did not terminate upon the farmer's death; as explained above, it was an option contract. Answer choice D is incorrect. Although the tractor did go to the farmer's estate, it is still subject to the option contract.

A math tutor entered into an agreement with a father to provide one month of tutoring for the father's son. The agreement stated that the math tutor would provide lessons for the son twice a week during the month for eight lessons at a total cost of $1,000. The cost included the materials valued at $350 that the math tutor intended to purchase from a particular educational services provider. The father knew a salesman for the educational services provider and notified him that his company should expect a sale in the next week. As the educational services provider was a new company without many sales, the salesman was excited at the prospect. A week after the agreement between the math tutor and the father was executed, the math tutor informed the father that he would not tutor the son unless the father provided an additional $250. The father refused. The educational services provider subsequently brought an action against the math tutor for breach of the agreement between the math tutor and the father. Will the educational services provider prevail? (a) No, because it did not detrimentally rely on the agreement. (b)No, because it was only an incidental beneficiary of the agreement. (c) Yes, because the father informed its salesman of the agreement concerning its materials. (d) Yes, because the provider sustained $350 in lost-profit damages as a result of the math tutor's breach.

Answer choice B is correct. An incidental beneficiary is one who benefits from a contract even though there is no contractual intent to benefit that person. An incidental beneficiary has no rights to enforce the contract. In this case, neither the father nor the math tutor entered into the agreement for tutoring with the intention to benefit the educational services provider. Any benefit to the educational services provider was incidental. Accordingly, the educational services provider was an incidental beneficiary of the agreement. As a result, it has no rights to enforce the agreement between the father and the math tutor. Answer choice A is incorrect. Although there are no facts to indicate detrimental reliance by the educational services provider, detrimental reliance is not relevant because there was no promise made to the educational services provider. The father merely told the salesman about the agreement, and there is no evidence that the provider detrimentally relied on the information in any way. Therefore, these facts are insufficient to create grounds for the invocation of the doctrine of promissory estoppel. Answer choice C is incorrect because the knowledge of the salesman, an agent of the educational services provider, as to the agreement does not give the educational services provider the right to enforce the contract because it does not change its status as an incidental beneficiary. Answer choice D is incorrect because the educational services provider is only an incidental beneficiary and cannot enforce the agreement. As a result, it cannot seek damages based on the agreement.

In June, a local chef learned of a new business that opened in the area. Hoping to attract the business as a new client, the chef sent the business an offer consisting of a catalog of menus available through his catering service and a form letter he sent to all new businesses in the area. The letter was signed by the chef and included the following language: "Welcome! I specialize in creating delicious meals with local and organic ingredients, and I would be honored to be your catering source for all your business, promotional, and personal needs! To welcome you to the community, I would like to offer you a 25% discount off of my catalog prices on any three-course meal order, for up to 100 people, submitted this calendar year. I hope to hear from you soon and I look forward to doing business with you!" No communication occurred between the parties until the end of November, when the business faxed an order form to the chef requesting a catered meal for 60 people at a promotional event for a 25% discount. The court has found that both parties are merchants with respect to this transaction. Did the business's faxed order create an enforceable contract at a 25% discount? (a) Yes, because the signed promotional letter created a firm offer. (b)Yes, because the chef had not revoked the offer before the end of the calendar year. (c) No, because the form letter was only an invitation to deal. (d) No, because the company's power of acceptance terminated after a reasonable period of time.

Answer choice B is correct. An offer may be accepted at any time before the offer is terminated. Here, the offer specified that the offer would terminate at the end of the calendar year. Because the business submitted its order before the end of the calendar year, and because the chef has taken no action to terminate the offer, the offer remained open at the time the business submitted its order. Answer choice A is incorrect. Under the UCC, an offer to buy or sell goods is irrevocable if the offeror is a merchant, there is an assurance that the offer is to remain open, and the assurance is contained in a signed writing from the offeror. However, the irrevocability of a firm offer cannot exceed three months unless the offeree gives consideration to validate it beyond the three-month period. Therefore, even if a firm offer was created here under the UCC, it was no longer irrevocable in November; the chef could have revoked the offer, but didn't. Answer choice C is incorrect because even if the chef sent the letter to many businesses, the letter is more than a mere advertisement or invitation to deal. Here, the letter noted the terms of the offer and when the offer would terminate. This is sufficient to create an offer that the business could accept in November. Answer choice D is incorrect because this is only a true statement of law for situations in which the offer does not specify a time limit for acceptance. Because this offer specifically noted that the discount would apply to any order made before the end of the calendar year, the power of acceptance continued until the end of the calendar year.

A homeowner entered into a contract with a landscaper. The contract specified that the homeowner would pay the landscaper $10,000 upon completion of a list of projects. The landscaper performed the work while the homeowner was away on vacation. When the landscaper sought payment, the homeowner refused, noting that a tree had not been trimmed as required by the contract. The landscaper responded that, since he would now have to forego other work in order to trim the tree, he would do it but only if the homeowner agreed to pay him a total of $10,500 for his services. The homeowner, desperate to have the work completed, agreed. Once the work was completed, however, the homeowner gave the landscaper a check for $10,000, and refused to pay more. The landscaper sued for breach of contract. Is the landscaper likely to succeed in his claim? (a) No, because an enforceable contract cannot be renegotiated. (b) No, because there was no consideration for the promise to pay $10,500 and no unanticipated circumstances arose. (c) Yes, because there was a valid modification of the contract. (d) Yes, because the landscaper suffered a detriment by foregoing other work.

Answer choice B is correct. At common law, a promise to perform a preexisting legal duty does not qualify as consideration because the promisee is already bound to perform. In this case, the landscaper had a preexisting legal duty to trim the tree, and thus there was no consideration to support the homeowner's promise to pay an additional $500. Answer choice A is incorrect because an enforceable contract may be renegotiated. Even when there is a preexisting legal duty, there will be consideration if the promisee gives something in addition to what is already owed or varies the preexisting duty. Answer choice C is incorrect because modification of a services contract must be supported by consideration. Or some circumstances that were not anticipated when the contract was made must have arisen, and modification is fair and equitable in light of those circumstances. Answer choice D is incorrect because the fact that the landscaper had to forego other work would not serve as consideration in this case because the landscaper was under a preexisting legal duty.

A chemistry professor offered to sell her colleague an autographed first edition novel for $1,000. The professor provided her colleague with a signed written statement specifying the terms of the offer, and stating that the offer would remain open for one week. Two days later, the colleague learned that the professor had sold the book to someone else in their department. The next day, the colleague showed up at the professor's office with $1,000, asking to purchase the book. The professor apologized, saying that the book had already been sold. Is the colleague likely to succeed in an action for breach of contract? (a) No, because an option contract is not valid unless the offeror is a merchant. (b) No, because the colleague learned that the book had been sold before accepting the offer. (c) Yes, because the professor did not revoke the offer prior to the colleague's acceptance. (d) Yes, because the offer was contained in a signed writing, and thus could not be revoked.

Answer choice B is correct. In general, an offer can be revoked by the offeror at any time prior to acceptance. An offer is revoked when the offeror makes a manifestation of an intention not to enter into the proposed contract. If the offeree acquires reliable information that the offeror has taken definite action inconsistent with the offer, the offer is automatically revoked (i.e., a constructive revocation occurs). In this case, because the book was a unique item, the offer was revoked when the colleague learned that the professor had already sold the book. Answer choice A is incorrect because an option contract, in which an offeree gives consideration to limit an offeror's power to revoke an offer, may be valid even if entered into by parties who are not merchants. In this case, however, no consideration was provided to keep the offer open, and thus it was not a valid option contract. Answer choice C is incorrect because the offer was effectively revoked when the colleague learned that the book had already been sold. Answer choice D is incorrect because the firm offer rule, under which an offer may be irrevocable if contained in a signed writing, applies only when the offeror is a merchant. For the purposes of the firm offer rule, a merchant includes not only a person who regularly deals in the type of goods involved in the transaction, but also any businessperson when the transaction is of a commercial nature. Because the chemistry professor was not a merchant with respect to an autographed first edition novel, and was engaged in a personal rather than commercial transaction, the fact that the offer was in a signed writing is not relevant.

A general contractor was preparing a bid to build a "green home" designed to be environmentally friendly and sustainable. The general contractor received five bids from subcontractors ranging from $22,000 to $29,000 for the installation of solar panels on the roof of the house. In computing his own bid, the general contractor used the lowest bid of $22,000 for installation of the solar panels. The general contractor was awarded the contract to build the green home. After winning the bid, another solar panel installer approached the general contractor and offered to install the solar panels for $20,000. The general contractor entered into a contract with this installer to mount solar panels onto the roof of the green home. The subcontractor who submitted the $22,000 bid sued the general contractor for breach of contract. Will he succeed? (a) No, because his bid was higher than the bid accepted by the general contractor. (b) No, because his bid was never accepted. (c) Yes, because the general contractor was bound to accept his bid. (d) You Selected: Yes, because of detrimental reliance.

Answer choice B is correct. Because a sub-bid is only an outstanding offer, the general contractor is not bound to accept it upon becoming the successful bidder for the general contract. A general contractor can enter into a subcontract with another subcontractor for a lower price. In this case, the subcontractor's $22,000 bid was only an offer. As a result, the general contractor is not bound to accept it upon becoming the successful bidder for the contract. Answer choice A is incorrect. The difference in amount between the bid ultimately accepted by the general contractor and the subcontractor's bid has no bearing on the subcontractor's legal rights. Even if the subcontractor's price for the work was the lowest overall (including the bid offered by the solar panel installer after the contract was awarded), the general contractor would not be contractually obligated to use the subcontractor's services. Answer choice C is incorrect because the general contractor is not bound to accept the $22,000 bid even though the general contractor relied on that bid in making its own bid. By doing so, the general contractor does not commit itself to accept that bid. Answer choice D is incorrect. Although the subcontractor may not be able to revoke a bid on which a general contractor has relied in making its own bid, the reverse is not required by contract law. Moreover, under these facts, there is no indication that the subcontractor who bid $22,000 detrimentally relied upon a promise by the general contractor to accept that bid if the general contractor was awarded the general contract.

A candy store sent an order on December 1 to a jelly bean manufacturer for 500 bags filled with red, heart-shaped jelly beans in anticipation of Valentine's Day on February 14. The order stated that the jelly beans should not arrive earlier than January 15 in order to ensure their freshness, but no later than February 1, in order to maximize sales before Valentine's Day. The order also clearly stated in bold lettering that the candy store could cancel the order at any time before December 15. The jelly bean manufacturer mailed an acceptance of the order, which was received by the candy store on December 3. On January 20, the candy store received the 500 bags filled with red, heart-shaped jelly beans from the manufacturer in compliance with the order. However, the candy store refused to accept the jelly beans. Is the candy store in breach of contract? (a) Yes, because the candy store's right to cancel the order was a condition subsequent. (b) Yes, because an enforceable contract existed. (c) No, because the manufacturer's shipment was only an offer. (d) No, because the candy store's offer was an illusory promise.

Answer choice B is correct. Because the candy store did not cancel the order before December 15, the order became a valid offer to accept and pay for jelly beans that met the requirements of the order. An offer may generally be accepted by a promise or by performance. Here, the manufacturer accepted the store's offer by delivering jelly beans that conformed to the terms of the order within the time frame set by the order. Consequently, the candy store's refusal to accept the jelly beans constituted a breach of the contract. Answer choice A is incorrect because the candy store's right to cancel the jelly bean order was not a condition subsequent. A condition subsequent exists only with respect to a duty that is absolute. In this case, the candy store's right to cancel prevented the creation of a duty until that right expired, rather than discharging a duty that had already been created. Answer choice C is incorrect because the manufacturer's shipment was made in response to the candy store's outstanding offer. As such, it was an acceptance of the candy store's offer by performance. Answer choice D is incorrect. The candy store's order was initially an illusory promise because the candy store could cancel the order at any time prior to December 15. When the candy store failed to cancel the order before December 15, the order became a valid offer to accept and pay for jelly beans that met the requirements of the order.

A shoe manufacturer contends that the owner of a shoe store called and ordered 50 pairs of Oxford-style dress shoes at $100 per pair to be shipped within three weeks and that the manufacturer's representative immediately accepted this order. The manufacturer promptly sent the owner a signed, written acknowledgment of the alleged order that reflected the manufacturer as seller and the shoe store owner as buyer, as well as the number and style of shoes, but that did not indicate the price of the shoes. The owner admits to receiving the acknowledgment the following day and taking no action regarding it. Two weeks later, the owner received a shipment of 50 pairs of Oxford-style dress shoes. The owner immediately called the manufacturer and asserted that he had never ordered the shoes. Will the Statute of Frauds prevent the manufacturer from enforcing this contract against the owner? (a) No, because an oral contract made between merchants is enforceable. (b) No, because the owner received and did not respond to the written acknowledgment in a timely manner. (c) Yes, because the acknowledgment did not indicate the price of the shoes. (d) Yes, because the price of the shoes exceeds the $500 threshold of the Statute of Frauds.

Answer choice B is correct. Generally, a contract that falls within the Statute of Frauds is unenforceable unless evidenced by a writing. The writing must (i) be signed by the party to be charged and (ii) contain the essential elements of the deal. A contract for the sale of goods for a price of at least $500 falls with the Statute of Frauds. Consequently, the purported order of 50 pairs of shoes at $100 per pair is subject to the Statute of Frauds. Although the acknowledgment sent by the manufacturer to the owner is otherwise sufficient to satisfy the Statute of Frauds, it was not signed by the owner. However, if both parties are merchants and a memorandum sufficient against one party is sent to the other party, who has reason to know its contents, and the receiving party does not object in writing within 10 days, then the contract is enforceable against the receiving party even though he has not signed it. Here, this merchant's exception applies to the owner who received the acknowledgment of his order and did not reply for more than 10 days after receiving it. Answer choice A is incorrect. Although a memorandum of an oral agreement sent by one merchant may permit the enforcement of the agreement against another merchant, there is no blanket exception to the Statute of Frauds that permits enforcement of an oral agreement between two merchants. Answer choice C is incorrect. Under the Statute of Frauds for the sale of goods, the price of the goods need not be included in the contract. If the price is omitted, the parties can present evidence of the agreed-upon price or (if that fails) the market price. Therefore, the failure of the acknowledgment to reflect the price of the shoes does not prevent the acknowledgment from satisfying the merchant's exception to the Statute of Frauds. Answer choice D is incorrect. Although the Statute of Frauds for a sale of goods does apply to this purported transaction and was not satisfied because the owner did not sign the acknowledgment, the acknowledgment sent by the manufacturer satisfies an exception to the Statute of Frauds when both parties to the oral agreement are merchants.

A men's apparel wholesaler was trying to expand its business, so it reached out to an online company that sold men's ties. The wholesaler mailed a letter to the online company offering to sell them 1,000 silk ties at a wholesale price of $15 per tie. The signed letter, dated July 1, assured the online company that the option to purchase would stay open, but did not specifically state how long the option would remain open. A year later, the online company sent a letter to the wholesaler accepting its offer to sell them 1,000 silk ties at a wholesale price of $15 per tie. The wholesaler and online company had no prior dealings, and offers of this kind in the industry generally do not remain open for a year with no further contact between the parties. If no other correspondence or action was taken by either party between the wholesaler's offer and the online company's purported acceptance, is there an enforceable contract between the parties? (a) No, because the online company did not give consideration to keep the offer open. (b) No, because the online company did not accept the offer within a reasonable period of time. (c) Yes, because the time period of irrevocability was not stated. (d) Yes, because the wholesaler's offer had no termination date.

Answer choice B is correct. If an offer does not set a time limit for acceptance, the power of acceptance terminates at the end of a reasonable period of time. What is reasonable is a question of fact and depends on a variety of factors, including the nature of the contract, the purpose and course of dealing between the parties, and trade usage. In this case, under the UCC firm offer rule, the online company had a maximum of three months to accept the wholesaler's offer. However, it would not be reasonable to extend the time for acceptance indefinitely after the end of the three-month period. Here, it would be unreasonable for the online store to expect the offer to still be open a year after the offer was made. Therefore, the offer has terminated by lapse of time. Answer choice A is incorrect. Under the UCC firm offer rule, no consideration by the offeree is needed to keep the offer open. Consideration can be the basis of an option contract under the common law. However, an option is a promise to keep an offer open for a definite period of time. Thus, this promise to keep the offer open for an unspecified period of time cannot be the basis of an option contract, even if the company gave consideration. Answer choice C is incorrect. Under the UCC firm offer rule, an offer to buy or sell goods is irrevocable if the offeror is a merchant, there is an assurance that the offer is to remain open, and the assurance is contained in signed writing from the offeror. If the time period during which the option is to be held open is not stated, a reasonable term is implied, but irrevocability cannot exceed three months. Here, the wholesaler and the online company are merchants, the wholesaler gave assurances that the offer would remain open, and the assurances were contained in the wholesaler's signed letter. The letter did not state a time period during which the offer would remain irrevocable. Accordingly, after three months, the wholesaler could have revoked the offer. Moreover, the offer was no longer open after a year because an unreasonable amount of time had passed. Answer choice D is incorrect because if the offer does not set a time limit for acceptance, the power of acceptance terminates at the end of a reasonable period of time.

A woman sent an offer to sell her office printer to her friend for $450. In her offer, the woman mentioned that an acceptance could be mailed to her business address, and that the friend should let her know within the next couple of weeks whether she was interested. The friend needed an office printer, so she immediately accepted the woman's offer by mailing a letter to the woman's home address. The woman only checked her mailbox at home once a week because she received so much junk mail, so she did not see the acceptance letter. Thinking that her friend was not interested, the woman sold her office printer to a different person. The next day, the friend came to the woman's house with a check for $450. The woman told the friend she had already sold the office printer. Will the friend succeed in an action for breach of contract? (a) Yes, because the offer was irrevocable for at least two weeks. (b) Yes, because the woman did not specify that mailing an acceptance to her business address was the only mode of acceptance. (c) No, because it was not reasonable for the friend to mail her acceptance to the woman at the woman's home address. (d) No, because the woman did not see the friend's acceptance letter.

Answer choice B is correct. The offeror can dictate the manner and means by which an offer may be accepted. Unless the offeror specifically requires the offeree to accept in a particular manner or by using a particular means, the offeree can accept in any reasonable manner and by any reasonable means. Here, the woman did not specify that the acceptance could only be mailed to her business address; she merely suggested that an acceptance could be mailed to her business address. Thus, another reasonable mode of acceptance, such as mailing it to the woman's home address, is permissible. Accordingly, answer choice C is incorrect. Answer choice A is incorrect because offerors may generally revoke offers at any time prior to acceptance, even if the offeror claims that the offer will be held open. Here, the friend would not succeed because the offer was irrevocable, but because she sent her acceptance before the offer was revoked. Answer choice D is incorrect. Under the mailbox rule, an acceptance is valid upon posting, not receipt. Here, the friend's acceptance was effective when she mailed the letter to the woman's home. Upon mailing the letter, an enforceable contract was created between the woman and the friend, even though the woman did not see the friend's acceptance letter.

A jeweler and a goldsmith signed a written agreement that provided as follows: "For $3,000, the goldsmith shall sell to the jeweler a size six gold ring setting that the jeweler shall select from only the goldsmith's white gold ring designs." The agreement did not address any other specific terms with regard to the business arrangement between the jeweler and the goldsmith. When the jeweler arrived to select a ring, he refused to select one, claiming that the goldsmith, immediately prior to the execution of the written agreement, had orally agreed to broaden his choices to also include rose gold ring designs. The jeweler claimed that the goldsmith had, at the same time, also orally agreed to include a set of earring settings, valued at $1,000, as an incentive for his continued business. The goldsmith refused to sell to the jeweler any of his rose gold ring designs or include the earring settings. If the jeweler sues the goldsmith for damages, how should the court handle the evidence of the alleged oral agreements? (a) The court should admit the evidence as to both the earring settings and the option to choose a rose gold ring design. (b) The court should admit the evidence as to the earring settings but not the option to choose a rose gold ring design. (c) The court should admit the evidence as to the option to choose a rose gold ring design but not the promise to include the earring settings. (d) The court should admit neither the option to choose a rose gold ring design nor the promise to include the earring settings.

Answer choice B is correct. The parol evidence rule generally prevents a party to a written contract from presenting prior extrinsic evidence that contradicts the terms of the contract as written. If the writing completely expresses all of the terms of the parties' agreement, then it is a total integration, and the parties cannot introduce any extrinsic evidence (oral or written) of prior or contemporaneous understandings or negotiations. If, on the other hand, the writing sets forth the parties' agreement about some terms, but not all terms, then it is a partial integration. The UCC essentially presumes that a contract is a partial integration. Here, the UCC applies, so the contract will be presumed to be a partial integration, allowing additional consistent terms unless they certainly would have been included. The agreement between the jeweler and the goldsmith set forth only the size, price, and type of gold for the gold ring setting; it did not set forth any other relevant terms to their business agreement. When a writing is a partial integration, the parties are permitted to introduce supplementary extrinsic evidence (oral or written) of other terms as long as the evidence is consistent with the writing, but not if the evidence contradicts the terms of the writing. Here, the evidence about the earring settings only supplements the written agreement. However, the evidence about the option to choose a rose gold ring design directly contradicts the written agreement, which explicitly states that the jeweler can choose only from the white gold ring designs. Therefore, only the evidence as to the earring settings should be admitted. Answer choices A and C are incorrect, as the evidence about the option to choose a rose gold ring design expressly contradicts the written agreement and thus cannot be admitted. Answer choice D is incorrect, as the evidence about the earring settings only supplements the written agreement and thus would be admissible under the parol evidence rule.

The owner of a restaurant who highlighted local ingredients when creating his menu bought cheese and other dairy products from a local dairy farmer. The owner and the farmer had entered into written requirements contracts each spring for the past ten years. In the winter of the tenth year, the farmer purchased a substantial amount of new dairy cows and expanded his farming capabilities. He notified all customers that he would have a higher volume and amount of available products the following spring, and would adjust deliveries accordingly. The owner responded with a date he wished for the products to be delivered, as per custom, but said nothing else. On the agreed upon date, the farmer delivered substantially more products than he would customarily provide. The owner attempted to accept half of the shipment, as that was roughly his customary quantity, but the farmer stated that the products were already packaged and that the owner should have spoken up after receiving the notice from the farmer. The owner then rejected the shipment in its entirety. Did the owner breach the contract with the farmer as to this shipment? (a) No, because no contract existed, as the parties did not agree to a quantity. (b) No, because the farmer made a nonconforming tender of goods. (c) Yes, because the owner should have given the farmer time to cure the nonconformity. (d) Yes, because the owner rejected the shipment in its entirety.

Answer choice B is correct. The parties here had a requirements contract, and the farmer delivered much more than the owner reasonably required, thereby making a nonconforming tender of goods. While some variation is permissible in requirements contracts, the farmer supplied double the amount of customary products, which is unreasonable, especially considering the shelf life of dairy products. Upon receiving a nonconforming tender of goods, the owner had the right to accept or reject all or part of shipment. Here, the owner tried first to accept half of the goods, and then rejected all of the nonconforming tender after the farmer did not allow him to do so. Answer choice A is incorrect because this is a requirements contract, under which a buyer agrees to buy all that he will require of a product from the other party. Because a covenant of good faith and fair dealing is implied in all contracts (common law and UCC), any quantities under such a contract may not be unreasonably disproportionate to any stated estimates, or if no estimate is stated, to any normal or otherwise comparable prior requirements or output. Here, no quantity was stated, but the court could reference prior performance between the farmer and the owner. Answer choice C is incorrect because a seller only has a right to cure a nonconforming tender if the time for performance has not yet passed. Here, the time for performance was the date of the delivery, so the farmer did not have a right to cure; in any case, the farmer refused to tender a conforming amount when requested by the owner. Answer choice D is incorrect because the owner had the right to accept or reject part or all of the farmer's nonconforming tender.

A retailer received a written firm offer signed by a supplier. The offer committed the supplier to providing the retailer with up to 10,000 tubes of toothpaste over the next 45 days at $1 a tube. Thirty days later, the supplier informed the retailer that the price per tube of toothpaste would be $1.10. The next day the retailer ordered 6,000 tubes of toothpaste from the supplier, which the supplier promptly shipped. Sixty days after the receipt of the offer, the retailer ordered another 4,000 tubes of toothpaste, which the supplier also promptly shipped. What price is the supplier permitted to charge the retailer for the toothpaste? (a) $10,000 (10,000 * $1), because the supplier's firm offer was effective for three months regardless of its terms. (b) You Selected: $10,400 ((6,000 * $1) + (4,000 * $1.10)), because the supplier's firm offer was effective for only 45 days. (c) $11,000 (10,000 * $1.10), because the firm offer rule does not apply where the buyer is a merchant. (d) $11,000 (10,000 * $1.10), because the supplier informed the retailer that the price was increased to $1.10 before the retailer's placement of either order.

Answer choice B is correct. The supplier's written and signed firm offer was effective and irrevocable for 45 days, even though no consideration was provided by the retailer to the supplier for making the firm offer. Consequently, the price for the first shipment of 6,000 tubes of toothpaste was $1 per tube. Since the firm offer had expired prior to the placement of the second order, the supplier's price of $1.10 per tube was in effect with regard to this order. Answer choice A is incorrect because, while a firm offer made by a merchant may be effective up to three months, the merchant offeror, as with any offeror, is master of the offer. Consequently, the merchant offeror could set the time period for the firm offer at 45 days. Answer choice C is incorrect because the firm offer rule applies, even where the buyer is a merchant, as long as the seller is a merchant. Answer choice D is incorrect because, as noted with regard to answer choice B, the firm offer made by the supplier applies to the first order placed by the retailer because such offer was irrevocable.

A homeowner met with a number of general contractors regarding significant renovations on his home. After a couple of successful meetings with one particular contractor, the contractor sent a letter to the homeowner stating that the contractor would perform the renovation work for $10,000. The homeowner responded with a letter stating that he would "only pay $8,500 and not more." Upon receiving the homeowner's letter, the contractor immediately commenced preparations for the renovation work, including beginning to acquire the materials necessary to complete the renovation. One week after beginning work on the homeowner's home, the contractor realized the renovation would cost $1,600 more than he anticipated due to the homeowner's very particular aesthetic requirements. After being informed of the higher cost, the homeowner refused to pay the contractor any amount over $8,500. The contractor promptly discontinued working on the renovation. The homeowner subsequently brought a breach of contract action against the contractor, seeking damages. Which party will prevail? (a) The contractor, because the homeowner's offer could only be accepted by a return promise. (b) The contractor, because the unexpected costs discharged his obligation to complete performance. (c) The homeowner, because the homeowner's offer was effectively accepted by commencement of performance. (d) The homeowner, because the unilateral contract between the parties is enforceable.

Answer choice C is correct. A bilateral contract is one in which a promise by one party is exchanged for a promise by the other. An offer requiring a promise to accept can be accepted either with a return promise or by starting performance, unless the offer specifies the mode of acceptance. Commencement of performance of a bilateral contract operates as a promise to render complete performance. In this case, the homeowner's offer was one for a bilateral contract that the contractor could accept by a return promise or by starting performance. The contractor accepted the homeowner's offer by commencing performance. By doing so, the contractor promised to render complete performance. Answer choice A is incorrect because the homeowner's offer could be accepted either with a return promise or by starting performance. Answer choice B is incorrect. Unforeseen natural disasters, wars, trade and military embargoes, strikes, and local crop failures have all been found sufficient to excuse performance. However, non-extraordinary increases in the cost of performance do not excuse performance. Answer choice D is incorrect because an enforceable bilateral contract was formed based on the commencement of performance by the contractor. Additionally, even if the homeowner had made a unilateral offer, the offer could only have been accepted by the contractor's completed performance, and the contractor would not be contractually bound to complete any performance he merely began.

On April 1, a buyer agreed in writing to purchase an antique car from a seller for $20,000. The parties met on April 10, the scheduled date of the sale, at which time the buyer accepted the car and gave the seller a check for $15,000. The buyer, seeking to create an accord and satisfaction, had added the following conspicuous notation to the check: "This check is in full and final satisfaction of my obligation under our April 1 agreement." The seller did not realize that the check was for only $15,000 and that it contained the notation until the seller sought to deposit it at her bank later that day. Needing the money, the seller deposited the check anyway. If the seller sues the buyer for breach of contract seeking damages of $5,000, the difference between the amount paid and the contract price, will the buyer's accord and satisfaction defense likely succeed? (a) No, because the buyer could not modify the agreement without consideration. (b) No, because the buyer did not dispute the initial purchase price of the car. (c) Yes, because the notation on the check formed a substituted contract. (d) Yes, because the seller deposited the check knowing it was offered in full and final satisfaction of the buyer's obligation.

Answer choice B is correct. Under an accord and satisfaction, a party can fulfill its contractual obligation by rendering a different performance than the one initially promised. This can be accomplished by a negotiable instrument (e.g., check) if three conditions are met: (1) the obligation must be unliquidated (i.e., uncertain in amount) or otherwise in dispute; (2) the obligor must, in good faith, tender the negotiable instrument with a conspicuous statement that the instrument is tendered as full satisfaction of the obligation; and (3) the obligee must obtain payment of the instrument (e.g., by cashing the check). Here, the seller (obligee) cashed the buyer's (obligor's) $15,000 check, which contained a conspicuous statement that it was "in full and final satisfaction" of the buyer's obligation to purchase the car for $20,000. However, the buyer did not dispute the initial purchase price of the car. Absent a dispute, the check could not have been offered in good faith. Therefore, the buyer's accord and satisfaction defense is unlikely to succeed. Answer choice A is incorrect. Under the UCC, which governs contracts for the sale of goods (e.g., car), a contract can be modified without consideration. Therefore, the fact that the buyer did not provide consideration is irrelevant. Answer choice C is incorrect. The buyer did not raise a substituted contract as a defense. But even if he had, the notation on the check is likely insufficient to form a substituted contract, which requires greater formality than an accord and satisfaction. Answer choice D is incorrect. For an accord and satisfaction, the seller did not need to actually know that the check was offered in full and final satisfaction of the buyer's obligation. Instead, the check needed to have a conspicuous statement to that effect, which it did. But the defense will still fail because the purchase price of the car was undisputed.

A dancer signed a contract with a traveling circus to travel and perform as an aerialist for six months. The contract provided that the dancer would be paid $500 per week and would be guaranteed employment for the full six months, with an option to renew the contract for the next traveling season. Excited for the opportunity to perform for a traveling circus, the dancer turned down an invitation to dance with a theatre group for the same time period as the circus contract. After two weeks of traveling and dancing for the circus, the dancer sprained her ankle and was briefly hospitalized for one week. The circus was forced to hire another aerialist. After an additional week, the dancer's doctor gave her approval to return to work, but the circus refused to honor the remainder of the contract. The dancer brought an action against the circus for breach of contract. If the dancer wants to recover the highest possible amount of damages, which of the following is the dancer's best legal theory? (a)The dancing contract with the circus is legally severable into weekly units. (b) The dancer's failure to perform for two weeks was not a material breach of the contract. (c) The dancer's performance of the terms of the contract is impracticable given her injury. (d) The dancer detrimentally relied on the contract by declining the other dancing job.

Answer choice B is correct. Under common law, a material breach of contract occurs when the nonbreaching party does not receive the substantial benefit of its bargain. A material breach of contract allows the nonbreaching party to withhold any promised performance and to pursue remedies for the breach, including damages. A breach is considered minor when the breaching party has substantially performed. In this case, if the dancer is able to successfully argue that a two-week delay is only a minor breach and that she has otherwise substantially performed under the contract, then she will be able to recover damages. Answer choice A is incorrect because a severable contract would work against the dancer's desired recovery. A divisible or installment contract is one in which the various units of performance are divisible into distinct parts. Recovery is limited to the amount promised for the segment of the contract performed. Thus, if the contract was severable into weekly units, then the dancer would not have a strong claim to enforce the remainder of the contract, and she would be able to recover only for the two weeks that she did perform for the circus before injuring herself. Answer choice C is incorrect. A party's duty to perform can be dismissed by impracticability. In a personal-services contract, as is the case here, performance is considered impracticable if the performing party to the contract dies or becomes incapacitated. Here, if a court found that the dancer's performance under the contract was impracticable due to her injury, then it would result in a discharge of the contract, which would work against the dancer's desired recovery. Answer choice D is incorrect. Promissory estoppel is referred to as a "consideration substitute." The doctrine of promissory estoppel (detrimental reliance) can be used under certain circumstances to enforce a promise that is not supported by consideration. This doctrine would not help the dancer because there was an enforceable contract here that was supported by consideration. Estoppel is a consideration substitute, and this would not help the dancer recover damages.

A wholesaler of bicycle chains sent a retailer the following fax on December 1: "Because of your continued loyalty as a customer, I am prepared to sell you up to 1,000 units of Bicycle Chain Model D at $7.50 per unit, a 25% discount off our original $10.00 price. This offer will remain open for 7 days." The fax lacked a full, handwritten signature, but was on the wholesaler's letterhead and had been initialed by the wholesaler's head of sales. On December 4, the wholesaler's head of sales called the retailer and informed the retailer that he had decided to revoke his December 1 offer. On December 5, the retailer placed an order for 1,000 bicycle chains, stating that he would pay the discounted price of $7.50 per unit. What is the correct value of the order placed by the retailer? (a) $7,500, because the wholesaler's revocation was not in writing. (b) $7,500, because the wholesaler was bound to keep the offer open for 7 days. (c) $10,000, because the offer was not signed by the wholesaler. (d) $10,000, because the retailer did not provide consideration to hold the offer open.

Answer choice B is correct. Under the UCC's firm offer rule, an offer to buy or sell goods is irrevocable if the offeror is a merchant, there is an assurance that the offer is to remain open, and the assurance is contained in a signed writing from the offeror. No consideration by the offeree is needed to keep the offer open. Here, all three conditions are satisfied (note that letterhead and an agent's initials suffices as a "signature"), and the wholesaler's offer was irrevocable until the offer period expired. The retailer's order was within the 7-day window, and thus the order for 1,000 units is priced at $7.50 each, or $7,500 total. Answer choice A is incorrect because the wholesaler was bound by the UCC firm offer rule, and his offer was irrevocable regardless of the method of his attempted revocation. Answer choice C is incorrect because the initials of the head of sales on the letterhead are sufficient to constitute a signed writing under the UCC. Answer choice D is incorrect because, under the UCC's firm offer rule, no consideration was required to hold the offer open.

A construction company contracted with a manufacturer to purchase 100 identical prefabricated windows to use while constructing houses in a gated community. The windows were to be delivered in shipments of 25 windows each on April 1, May 15, July 1, and August 15. The written contract, signed by both parties, was silent as to when payment for each shipment would be due. The manufacturer made the first two shipments in conformity with the contract requirements, and the construction company paid one-fourth of the full contract price upon each delivery. However, on June 1, the manufacturer demanded that the construction company pay the entire remainder of the contract price before the manufacturer would make any further shipments. Which of the following statements is true? (a) The construction company has no duty under the contract to make any payments until the final delivery is made. (b) The construction company must pay the manufacturer one-fourth of the contract price upon delivery of each conforming shipment of windows. (c) The construction company's failure to pay the requested sum will amount to a repudiation of the contract. (d) The manufacturer waived his right to demand immediate payment of the full contract price when he accepted the first payment of one-fourth the contract price on April 1.

Answer choice B is correct. Under the UCC, an installment contract is defined as one in which the goods are to be delivered in multiple shipments, and each shipment is to be separately accepted by the buyer. Payment by the buyer is due upon each delivery, unless the price cannot be apportioned. Therefore, the construction company is only obligated to pay the manufacturer an apportioned price for each conforming delivery (i.e., one-fourth of the full contract price). Answer choice A is incorrect because the price of these windows can be easily apportioned between the four shipments. Therefore, an apportioned payment is due upon each delivery. Answer choice C is incorrect because the construction company's denial of the manufacturer's demand for immediate payment of the remainder of the contract price will not amount to a clear and unequivocal refusal to perform under the contract. Therefore, this conduct would not operate as a repudiation. Answer choice D is incorrect. Under the UCC, payment for installment contracts is due upon each delivery, unless the price cannot be apportioned. Parties cannot vary or contract out of this definition under the code. Accordingly, the manufacturer did not have the right to demand immediate payment of the full contract price at any time.

An employee signed an employment contract with a company to be a remote salesman. The contract provided the employee's yearly salary and a requirement that he work at least 30 hours per week, but it did not provide any guarantees about how or when a potential bonus would be paid or earned. The contract also required the employee to log his monthly sales figures online by the last business day of the month. After the employee worked remotely for the company for a few months, the company called the employee on March 15. Over the phone, a representative of the company explained that the company was establishing a new bonus program for employees who reached certain monthly sales goals. The representative explained that in addition to logging sales online, the employee must call the company's central office on the first Friday of each month before the end of business hours and report his sales figures for the previous month in order to receive a bonus. The employee worked diligently for the remainder of March to meet the required sales figures in order to receive a bonus, working approximately 40 hours per week. He logged his sales online at the end of March, as required by his contract. On the first Friday in April, the employee called the company to report his March sales figures. However, the company had disconnected its phone lines and did not receive the employee's call. The company subsequently refused to pay the employee a bonus for his work in March. Will the employee likely prevail in a breach-of-contract action against the company for his unpaid March bonus? (a) No, because the employee's performance of his preexisting legal duty to sell merchandise for the company will not qualify as consideration. (b) No, because the employee failed to notify the company during his conversation on March 15 that he had accepted the offered bonus plan. (c) Yes, because the company itself prevented the employee from satisfying the express condition of the March 15 offer. (d) Yes, because there is no evidence that the March 15 oral agreement between the employee and the company was unenforceable.

Answer choice C is correct. A performance that is subject to an express condition cannot become due unless the condition occurs or its nonoccurrence is excused. However, a condition's nonoccurrence is excused under the doctrine of prevention when a nonperforming party wrongfully prevents or interferes with the occurrence of a condition. Here, the employee's entitlement to the bonus was subject to two conditions: attaining the requisite sales figures and reporting those figures in a specified manner. The employee satisfied the first condition by meeting his March sales goal. The employee was unable to satisfy the second condition by properly reporting his sales figures because the company disconnected its phone lines. Thus, the prevention doctrine would forbid the company from raising the failure of the condition precedent—reporting the sales figures—as grounds to avoid paying the bonus. Because the company prevented the employee from reporting his sales figures and thus fulfilling the condition, the company cannot rely on the nonoccurrence of that condition to defeat its liability. Answer choice A is incorrect because if the promisor gives something in addition to what is already owed or varies the preexisting duty in some way, however slight, most courts find that consideration exists. Here, even though he is performing the same kind of work as his employment contract demands, the employee worked more hours than his preexisting legal duty required. Answer choice B is incorrect. In a unilateral contract, an offeree is not required to give notice after performance is complete, unless he has reason to know that the offeror would not learn of performance within a reasonable time, or the offer requires notice. Such is not the case here because the company would have learned that the employee had met the sales requirement at the end of March when he logged his sales online. Answer choice D is incorrect because it fails to address the employee's failure to properly report his sales figures on the first Friday in April due to the company's disconnection of its phone lines.

On May 1, a clothing manufacturer sent a written offer to a retailer for the sale of 1,000 pairs of designer jeans at a price of $50 per pair, including delivery costs. The proposed delivery date was June 15. On May 5, the retailer mailed a letter accepting the manufacturer's offer. On May 15, a natural disaster occurred causing fuel prices to significantly increase. As a result, the clothing manufacturer sent a letter to the retailer requesting an additional $500 to cover the increased delivery and transportation costs for the shipment of jeans. The retailer considered seeking out a new supplier, but hoped to continue doing business with this manufacturer in the future, so decided against it. The retailer returned a signed letter promising to pay the additional $500. On June 15, the clothing manufacturer delivered 1,000 pairs of jeans to the retailer. The retailer paid the clothing manufacturer $50,000, but refused to pay the additional $500. Can the manufacturer enforce the retailer's promise to pay an additional $500? (a) Yes, because the manufacturer relied on the promise when it delivered the jeans to the retailer. (b) Yes, because the manufacturer made its request for the additional $500 in good faith. (c) No, because the attempted modification was unconscionable. (d) No, because no consideration supported the promise to pay the additional $500.

Answer choice B is correct. Unlike under the common law, under Article 2, no consideration is necessary to modify a contract; however, good faith is required. Good faith requires honesty in fact and fair dealing in accordance with reasonable commercial standards. If a party demands an increase in price because the other party has no choice but to agree, the courts will invalidate such a bad-faith modification. Here, a natural disaster occurred causing fuel prices to significantly increase. As a result, the clothing manufacturer's request for an additional $500 to cover the increased delivery and transportation costs was made in good faith, based on the natural disaster. Also, the retailer was not forced into the agreement because other suppliers were available. Therefore, the attempted modification was valid, and the manufacturer can enforce the retailer's promise to pay an additional $500 without reliance or additional consideration. Answer choice A is incorrect. Promissory estoppel is not necessary here because there was an enforceable modification of the contract. Thus, the manufacturer can enforce the promise under the valid contract and need not show reliance. Answer choice C is incorrect because the request for an additional $500 does not make the modified contract unconscionable. A contract (or part of a contract) is only unconscionable when it is so unfair to one party that no reasonable person in the position of the parties would have agreed to it. Answer choice D is incorrect because contracts under the UCC, such as the one here, do not require consideration for modification; rather, only good faith is required.

The owner of a rare eighteenth-century chest offered to sell it to a connoisseur of antiques for $75,000. The connoisseur countered that she would buy the chest for $50,000. The owner rejected this price. The owner and the connoisseur then executed a written agreement for the sale of the chest at a price to be determined only by an antiques dealer whose expertise in valuing this rare item they both trusted. The dealer examined the chest. He told the owner and the connoisseur that he had to do further research on the chest, but that he would let them know his decision in several days. Unfortunately, the dealer died before doing so. A reasonable price can be established for the chest by the court. Is there likely an enforceable contract? (a) No, because the price of the chest was not determined at the time the agreement was executed. (b) No, because the owner and the connoisseur did not intend to be bound unless the dealer set the price of the chest. (c) Yes, because a reasonable price can be established for the chest by the court. (d) Yes, because the owner and the connoisseur executed a written agreement for the sale of the chest.

Answer choice B is correct. When parties enter into an agreement for the sale of goods and the price has not been set, there is no contract if the agreement reflects an intent not to be bound unless the price is subsequently set and the price is never set. Here, the owner and the connoisseur agreed to the sale of the chest only at a price set by the dealer. Because the dealer did not set a price before his death, there is no enforceable agreement for the sale of the chest. Answer choice A is incorrect because such an agreement may be enforceable despite an unresolved price at the time that the agreement is entered into, as long as the price is arrived at in the manner agreed to by the parties. Here, had the dealer named a price for the chest, there would have been a contract that could have been enforced by either party against the other. Answer choice C is incorrect. Although a court may fix a reasonable price for goods when the parties to a contract fail to do so, the parties must intend to enter into a contract for the goods despite the absence of the price term. Here, the owner and the connoisseur did not ignore the price term, but rather set out a specific and exclusive method by which the price was to be determined. Because, due to the death of the dealer, that method cannot be used to determine the price, and the parties did not provide for an alternative method, the implication is that the parties did not intend to be bound if that specified method failed. Answer choice D is incorrect. Although the owner and the connoisseur did execute a written agreement, this agreement left the price of the chest to be determined by the dealer. Because the dealer's death makes it impossible for him to do so, the agreement fails. The parties' disagreement over the price of the chest prior to entering into the agreement also suggests that the parties did not have the intent to be bound if the dealer were unable to determine a price.

A new florist placed a written order with a wholesaler for $15,000 worth of fresh flowers. Delivery was to be made to the florist's shop via a national delivery service. Because the florist was a new customer, the wholesaler accepted the order on the condition that he pay $5,000 in advance, and the remaining $10,000 within 20 days of delivery. There was no discussion as to who bore the risk of loss. The wholesaler arranged with a national delivery service to pick up and deliver the flowers to the florist. The delivery service picked up the flowers, but, due to malfunction of the temperature controls on the transporting plane, the flowers were worthless upon arrival. The florist rejected the flowers and notified the wholesaler, who refused to ship other flowers. The wholesaler filed a claim against the florist for the remaining $10,000. The florist counterclaimed for the return of its $5,000 payment to the wholesaler. How should the court rule on these claims? (a) Grant the wholesaler's claim for $10,000 and deny the florist's claim for $5,000 because the risk of loss passed to the florist. (b) Grant the florist's claim for $5,000 and deny the wholesaler's claim for $10,000 because the risk of loss remained with the wholesaler. (c) Offset the two claims against each other and require the florist to pay the wholesaler $2,500 because, since neither party was at fault for the loss, each should bear the loss equally. (d) Deny both claims because the florist accepted the risk of loss up to the amount he had paid for the goods.

Answer choice B is correct. When the goods are required to be delivered to a specific place (e.g., the buyer's place of business) and delivery is to be made by a third-party carrier (i.e., a destination contract), the risk of loss does not pass to the buyer until the goods are tendered to the buyer at the designated place. Because this was a destination contract, the risk of loss remained with the wholesaler because the flowers were destroyed prior to reaching the buyer. Due to the condition of the flowers, the wholesaler failed to make a perfect tender, and the florist had the option to reject the goods. Consequently, the florist is entitled to a return of its $5,000 payment. When the wholesaler refused to ship other goods despite notification of the florist's rejection, the florist could seek a return of his payment. Answer choice A is incorrect because, although a shipment contract (i.e., a contract that requires the seller to deliver the goods to a third-party carrier rather than the buyer) is the default type of contract, the parties are free to specify the place of delivery. In this case, the parties entered into a destination contract specifying the local florist as the delivery location. As a consequence, the risk of loss did not pass to the local florist. Answer choice C is incorrect because, although the loss of the flowers was not due to the fault of either party, the risk of loss is not shared in such case. Answer choice D is incorrect because payment, in part or full, by the buyer of the purchase price does not shift any portion of risk of loss to the buyer.

A mechanic and a farmer contracted in writing for the repair of the farmer's tractor, with a payment of $2,000 due upon completion. The mechanic called the farmer on April 15 to inform the farmer that the work was complete. When the farmer went to pick up the tractor the next day, he told the mechanic that due to an unforeseen rise in feed costs, he couldn't pay the full contract price. The farmer paid the mechanic the first $1,000, and the mechanic told the farmer that, if the farmer promised to pay the remainder by June 1, then the mechanic would not sue to recover the remaining $1,000. The farmer orally agreed. On May 1, the mechanic sued the farmer for the unpaid $1,000, and the farmer filed a motion to dismiss. Should the court grant the motion to dismiss? (a) No, because the new cost of feed is not an unforeseen difficulty that would allow for modification of the existing contract. (b) No, because there is no consideration to support the mechanic's promise not to sue. (c) Yes, because a promise to allow a debtor to delay payment on a past debt is enforceable without consideration. (d) Yes, because the payment of $1,000 was sufficient consideration to support the mechanic's promise not to sue.

Answer choice B is correct. While the law permits the settlement of debts, consideration is required for a settlement to be enforceable. Under the preexisting duty rule, the mechanic's promise to forbear from suing to collect was not supported by consideration from the farmer, because the amount due was liquidated and the farmer did not promise to do anything more than he was already obligated to do. The farmer did pay $1,000, but as a partial payment of an undisputed debt, that money is insufficient consideration for the mechanic's promise not to sue. Answer choice A is incorrect because impracticability would only apply as a defense to the original repair contract, not the forbearance agreement. Answer choice C is incorrect because a promise to allow a debtor to delay payment on a past debt is not enforceable without consideration. Answer choice D is incorrect because the payment of $1,000 of the $2,000 owed was a partial payment of an undisputed debt, and cannot serve as consideration for the mechanic's agreement not to sue the farmer.

The owner of a fur coat stored it with the furrier from whom she bought the coat during the warm months of the year. While the coat was at the furrier, a salesperson, mistakenly thinking that the coat was for sale, sold it to a customer. The customer was allowed to reduce the purchase price by the amount of an outstanding debt owed by owner of the furrier to the customer; the customer paid the remainder in cash. In the process of purchasing the coat, the customer was told by the salesperson about the furrier's storage service, but, like the salesperson, was unaware that the coat was not part of the store's merchandise. After the sale, the owner learned of the transaction between the furrier and the customer. Since the coat had significant sentimental value to the owner, the owner sought its return from the customer. When the customer refused, the owner filed an action to recover the coat from the customer. Will the owner prevail? (a) Yes, because the furrier transferred only voidable title in the coat to the customer. (b) Yes, because the customer did not give full value in acquiring the coat. (c) No, because the customer was a good faith purchaser of the coat that had been entrusted to the furrier. (d) No, because the owner is entitled to damages from the furrier.

Answer choice C is correct. A good faith purchaser of goods in the ordinary course of business from a merchant takes good title to the goods if the goods have been entrusted by the owner to the merchant and the merchant deals in the same kind of goods. Here, the customer bought the fur coat at a furrier where it had been left by the owner. The customer was unaware that the coat belonged to the owner. Answer choice A is incorrect because although the furrier did not have title to the goods, the furrier could nevertheless transfer good title. Answer choice B is incorrect because the purchaser did give value: satisfaction of a pre-existing debt constitutes value. Answer choice D is incorrect because the owner's right to seek damages from the furrier does not, in itself, preclude the owner from seeking to recover the coat from the customer. The entrustment of the coat to a furrier and the subsequent sale of that coat to a good faith purchaser, however, does.

A licensing agreement provided that a manufacturer could use an inventor's patent in manufacturing its products for ten years. Immediately thereafter, the inventor assigned his rights to receive payments pursuant to the licensing agreement to a corporation in which he was the controlling shareholder. The inventor did not receive compensation for this assignment. The inventor, upon his death five years later, devised his stock in the corporation to his daughter, and all of his remaining property to his son. To whom should the manufacturer make its payments under the licensing agreement? (a) The corporation (b) The inventor's daughter (c) The inventor's son (d) The manufacturer's obligation to make payments under the licensing agreement terminated upon the death of the inventor.

Answer choice C is correct. A gratuitous assignment of contract rights automatically terminates upon the death of the assignor. Accordingly, the assignment to the corporation was revoked upon the inventor's death. However, a contract generally does not terminate upon the death of one of the parties. Thus, the manufacturer remains obligated to make its payments under the licensing agreement to a person who, by the terms of the inventor's will, has been devised the right to receive those payments. Because the right to the inventor's property, other than his corporate stock, was devised to his son, his son is entitled to receive the licensing payments from the manufacturer for the remaining term of the licensing agreement. Answer choice A is incorrect because the corporation's right to receive the payments pursuant to the licensing agreement terminated upon the inventor's death. Answer choice B is incorrect. Although the inventor's daughter is entitled to his corporate stock, the inventor's property, including the right to receive payments under the licensing agreement as well as the patent itself, was devised to the inventor's son. Answer choice D is incorrect because the inventor's death does not terminate the manufacturer's right to continue to use the inventor's patent for another five years and similarly does not terminate the manufacturer's obligation to make payments for that use pursuant to the licensing agreement for that time period as well.

A widow offered to sell her small business, together with all of the business's assets, to a non-profit organization. The organization accepted, and on June 1, they signed and executed a contract providing for the sale of the business for $25,000 at the end of the month. When the organization's agent signed the contract, she orally informed the widow that the organization's duty to purchase the business was conditioned on obtaining approval from a local zoning board to convert the business's primary office into an affordable healthcare clinic. At the end of the month, the widow refused to honor the contract because the organization neglected to request the necessary approval from the zoning board. The organization sued the widow for breach of contract. The organization presented clear evidence that they had the necessary funds to perform on the contract at the end of the month, and that the zoning board would have routinely approved the organization's plans for the office. Is the organization likely to prevail in its action against the widow? (a) No, because the express condition of the zoning board's approval had not occurred by the end of the month. (b) No, because the organization's failure to seek approval from the zoning board was a repudiation of the contract. (c) Yes, because the condition of approval by the zoning board can and has been waived by the organization. (d) Yes, because the condition of approval by the zoning board was not included in the written contract.

Answer choice C is correct. A party whose duty is subject to a condition can waive the condition, either by words or by conduct. Here, the organization has waived the condition by seeking to enforce the contract without the approval of the zoning board. Due to this waiver, answer choice A is incorrect. Answer choice B is incorrect. The doctrine of anticipatory repudiation is applicable when a promisor repudiates a promise before the time for performance is due. The repudiation must be clear and unequivocal and may be by acts or words. Here, the organization's failure to seek routine approval from the zoning board is not a clear and unequivocal repudiation of the contract; it is merely a waiver of the condition. Answer choice D is incorrect because parol evidence may be admitted to prove a condition precedent to the existence of the contract. Therefore, this condition was admissible and enforceable even if it was not included in the written contract.

On January 5, a buyer and seller contracted for the delivery of 100 widgets if they could be delivered by February 20. The agreement was made in a writing signed by both parties and provided that the buyer would pay the contract price of $1,000 upon delivery. On February 3, the buyer and seller orally agreed to postpone delivery until March 1. However, when the widgets arrived on March 1, the buyer refused to accept or pay for the widgets. If the seller sues the buyer for breach of contract, who is most likely to succeed in the action? (a) The buyer, because any modification of the parties' contract must satisfy the Statute of Frauds. (b)The buyer, because the agreement on February 3 was not supported by consideration. (c) The seller, because the oral agreement on February 3 waived the February 20 delivery date. (d)The seller, because the contract modification on February 3 was immediately binding on both parties.

Answer choice C is correct. A party whose duty is subject to the condition can waive the condition, either by words or by conduct. A party who indicates that a condition will not be enforced may be estopped from using that condition as a defense if the other party reasonably relied on the party's words or conduct that the condition had been waived. Here, the buyer and the seller agreed to postpone delivery (i.e., the buyer waived the condition that the widgets be delivered by February 20). The seller relied on that waiver (as evidenced by the fact that he did not deliver the widgets until March 1), and it would be unjust for the buyer to now claim that the contract was breached. Therefore, the seller will prevail because the February 20 delivery date was waived. Answer choice A is incorrect. A promise that the promisor should reasonably expect to induce action or forbearance on the part of the promisee or a third person and that does induce the action or forbearance is enforceable notwithstanding the Statute of Frauds if injustice can be avoided only by enforcement of the promise. Here, the buyer's indication that delivery would be accepted on March 1 prevents the buyer from using the seller's failure to deliver by the February 20 delivery deadline to justify the buyer's failure to perform under the contract. Answer choice B is incorrect because consideration is not needed to modify a contract under the UCC. Answer choice D is incorrect because contract modification (i.e., the waiver) was not technically enforceable until there had been detrimental reliance. The requirements of the Statute of Frauds must be satisfied if the contract as modified is within its provisions and as a contract for the sale of goods over $500, such a modification would require a writing. However, once the buyer let the original delivery date pass, the buyer was estopped from using the passing of that deadline as justification for failing to perform.

A car salesman on the lot at a car dealership specializing in expensive high-end vehicles called out with a megaphone, "Free set of wheels to the next person who buys a car from me!" He was surrounded by stacks of tires and a display case with informational pamphlets advertising all-weather tires. A customer, believing that the car salesman was offering a two-for-one deal on the expensive vehicles sold on the lot, immediately approached the salesman and purchased a new vehicle for his son, believing that he would have a nice new car to give to his daughter as well. The salesman then asked if the customer would like his free set of tires installed on the new car. The customer responded that he did not want the tires, and that the salesman was contractually obligated to give him a second car. The salesman laughed and said that he was never offering a free car, but would cancel the sale of the first car as a consolation. Is the salesman obligated to give the customer a free car? (a) Yes, because the customer reasonably believed the salesman was making a valid offer. (b) Yes, because a reasonable person would have believed the salesman was offering a free car. (c) No, because no reasonable person would believe that the salesman was offering a free car. (d) No, because the salesman was willing to cancel the sale.

Answer choice C is correct. A statement is an offer only if the person to whom it is communicated could reasonably interpret it as an offer. The primary test of whether a communication is an offer is based on the objective theory of contracts; i.e., whether an individual receiving the communication would believe that he could enter into an enforceable agreement by satisfying the condition. Here, although the salesman made a specific offer with a specific method to accept the offer, no reasonable person would believe that the dealer in this scenario intended to provide buyers with an additional car of the same value. Answer choice A is incorrect because the customer's subjective belief does not determine whether a contract exists. Answer choice B is incorrect because no reasonable person would believe that the offer and acceptance reflected a free car in exchange for the purchase of another car, though a reasonable person may believe that the dealer intended to provide buyers with a free set of tires. Answer choice D is incorrect because the salesman cannot generally invalidate the offer and acceptance in order to avoid performance, if the customer wanted to go through with the sale.

A seller offered his antique sports car to a buyer for $10,000, with the option expiring in forty-eight hours. The morning after the parties discussed the offer on the phone, the seller offered to drive the car to the buyer's home so the buyer could inspect the car, see how it looked in his own driveway, and make a final decision on whether he wanted to buy the car. On the way to the buyer's house, the seller was in an accident that totaled the car. Before the seller called to tell the buyer about the accident, the buyer called him to accept the offer. Once he learned of accident, the buyer demanded that the seller provide him with a similar antique sports car. Is the seller required to provide the buyer with a similar antique sports car? (a) Yes, because driving the first car to the buyer's home constituted partial performance. (b) Yes, because the first car would not have been destroyed but for the drive to show it to the buyer. (c) No, because the first car was destroyed. (d) No, because the buyer did not accept the offer during the original phone call.

Answer choice C is correct. An offer involving subject matter that is destroyed is terminated. Here, when the car was destroyed in the accident, the offer terminated as well. Therefore, the buyer could not have accepted it or demanded a similar car. Answer choice A is incorrect because performance would not apply when the offer was only an option. Further, the destruction of the car terminates the option. Answer choice B is incorrect because it is not an accurate statement of the law; the manner in which the car was destroyed is irrelevant here. Answer choice D is incorrect because the fact that the buyer did not accept the offer during the phone call would not affect his ability to do so, had the car not been destroyed.

At a local market, a buyer offered to purchase a large, framed mirror from an artist for $1,000. The artist stated that he wanted to wait to see how many people went through the market that day before he decided on whether he would accept the offer. The next morning, the buyer returned to the market only to learn that the mirror had been dropped and broken. Because the frame of the mirror was still in good condition, the buyer wrote a check for $500 and gave it to the artist without further remark. The artist loaded the frame into the buyer's vehicle and demanded the remaining $500 offered the day before. Is the buyer liable for the remaining $500? (a) Yes, because the original offer was still valid. (b) Yes, because the artist thought that he had accepted the original offer. (c) No, because the original offer terminated. (d) No, because the buyer reasonably believed the original offer terminated.

Answer choice C is correct. An offer involving subject matter that is destroyed is terminated. In this case, when the buyer originally offered $1,000, the subject of the offer was a large, framed mirror. When the mirror broke, the buyer's offer terminated. The artist could no longer accept it. Answer choice A is incorrect because the original offer terminated upon the destruction of the mirror. Answer choice B is incorrect because the offer terminated upon the destruction of the mirror even if the artist believed otherwise. Further, the artist's subjective belief is irrelevant, as contract law takes an objective approach in determining intent. Answer choice D is incorrect because regardless of the buyer's belief, the offer had terminated.

An honest dispute develops between a condominium owner and a plumber over whether plumbing installed in the kitchen and bathrooms of the condominium satisfied contractual specifications. If the plumbing meets those specifications, the condominium owner owes the plumber $15,000 under the terms of the contract. The condominium owner offers to pay the plumber $10,000 in satisfaction of the owner's contractual obligations, if the plumber replaces the plumbing in the kitchen with another grade of pipe. The plumber accepts the condominium owner's offer. After the plumber replaces the kitchen plumbing, the condominium owner refuses to pay the plumber. In a breach of contract action brought by the plumber, the fact-finder determines that the plumbing originally installed by the plumber did satisfy the contract specifications. The fact-finder also determines that the plumber and the condominium owner entered into a substitute agreement for which the owner failed to deliver the required performance. What is maximum amount that the plumber can seek in damages from the condominium owner? (a) $25,000 (b) $15,000 (c) $10,000 (d) Nothing

Answer choice C is correct. Because the plumber and the condominium owner entered into a substitute agreement rather than an accord, the substitute agreement completely replaces the original contract. Consequently, the plumber may only seek damages of $10,000, the amount that the condominium owner promised to pay under that agreement. Had the second agreement between condominium owner and the plumber been an accord, the plumber could have sought damages upon the condominium owner's breach of the accord under either the original contract (i.e., $15,000) or the accord (i.e., $10,000), but not both. For this reason, answer choices A, B, and D are incorrect. NOTE: Be careful with this question. Even though the facts tend to read like an accord, we know that this is instead a substitute agreement because the question states that the fact-finder determined this to be so.

A library contacted a local artist expressing an interest in purchasing a particular one of the artist's sculptures for display at the library. The library's agent and the artist executed a written contract signed by both parties providing that the library would purchase the sculpture for $1,000 due upon delivery of the sculpture to the library. Just before they signed the contract, the agent told the artist, "Plan on delivering the sculpture in 10 days, but please remember that the library's obligation to purchase the sculpture will be conditioned on the approval of the chairperson of the Artistic Patronage Council, as they will be providing the library with the funds for this sale." The chairperson of the Artistic Patronage Council orally approved the sale the next day. However, ten days after the contract was executed, the artist decided he did not want to sell the sculpture. If the library sues the artist for breach of contract, is the library likely to prevail? (a) No, because the library's agent made an illusory promise. (b) No, because there was no mutuality of remedy when the contract was executed. (c) Yes, because the agreement was supported by good consideration even though it was conditioned on an uncertain event. (d) Yes, because the artist waived any lack of consideration by signing the contract.

Answer choice C is correct. Consideration can take the form of a return promise to do something, a return promise to refrain from doing something legally permitted, the actual performance of some act, or refraining from doing some act. Consideration does not fail simply because a party's performance is conditioned on an uncertain event, such as approval by a third party. Therefore, the library can succeed in a breach of contract action against the artist. Answer choice A is incorrect. An illusory promise is one that essentially pledges nothing because it is vague or because the promisor can choose whether or not to honor it. A promise that is based on the occurrence of a condition within the control of the promisor may be illusory, but there is no evidence on the facts that the approval of the chairperson of the Artistic Patronage Council is within the control of either the library or the library's agent. Therefore, this does not appear to be an illusory promise. Answer choice B is incorrect because mutuality of remedy is not a requirement to form a valid contract. Answer choice D is incorrect. The basic concept of consideration is that there must be something of substance given in exchange for the promise that is to be enforced. The court generally will not review the adequacy of the consideration supporting a contract, but the requirement for some form of consideration cannot be waived.

A supervisor and an employee in neighboring cubicles had been chatting about sports. Ten minutes after their sports discussion had ended, the supervisor offered to sell the employee tickets to an upcoming baseball game for $500, even though he saw the employee had placed earphones into his ears and begun working again. The employee, realizing that he missed something his supervisor said, nervously responded, "Sure, boss." The day of the game, the supervisor told the employee that he would accept cash or check. When the employee indicated he had no idea what the supervisor was referencing, the supervisor angrily restated the terms of the discussion. Upon hearing the facts, the employee, who was short on funds, refused to pay the supervisor. Has a contract between the supervisor and employee been formed? (a) Yes, because the supervisor reasonably believed the employee accepted the offer. (b) Yes, because the employee accepted the terms of the contract. (c) No, because a reasonable person would not believe the parties had entered into a contract. (d) No, because the employee did not intend to accept the contract.

Answer choice C is correct. Contract formation is determined by an objective theory: whether reasonable people in the parties' position, given the facts and circumstances, would conclude that a contract had been made based on objective manifestations of intent (regardless of actual subjective intent). In considering the circumstances surrounding the statement, it would not be reasonable to assume the parties entered a contract when the employee never even heard the offer. Answer choice A is incorrect because the subjective belief of a party is not the relevant consideration; instead, the court would consider the existence of the contract based on an objective theory. Answer choice B is incorrect because a reasonable person would not believe the employee intended to accept the terms of the offer, as he never even heard the offer. Answer choice D is incorrect because the intent to contract is generally determined by an objective theory, rather than a subjective one. While the earphones would indicate that the employee did not intend to be bound by the contract, it would be an objective standard that would determine the validity of the contract. While both answer choices C and D would likely arrive at the same conclusion—that there is no valid contract—the correct answer choice reflects that this would be based on an objective standard rather than a subjective one.

A public high school with over 800 students hosted a walkathon to raise money to purchase new textbooks for their students. The school administration solicited various businesses to sponsor the walkathon, including the CEO of a large food services company. The CEO wanted the administration to hire his food services company to provide lunch in the school cafeteria when the school's contract with their current food services provider expired in one month. Hoping to catch the administration's attention, the CEO pledged that his company would match all of the sponsors' pledges. Because he was a business sponsor, the CEO was required to fill out and sign a writing stating this promise. The walkathon was a success, and the total amount raised, excluding the CEO's pledge, was $42,000. After collecting the pledge money from all of the sponsors except for the CEO, the school was able to replace the old textbooks at a total cost of $42,000. The school planned on using the CEO's pledge to purchase supplemental learning materials to further advance the education of its students. Prior to being asked for his matching pledge, the CEO learned that the contract of the school's current food services provider had been renewed for another five years. The CEO subsequently repudiated his promise to match all of the sponsors' pledges. If the school sues the CEO for $42,000, will it succeed? (a) No, because the school did not actually rely to its detriment on the CEO's pledge when purchasing the textbooks. (b) No, because there is no substantial injustice since the school was able to pay for all of the textbooks without the CEO's pledge. (c) Yes, because the court will presume that the school detrimentally relied upon the CEO's written promise. (d) Yes, because the CEO acted in bad faith when he repudiated his promise to match the pledges of the other sponsors.

Answer choice C is correct. Courts often apply the doctrine of promissory estoppel to enforce charitable subscriptions. In some cases, they presume that the charity detrimentally relied on the promised contribution. A charitable subscription (i.e., a written promise) is enforceable under the doctrine of promissory estoppel without proof that the charity relied on the promise. Here, there is no proof in the facts that the school relied upon the CEO's promise, but that promise is a charitable subscription, the court will presume that the school detrimentally relied on the CEO's written promise. For this reason, answer choice A is incorrect. Answer choice B is incorrect because the CEO should have reasonably expected that the school would rely upon his pledge, and that there would be injustice if he did not pay the $42,000. Answer choice D is incorrect. Whether the CEO acted in bad faith is not the issue. Rather, the central issue here is that a court will enforce the written promise under the doctrine of promissory estoppel without proof that the school relied on the CEO's pledge.

A butcher and a seller entered into a written contract for the purchase and sale of a building to be used as a butchery. The closing was scheduled for June 1. On May 25, the seller was notified by the city that the building, which had previously been used as a butchery, had a number of significant city code violations. The seller immediately contracted with an electrician and others to correct the issues. Despite his best efforts, the seller realized the building would not be brought up to code until at least June 10. The seller promptly sent written notification of this issue to the butcher and informed him that he would be unable to take possession of the building until June 10. Based on his agreement with the seller, the butcher had declined to renew his lease at his current location and was forced to remove his equipment and inventory from his current location by the end of May. Between June 1 and June 10, to prevent spoilage of his inventory due to the delay, the butcher had to rent space to store his equipment and inventory. He moved his freezer to the rented space to store his meat at a cost of $200 per day, plus the cost of electricity to run the freezer. On June 10, the building was up to code. The butcher paid the seller the agreed-upon purchase price of $300,000 and took possession of the property. What damages, if any, may the butcher recover from the seller? (a) Nothing, because the seller acted in good faith. (b) Nothing, because the contract did not contain a "time is of the essence" clause. (c) $2,000, the rental cost he had to pay from June 1 to June 10. (d) $2,000 plus the cost for electricity to run the freezer.

Answer choice C is correct. Expectation (benefit-of-the-bargain) damages are intended to put the nonbreaching party in the same position as if the contract had been performed as agreed. Expectation damages must be calculated with reasonable certainty. If the seller breached but acted in good faith, then damages are limited to the buyer's out-of-pocket expenses. In this case, the butcher is entitled to be compensated for the loss that resulted from the seller's (minor) breach, which is the $2,000 in rental costs that the butcher had to pay from June 1 to June 10. Answer choice A is incorrect. The seller remains liable for breach even though he acted in good faith. However, the seller's good faith limits the butcher's damages to out-of-pocket expenses only. Answer choice B is incorrect because the absence of a "time is of the essence" clause in the contract does not prevent the butcher from recovering his out-of-pocket expenses from the seller. Answer choice D is incorrect because the butcher would have had to pay the same cost for electricity to run the freezer in the building he purchased if he had been able to move in.

A homeowner entered into a written contract with a contractor to construct an elaborate tree house among the large trees located in the homeowner's backyard. After commencing construction of the tree house, the contractor discovered that one of the trees intended to be used as support for the tree house had a relatively common fungal infection in its core that would cause the strength of the tree's branches to falter if left untreated. Neither the homeowner nor the contractor had knowledge of the fungal infection when they entered into the contract, but the contractor knew that such infections were common in the area and did not request an inspection of the trees before entering the contract. The contractor also knew that treatment was available at a high cost, but even after treatment, he would need to create additional heavy-load bearing supports for the tree at a substantial cost. When the contractor informed the homeowner that he would not perform under the contract unless the homeowner provided at least 75% of the additional costs needed to make the structure safe, the homeowner refused to pay the additional amount. The homeowner then sued the contractor for breach of contract. What is the likely result? (a) The contractor wins, because his performance was discharged due to impracticability. (b) The contractor wins, because neither party was aware of the fungal infection. (c) The homeowner wins, because the contractor assumed the risk of the fungal infection. (d) The homeowner wins, because the fungal infection did not render performance impossible.

Answer choice C is correct. For the defense of impracticability to be available, an unforeseeable event must occur and the nonoccurrence of that event must have been a basic assumption on which the contract was made. If a party assumes the risk of an event happening that makes performance impracticable, then the defense of impracticability will not apply. In this case, the contractor discovered a relatively common fungal infection in the core of one of the trees. Because the contractor knew that the fungal infection was relatively common in the area, the nonoccurrence of the infection was likely not a basic assumption on which the contract was made. Therefore, the contractor assumed the risk of encountering the fungal infection in the tree when he entered into the contract with the homeowner. Answer choice A is incorrect because the defense of impracticability cannot be raised by the contractor in this case due to the contractor's assumption of this risk. Answer choice B is incorrect. The fact that neither party was aware of the fungal infection does not excuse the contractor's performance. Although mutual mistake can make a contract voidable, a party may bear the risk of a mistake when he is aware, at the time of the contract, that he has only limited knowledge of the facts to which the mistake relates, and he accepts his limited knowledge as sufficient. Here, the contractor entered the agreement knowing that these infections were common and that the trees had not been inspected. Answer choice D is incorrect. The defense of impracticability does not require performance to be rendered impossible. Rather, the reason the defenses of impracticability and mistake do not apply here is that the contractor assumed the risk.

Prior to her death, a celebrity commissioned an artist to paint a portrait of her. The celebrity hired this particular artist because he only painted using an old-fashioned and rarely used style that required two months of daily appointments during which the subject would sit for the painting over a few hours each day. The contract between the parties specified that this live-model method be used, and that the celebrity would deliver increasing payments throughout the process, with the first payment occurring after two weeks of painting. One week into the process, after the painting had begun, the celebrity died. Her family demanded that the artist continue with the painting, using photographs as a substitute for the daily sessions. Is the artist required to complete a painting of the celebrity? (a) Yes, because he can complete it by relying on pictures of the celebrity. (b) Yes, because he had begun painting the celebrity. (c) No, because the celebrity died after only one week. (d) No, because no payment had yet occurred.

Answer choice C is correct. If an offer has actually been accepted, death of the offeror does not automatically terminate the contract. The contract may still be enforceable unless there is some reason, such as impracticability, that warrants a discharge of the obligation. Here, the woman has died, so she cannot sit for the painting for the two months. Once the woman died, it became impracticable (and impossible even) to complete the painting under the terms of the contract. Therefore, the artist is not required to complete a painting of the celebrity. Answer choice A is incorrect because the contract did not specify the use of another method, and the facts state that the artist only uses the old-fashioned and rarely used style of painting. The family could not alter the terms of the contract after the celebrity's death. Answer choice B is incorrect because although the artist's performance reflects acceptance of the offer, the obligation is discharged upon the celebrity's death. Answer choice D is incorrect. Although payment had not yet occurred, that is not a valid reason to not enforce an otherwise enforceable contract. The first payment was not yet due under the contract.

A homeowner entered into oral contracts with both a painter and a landscaper to perform services at his home. The landscaper was the first to begin the services, and shortly after he began to work, he realized that the type of soil in the homeowner's yard would cause the projected cost of the work to increase dramatically. After the homeowner realized how high the cost of the landscaping services were going to be, he called the painter to tell her that he could not go through with their contract at that time. The painter stated that she had already purchased paint and brushes for the job, as well as painted glass to create a small, artistic mosaic on a back corner of the house as a sort of signature expression she planned to begin using. She had also paid for a temporary city permit to park her utility van on the residential street where the homeowner lived. Which of the following would not be a possible liability for the homeowner? (a) The contract price minus the market cost of performance (b) Cost of the paint and brushes (c) Cost of the glass (d) Cost of the permit

Answer choice C is correct. Non-breaching parties may often choose between several different types of damages, including reliance damages (those the plaintiff reasonably incurred in reliance of the contract) and expectation damages (which put the nonbreaching party in the position that she would have been in if not for the breach). To calculate expectation damages, compare the value of performance without the breach (what was promised) with the value of the performance with the breach (what was received). Expectation damages must be foreseeable, so the glass tiles for the mosaic would not fall under the umbrella of expectation damages, as there was no way for the homeowner to know she was using them. Incidental damages may also be recovered for commercially reasonable expenses incurred as a result of the other party's breach, and the cost of the permit would fall under this category. Note, however, that parties cannot recover both reliance and expectation damages, so the painter would not be able to recover both the contract price and the price of supplies. Answer choice A is incorrect because it reflects expectation damages, which is one of the possible remedies for the non-breaching party. Answer choice B is incorrect because the cost of supplies could constitute reliance damages. Answer choice D is incorrect because the painter incurred the cost in anticipation of completing work at the homeowner's residence, and she would be entitled to reimbursement for that expense incurred in reliance on the contract.

Upon the completion of an interview, an attorney offered an assistant $3,000 each month to perform administrative tasks at his office; he emailed her an offer confirming these terms. She responded via email, stating that she accepted the offer but inquired as to whether she would have to work in the office or could work remotely, expressing that the option to work from home was essential to her. The parties had discussed in the interview that the assistant would likely work in the office but that the attorney would be open to her working remotely at some point. The emailed offer did not address this issue. The attorney did not respond that day but went out and bought the assistant a laptop conforming to her preferences and many office supplies that she had requested. The next day, the attorney emailed her to confirm that he might allow her to work remotely after she demonstrated success within an office environment. She emailed him back that she no longer wished to work with him because she preferred to work from home right away. The attorney threatened to sue her for breach of contract. Under common-law principles, have the attorney and assistant entered into an enforceable contract? (a) Yes, because the assistant emailed him back to accept the offer. (b) Yes, because the attorney's offer did not propose that the assistant could work from home. (c) No, because the assistant suggested additional terms in her response. (d) No, because the assistant had a misunderstanding as to the terms of the contract.

Answer choice C is correct. The acceptance must mirror the terms of the offer. Any change to the terms of the offer, or the addition of another term not found in the offer, acts as a rejection of the original offer and as a new counteroffer. Mere suggestions or inquiries, including requests for clarification or statements of intent, made in a response by the offeree do not constitute a counteroffer. However, the addition of a term that is essential to the party likely indicates that the response was a counteroffer, not a mere inquiry. In this case, the assistant's acceptance of the attorney's offer was conditional upon her being able to work from home. Accordingly, it was not really an acceptance, it was a rejection and counteroffer. Accordingly, answer choice A is incorrect. Answer choice B is incorrect because it represents a fact that supports that the assistant's question was merely a means for clarification. However, it is not in and of itself a reason for enforcing the contract. Answer choice D is incorrect because she did not misunderstand the terms of the contract. Rather, she proposed an additional term.

On March 1, a company contracted with a singer for the singer to perform for the company picnic on May 1 for a fee of $10,000. On March 17, the singer informed the company that she signed a contract to film a movie. She suggested that the company hire another singer to take her place at the picnic. On April 1, the company hired the recommended replacement singer to perform at its picnic for $15,000. On April 25, the original singer informed the company that she has decided not to take the movie deal and will be available to perform on May 1. Even though the original singer arrived at the picnic on May 1 ready to sing, the company let the replacement singer perform. The company refused to pay $10,000 to the original singer. Is the company likely to prevail in a breach of contract claim against the original singer? (a) No, because the company prevented the original singer from fulfilling her contractual obligation by refusing to let her perform on May 1. (b) No, because the original singer retracted her repudiation before the scheduled performance. (c) Yes, because the company hired the replacement singer as a substitute for the original singer before she retracted her repudiation. (d) Yes, because the replacement singer's consent to the delegation of the original singer's duties did not create a novation.

Answer choice C is correct. The company's contract with the replacement singer as a substitute for the original singer's services constituted a material change in position and thus terminated the original singer's ability to retract her repudiation. Answer choice A is incorrect because acting in reasonable reliance on an unequivocal repudiation does not violate the doctrine of prevention. Answer choice B is incorrect because the company's material change in position happened before the original singer tried to retract her repudiation. Answer choice D is incorrect because, even if this personal service contract could be delegated, the original singer's suggestion that the company hire the replacement singer does not constitute an effective delegation of her contractual duties.

A company leased office space in a downtown building and subsequently entered into a written contract with a supplier to purchase furniture for the office. Among the provisions in the contract was the following: "This document contains the entire and final agreement of the parties. It supersedes any prior agreements, understandings, or negotiations, whether written or oral." A dispute subsequently arose over the tables and desks delivered by the supplier. The contract called for "cherry tables and desks" of designated designs. The company contended that the word "cherry" indicated the type of wood that the tables and desks were made of. The supplier, having delivered tables and desks made of a less expensive wood and finished with a cherry veneer, asserted that the use of the word "cherry" referred to the appearance of these items and did not require that the furniture be made solely of cherry wood. In the litigation of this dispute, the company sought to introduce a statement made by the supplier during negotiations that the tables and desks were of "solid-wood construction." In determining whether the contract was a total integration of the agreement between the company and the supplier, which of the following rules should the court apply? (a) The four-corners rule (b) The "naturally omitted" rule (c) The "certainly included" rule (d) The plain-meaning rule

Answer choice C is correct. The contract is for the sale of particular pieces of furniture, which are goods, and therefore it is governed by the Uniform Commercial Code (UCC). Under the UCC, a court, in determining whether a contract constitutes a total integration of the parties' agreement, should generally treat a written agreement as only a partial integration of the parties' agreement unless the court can conclude that the parties' "certainly would" have included the term in the written agreement. Answer choice A is incorrect. Under the common-law four-corners rule, a court is required to look only to the wording of the document itself (i.e., within its "four corners") in determining whether the parties intended the document to be a total integration of their agreement. However, this rule does not apply to contracts, such as this one, that are governed by the UCC. Answer choice B is incorrect. The "naturally omitted" rule of the Restatement (Second) of Contracts requires a court to consider whether an extrinsic term of an agreement would "naturally be omitted" from a writing in determining whether the parties intended the document to be a total integration of their agreement. However, this rule does not apply to contracts, such as this one, that are governed by the UCC. Answer choice D is incorrect because the plain-meaning rule, which provides that the objective definitions of contract terms control the meaning of the contract, may be applicable in ascertaining whether to admit extrinsic evidence despite the written agreement being a total integration. However, this rule is not applicable in determining whether, in the first instance, the written agreement is a total integration.

A couple, who wanted to open a pet grooming and supply store, contracted with a developer to lease space in a small strip mall that the developer was constructing. The lease was to begin on July 1, but on June 20, the developer informed the couple that the mall would not be finished nor would the space be available until August 1. The developer indicated that the first month's rent would be waived, but that, because the lease did not contain a liquidated-damages clause, he was not responsible for any damages attributable to the delay. As a consequence of the delay, the couple incurred storage costs and additional advertising expenses of $3,000, and they estimated in good faith that they lost $10,000 in sales. Which of the following amounts is the couple entitled to recover from the developer for the delay? (a) Nothing, because the lease did not contain a liquidated-damages clause. (b) Nothing, because, as a new business, damages are too speculative. (c) $3,000, the amount incurred as a consequence of the delay. (d) $10,000, the good-faith estimate of lost sales.

Answer choice C is correct. The couple incurred $3,000 in storage costs and advertising expenses as a consequence of the delay. They are entitled to recover this amount as reliance damages that they have suffered because they reasonably relied on the promise in the lease that they could move into the retail space on July 1. Answer choice A is incorrect. Although the lease did not contain a liquidated-damages clause, which may have given them the ability to recover fixed damages as a consequence of the delay, the couple can nevertheless recover their reliance damages, which they can establish with certainty. Answer choice B is incorrect. Although a new business generally cannot recover damages based on lost profits because such damages are too speculative even if estimated in good faith, the couple can recover their damages stemming from their reliance on the unfulfilled promise in the lease that they could occupy the retail space in the mall on July 1. Answer choice D is incorrect. The couple cannot recover expectation damages for their lost profits because such damages are too speculative. Even if they could recover their lost profits, the amount of their sales would have to be offset by the expenses that they would have incurred to generate such sales, such as the cost of the goods sold, in order to arrive at their profit for the month of July. Note: The couple could instead recover incidental damages, which happen to be the same amount as reliance damages in this case. Incidental damages may be awarded to the nonbreaching party as compensation for commercially reasonable expenses incurred as a result of the other party's breach. Here, the storage costs and advertising expenses are commercially reasonable expenses which were incurred as a result of the breach.

A sister convinced her brother that they should open a small coffee shop. Their friend, a guitarist, suggested bringing his band to play live music and attract customers. He did not request any payment, saying the publicity would be good for the band. The siblings agreed, and the band started playing at the shop weekly. The coffee shop became a success, in no small part due to the band's performances. When a businessperson offered to buy the coffee shop from the siblings, they orally agreed to each pay $10,000 out of their share of the sale proceeds to the guitarist for his help in making the shop popular. The sister told the guitarist about their agreement. He was so delighted with it that he put a down payment on a new car. By the time the sale of the business was finalized, the brother had encountered financial difficulties. After the sale, the siblings signed a written contract stating that the sister would pay the guitarist $10,000 and her brother would pay him $5,000. If, after the sale, the brother pays the guitarist only $5,000, will he have a valid basis for action against the brother for another $5,000? (a) No, because the guitarist was bound by the written modification of the contract made by the siblings. (b) No, because the guitarist was only a donee beneficiary of the oral contract between the siblings. (c) Yes, because the guitarist's reliance on the promised payment prevented the siblings from changing the obligations of their oral contract. (d) Yes, because the oral promise to pay $10,000 to the guitarist was made binding by the guitarist's valuable and uncompensated contributions to the business.

Answer choice C is correct. The guitarist was an intended beneficiary of the siblings' oral agreement to each pay him $10,000. His rights under that contract vested when he made a down payment on a car in reliance on the agreement. The power of the siblings to modify their duties by subsequent agreement between themselves was terminated when the guitarist detrimentally relied on their promised performance. Answer choice A is incorrect because, although parties who agree to confer a benefit on an intended beneficiary generally have the right to modify their duties by subsequent agreement, this right ceases to exist when the intended beneficiary's rights vest. Answer choice B is incorrect because, while the guitarist is the beneficiary of a gift, he is clearly an intended beneficiary of the siblings' oral agreement. As such, once his rights in the contract have vested, which they have in this instance, the parties to the agreement cannot modify the agreement without his consent. Answer choice D is incorrect because a promise made based on a performance that has already been rendered is typically not adequate consideration because that performance could not have been bargained for, nor was it undertaken in reliance upon the promise.

At lunch with two co-workers, the owner of a bike offered to sell it to his manager for $100. His manager replied, "That ancient thing? I'll give you $50 for it." The owner's assistant then stated, "I'll buy it for $75." Immediately after the owner responded, "OK," the manager said, "Wait a minute. How old is it?" When the owner replied, "Two years old," the manager said, "OK, I'll buy it for $100." Which of the following statements is true? (a) The owner may sell the bike to his manager for $100. (b) The owner must sell the bike to his manager for $100. (c) The owner must sell the bike to his assistant for $75. (d) The owner may accept his manager's offer of $50.

Answer choice C is correct. The owner made an offer to sell the bike to his manager for $100. Since the owner's offer was not directed to his assistant, the assistant's statement, "I'll buy it for $75" constituted an offer, which the owner accepted. Consequently, the owner must sell the bike to his assistant for $75. Answer choice A is incorrect because the owner accepted his assistant's offer prior to the manager's offer to buy the bike for $100. Therefore, the owner is bound by his contract with the assistant and cannot sell the bike to the manager. Answer choice B is incorrect because the manager rejected the owner's offer when he made the $50 counteroffer. The manager could not subsequently accept the owner's $100 offer because it had been terminated by that rejection. Answer choice D is incorrect because, although the owner could have accepted his manager's counteroffer of $50 for the bike, the owner's acceptance of his assistant's offer of $75 and the manager's awareness of this acceptance operated as a rejection of the manager's counteroffer. Therefore, the owner may not accept the manager's offer of $50.

A maker of hand-woven rugs contracted with a supplier to provide yarn made from sheep's wool. The written contract specified that, for four years, the supplier would provide the rug maker with 2,000 spools of yarn made from 100% sheep's wool per month, at $10 per spool, for a total of $20,000. Two years into the contract, the supplier sent 2,000 spools of yarn to the rug maker made from 90% sheep's wool and 10% synthetic fiber. The rug maker sent a check to the supplier for $15,000 for the shipment, and added a clear note on the check stating that the payment was in full for the shipment, but was $5,000 less due to the synthetic fiber in the yarn. The supplier promptly deposited the check, and then four months later filed suit against the rug maker for the remaining $5,000. The supplier has submitted evidence of the written contract, and the rug maker has submitted evidence of the deposited check. What is the rug maker's best defense in this situation? (a) The rug maker's and supplier's good faith dispute over the yarn composition suspended the rug maker's obligation to pay the remaining $5,000. (b) The act of knowingly depositing the check for $15,000 by the supplier was a novation that relieved the rug maker from any further liability. (c)The supplier deposited the check for $5,000 less than the contract price, thereby discharging the rug maker of any further duty to pay the remaining amount for that month's shipment. (d) By depositing the check, the supplier was estopped from claiming that the rug maker owed him an additional $5,000.

Answer choice C is correct. This is considered an "accord and satisfaction," which discharges both the original contract and the accord contract. Under an accord agreement, a party to a contract agrees to accept a performance from the other party that differs from the performance that was promised in the existing contract, in satisfaction of the other party's existing duty. Generally, consideration is required for an accord to be valid. By compromising, each party surrenders its respective claim as to how much is owed. If a claim is subject to dispute, it can be discharged if the person against whom the claim is asserted in good faith tenders a negotiable instrument (e.g., a check) that (i) is accompanied by a conspicuous statement indicating that the instrument was tendered as full satisfaction of the claim (e.g., "payment in full"), and (ii) the claimant obtains payment of the instrument. Here, the rug maker compromised by accepting the 90% wool yarn, and the supplier compromised by accepting $15,000 rather than $20,000. Thus, there was an accord and satisfaction and the rug maker is not liable for the remaining $5,000. Answer choice A is incorrect, because the good-faith dispute did not suspend the duty of the rug maker to pay for the yarn supplied. In the context of an accord and satisfaction, it is only the existence of an accord agreement that suspends the original duty of a party. Answer choice B is incorrect, as this is not a novation. A novation is the substitution of a new contract for an old one when the original obligor is released from his promises under the original agreement and a new obligor becomes liable. Here, there are only two original parties-the rug maker and the supplier. Answer choice D is incorrect because estoppel requires not only an assertion by a party but also justifiable reliance on the assertion by the party to whom the assertion is made, to that party's detriment. Here, even assuming that the supplier's deposit of the check constituted an assertion, there is no evidence that the rug maker has relied on this assertion to his detriment.

An organic produce supplier sent her produce catalog to a local restaurant on April 15. The catalog came with a signed letter stating: "I will supply you with as many of the items in the enclosed catalog as you order before August 1 of this year, and I assure you that this offer and the prices provided in the catalog will remain firm until August." The supplier received no reply. In June, the supplier's tomato crop was infested with a fungus that decimated her harvest. She was left with half the tomatoes she had expected to harvest. On June 15, the supplier sent the restaurant a signed letter stating that the price for the tomato crop was now twice the price listed in the catalog. On July 1, the restaurant sent the supplier an order for tomatoes. However, the restaurant demanded the tomatoes at the price listed in the catalog. Has a contract been formed for the sale of the tomatoes to the restaurant at the catalog price? (a) No, because the destruction of the tomatoes supports a defense of impracticability. (b) No, because the restaurant offered no consideration to make the original offer irrevocable. (c) Yes, because the original offer was irrevocable on June 15 and on July 1. (d) Yes, because the restaurant exercised its power of acceptance within a reasonable time.

Answer choice C is correct. Under the UCC, an offer to buy or sell goods is irrevocable if the offeror is a merchant, there is an assurance that the offer is to remain open, and the assurance is contained in a signed writing from the offeror. The irrevocability of a firm offer cannot exceed three months unless the offeree gives consideration to validate it beyond the three-month period. Here, the original offer meets these requirements, and both the date the supplier tried to change the price (June 15) and the date the restaurant accepted the offer (July 1) fall within the three-month period of irrevocability. Therefore, the offer was still open and irrevocable when the restaurant accepted it on July 1. Answer choice A is incorrect because the question asks whether a contract has been formed, and the answer is yes under the UCC firm offer rule. Impracticability would come into play only if a contract had been formed and the question asked whether the supplier had a defense to excuse nonperformance. Answer choice B is incorrect because both the date the supplier tried to change the price and the date the restaurant accepted the offer fall within the three-month period of irrevocability. Therefore, the offer was still open and irrevocable without consideration when the restaurant accepted it on July 1. Answer choice D is incorrect because an offer can be accepted only when it is still outstanding, regardless of whether the acceptance occurred in a reasonable time. If the offer here had been revocable, the fact that the restaurant tried to accept the offer after its revocation but within a reasonable time would not create an enforceable contract.

A refrigeration-unit manufacturer contracted with a kitchen appliance store to sell and deliver 100 refrigeration units to the store at a price substantially lower than market value. The written and signed contract included the term "F.O.B. kitchen appliance store, on or before March 30." Due to an unforeseen strike by the shipping company, the manufacturer delivered the units to the store on April 18. The store suffered no material harm due to the delay. The refrigeration appliance industry generally allows appliance manufacturers a 30-day leeway for any contractually specified time of delivery, unless such leeway is expressly forbidden by the contract. If the store brings suit against the manufacturer for breach of contract, which of the following facts provides the manufacturer with the strongest defense? (a) The delay was caused by an unforeseeable strike. (b) The manufacturer believed that due to the price at which it offered the refrigeration units, the store would accept a late delivery. (c) There is evidence of a trade usage in the refrigeration appliance industry allowing a 30-day leeway for appliance deliveries. (d) The store suffered no material harm from the delay.

Answer choice C is correct. Under the UCC, even if the terms of a written contract for the sale of goods appear to be unambiguous, a party may explain or supplement the terms by evidence of trade usage or course of dealings or performance. Trade usage is any practice or method of dealing in the particular business or industry that is practiced with such regularity so as to justify an expectation that it will be practiced in the instant case. Here, the manufacturer's strongest argument will be that a trade usage applies, and that the terms of the contract should be supplemented by the trade usage in the industry allowing a 30-day leeway for appliance deliveries. Answer choice A is incorrect. While it is true that unforeseeable strikes have been found to excuse performance, here, performance was not impracticable; it was simply delayed. The manufacturer itself ultimately delivered the units to the store, and there is no indication that they could not have done so by the agreed-upon delivery date. Therefore, although the foreseeability of the strike will be relevant to calculating damages, it is not the manufacturer's strongest defense against the breach of contract claim. Answer choice B is incorrect because the manufacturer will not be excused for breaching a contract simply because it believed offering low prices would encourage the store to accept even a nonconforming performance. Answer choice D is incorrect because material harm is not a necessary element of bringing a breach of contract action, though it may be relevant to damages.

A law student attended law school on a full scholarship. At the end of the law student's second year, the law student lost her scholarship. In order to fund her third year, she borrowed $50,000 from her rich uncle. They executed a written agreement stating that the law student would repay the loan two years from May 15, the date of her law school graduation. On May 15, two years later, the law student did not pay her uncle back because she was unable to find a job as a lawyer. Instead, she was working as a server at a coffee shop. The uncle took no legal action. Four years later, the law student was still unable to pay the uncle back, but she did write him a letter, stating "I know I still owe you $50,000. I will repay you $50,000 if I get a law firm job." The statute of limitations for collecting debts in the jurisdiction is three years. Is the law student's promise contained in the letter to repay the loan enforceable? (a) No, because the promise to repay is not supported by consideration. (b) No, because the repayment of the loan was contingent upon her getting a law firm job. (c) Yes, because the uncle's foregoing of legal action constitutes consideration. (d) Yes, because the promise was made after the statute of limitations had run.

Answer choice D is correct. A new promise to pay a debt after the statute of limitations has run is enforceable without any new consideration. Here, the law student promised to repay the loan after the statute of limitations had run. Thus, her new promise to pay the $50,000 debt is enforceable without consideration. For this reason, answer choice A is incorrect. Answer choice B is incorrect. The repayment of a loan can be subject to an express condition. The law student's promise is enforceable because the statute of limitations had run. However, the law student does not need to perform unless she gets a job with a law firm. Answer choice C is incorrect. Although the foregoing of a legal right can constitute consideration, here there is no indication that the law student made her promise in the letter in exchange for the uncle's promise not to sue her to collect the outstanding debt. In addition, at the time that the law student made her promise in the letter, the uncle could not successfully pursue his legal remedy because the statute of limitations had run on collection of the debt. The promise is only enforceable because the law student made a new promise to pay the debt after the statute of limitations had run.

A grocery chain whose main customer base was families with young children contacted a cereal manufacturer. After various discussions regarding the cereal and the box, the two parties entered into a written contract whereby the grocery chain would purchase 10,000 boxes of children's cereal on a monthly basis for $5,000, due upon delivery. The contract further stated that the cereal would be shaped like donuts, and each piece of cereal would be one of the seven colors of the rainbow. Finally, in listing the primary ingredients, the contract stated that the cereal would contain high fructose corn syrup. When the first shipment arrived, the grocery chain refused to pay the $5,000 and repudiated the contract. The cereal manufacturer sued the grocery chain for damages, and admitted the contract between the two parties into evidence. The grocery chain then attempted to offer evidence regarding the discussions that occurred between the two parties prior to the execution of the contract. It claimed that the cereal manufacturer had orally agreed to use evaporated cane juice as a sweetener, not high fructose corn syrup, and that the cereal manufacturer would also include a small prize in each cereal box at a cost of a penny a box. In deciding whether to admit evidence of the oral agreement, the court will most likely (a) Admit neither piece of evidence from the oral agreement. (b) Admit only the evidence regarding the use of evaporated cane juice as a sweetener. (c) Admit only the evidence regarding the small prize in each cereal box. (d) Admit both pieces of evidence from the oral agreement.

Answer choice C is correct. Under the parol evidence rule, evidence that contradicts the writing is generally inadmissible, but evidence that supplements a contract that is partially integrated is admissible if it is consistent with the writing and does not contradict its terms. The UCC essentially presumes that a written contract is only a partial integration. Evidence of any outside term is admissible unless a court concludes that the parties "certainly" would have included the term in the written contract. Here, the evidence about the small prizes supplements the written agreement and does not contradict it. The cost of the prize, both in actual amount (a penny) and in relationship to the overall price of the cereal (1 cent v. 50 cents ($5,000/10,000)) suggests that the parties would not certainly have included this item in their contract. Accordingly, this evidence is admissible. However, the evidence about using evaporated cane juice as a sweetener instead of high fructose corn syrup contradicts the terms of the written agreement and is therefore inadmissible. For these reasons, answer choices A, B, and D are incorrect.

A maker of perfume contacted a manufacturer about supplying 1,000 readily available glass bottles for retail sales of the perfume. The manufacturer offered to supply the bottles and to ship them within one week. The perfumer responded, "Ship them as soon as possible." The manufacturer shipped 1,000 bottles to the perfumer five days later. The perfumer accepted the bottles and filled them with perfume. Without waiting for the manufacturer's invoice, the perfumer sent a payment to the manufacturer based on a price of $2.50 per bottle. Prior to receiving this payment, the manufacturer sent the perfumer an invoice, which reflected a charge of $3.50 per bottle. When the perfumer refused to pay $3.50 per bottle, the manufacturer returned the payment to the perfumer and initiated an action for the price. The court determined that a reasonable price for the bottles at the time of delivery was $3.25 per bottle. What amount should the court award the manufacturer per bottle? (a) Nothing, because no contract was formed in the absence of a price term. (b) $2.50, because the perfumer, as offeror, was master of the offer. (c) $3.25, because this was a reasonable price for the bottles at the time of delivery. (d) $3.50, because the manufacturer, as offeror, was master of the offer.

Answer choice C is correct. When a buyer and a seller have entered into a contract for the sale of goods, the Uniform Commercial Code (UCC) provides that a court may supply the missing term, even when that term is the price of the goods. When a buyer has accepted the goods and fails to pay the price when it becomes due, the seller may sue for the price. If the contract omits a price or if the parties agree to set the price in the future and then fail to agree, the UCC supplies a reasonable price at the time of delivery. In this case, the perfumer and the manufacturer have entered into a contract for the sale of goods, as the manufacturer shipped the bottles and the perfumer received and accepted them. Because the price term is missing, the price will be a reasonable price at the time of delivery, which in this case is $3.25 per bottle. Answer choice A is incorrect. Although the absence of a price term can be fatal to the formation of a contract under common law, the lack of a price term will not cause a contract for the sale of goods under the UCC to fail due to indefiniteness if the parties to the contract have indicated their intent to contract. Here, that intent is evidenced by the manufacturer's shipment of the bottles together with the perfumer's receipt and acceptance of the bottles. In such situations, the burden falls on the court to ascertain a reasonable price for the goods at the time of delivery. Answer choice B is incorrect. It is unclear whether the perfumer or the manufacturer was the offeror. Arguably, the perfumer's initial contact with the manufacturer was not an offer, but only an invitation to offer, to which the manufacturer responded with an offer that the perfumer accepted. However, even if the perfumer's response to the manufacturer, which varied the shipping date, constituted a counteroffer, there is no indication that the perfumer conveyed to the manufacturer a price for the bottles until after the perfumer had received and accepted the goods. Consequently, the perfumer's price of $2.50 per bottle was not part of its offer. Answer choice D is incorrect. Although an offeror is master of the offer, in this case, the offer, even assuming it was made by the manufacturer in response to the perfumer's invitation to make an offer, did not specify the price of the bottles. The price sought by the manufacturer ($3.50 per bottle) was not expressed until after the bottles had been shipped and accepted, which is after the contract had been formed.

A bank that held a security interest in a delivery van conducted a forced sale of the van at an auction after the owner of the van defaulted on its loan from the bank. The proceeds of the loan had been used to purchase the van, which the owner had used in his floral business. At the auction, which was held in accordance with statutory requirements, the owner, without disclosing his ownership interest, made a good faith bid on the van. Twelve days after the auction, the highest bidder filed an action to void the sale after learning that one of the bidders had been the owner of the van. Can the sale be voided? (a) Yes, because the owner bid at the auction without disclosing his interest in the van. (b) Yes, because the owner was a merchant. (c) No, because the auction was a forced sale of the van. (d) No, because the highest bidder did not file his action to void the sale within 10 days of the auction.

Answer choice C is correct. When an auctioneer knowingly accepts a bid by the seller or on her behalf, or procures such a bid to drive up the price of the goods, the winning bidder may avoid the sale or, at her option, take the goods at the price of the last good-faith bid prior to the end of the auction. There are two exceptions to this rule, which are that (i) a seller may bid at a forced sale and (ii) a seller may bid if she specifically gives notice that she reserves the right to bid. In this case, the bank conducted a forced sale of the owner's delivery van. As a result, the owner (seller) was permitted to bid on the van at the auction, even without first disclosing his interest in the van and reserving the right to bid. Therefore, the sale cannot be voided. Answer choice A is incorrect. While the owner of goods subject to an auction is generally required to disclose his ownership interest in the goods and reserve the right to bid prior to bidding in the auction or run the risk that the buyer can void the sale, this disclosure is not required when the goods are sold at a forced sale. Answer choice B is incorrect because there is no prohibition on an owner who is a merchant from bidding on his own goods at a forced sale auction. Answer choice D is incorrect because there is no statutory time period under the U.C.C. in which the buyer at an auction must act in order to void the sale.

On October 1, a retail sporting equipment store telephoned a shoe manufacturer and offered to buy a minimum of 50 and a maximum of 100 pairs of running shoes at $40 a pair to be delivered in 60 days. The manufacturer accepted the offer orally and immediately faxed a signed letter to the store. The letter contained the following language: "This letter confirms our agreement by telephone on October 1 to sell you 50 pairs of running shoes for 60-day delivery." Sixty days later, the manufacturer delivered 100 pairs of conforming running shoes to the store. However, the store rejected all 100 pairs of shoes because it had found the same shoes from another seller for $30 a pair. Due to lack of demand, the manufacturer cannot resell the 100 shoes without suffering an economic loss. Can the manufacturer enforce a contract against the store? (a) No, because the manufacturer did not state the agreed-upon price term in the faxed letter. (b) No, because the store did not sign the faxed letter. (c) Yes, for the sale of 50 pairs of shoes because the manufacturer's faxed letter stated that quantity term. (d) Yes, for the sale of 100 pairs of shoes because the store wrongfully rejected that quantity of properly delivered and conforming shoes.

Answer choice C is correct. When both parties to a contract governed by the UCC are merchants, the acceptances are often default forms that contain different and additional terms from those specifically agreed upon by the parties. An additional term in the acceptance is automatically included in the contract when both parties are merchants, unless the term materially alters the original contract, the offer expressly limits acceptance to the terms of the offer, or the offeror has already objected to the additional terms, or objects within a reasonable time after notice of them was received. A term that results in surprise or hardship if incorporated without the express awareness by the other party materially alters the original contract. Here, the store did not clearly define a quantity term in its offer. Therefore, the quantity term in the manufacturer's confirming faxed letter qualifies as an additional term. However, this quantity term does not materially change the agreement because it is within the scope defined by the store's offer, and is therefore unlikely to result in surprise or hardship to the store. The offer does not expressly limit acceptance to a particular quantity term, and the store never objected to the additional term after receiving the letter. Therefore, the quantity term in the faxed letter controls, and the manufacturer can bring an action to enforce the sale of 50 pairs of shoes. Answer choice A is incorrect. Under the UCC, a contract is formed if both parties intend to contract and there is a reasonably certain basis for giving a remedy. The only essential term is quantity, and as long as the parties intend to create a contract, the UCC "fills the gap" if other terms are missing, such as the time or place for delivery, or even the price for the goods. Therefore, the absence of the price term in the faxed letter will not prevent it from being an enforceable contract under the UCC. Answer choice B is incorrect because if both parties to a transaction governed by the Statute of Frauds are merchants, a memorandum sufficient against one party is sent to the other party, and the receiving party does not object in writing within 10 days, then the contract is enforceable against the receiving party even though he has not signed it. Answer choice D is incorrect because the manufacturer is not entitled to enforce a contract for the sale of 100 pairs of shoes simply because 100 shoes were delivered. Instead, the quantity term in the faxed letter controls, and therefore the manufacturer can only enforce a contract for the sale of 50 pairs of shoes.

A painter entered into a contract with a homeowner to paint the exterior of the homeowner's home over a weekend while the homeowner was on vacation. After the homeowner left on his vacation, the painter was offered a second job that paid slightly more during the same weekend. The painter delegated his duty under the first contract to a second painter in exchange for a $50 advance and a promise to split his profits from both jobs with the second painter. The second painter took the advance and agreed to paint the homeowner's home that weekend. The first painter worked on the second job that weekend and did not check on the homeowner's home. When the homeowner returned from vacation, he discovered that his home had not been painted at all. In addition, no one has been able to locate the second painter. Does the homeowner have a cause of action against the first painter? (a) No, because the homeowner has not exhausted his remedies against the second painter. (b) No, because the second painter accepted consideration in exchange for his promise to paint the home. (c) Yes, because the first painter was not released from his liability to the homeowner. (d) Yes, because the homeowner is unable to recover from the missing second painter.

Answer choice C is correct. When obligations are delegated, the delegator is not released from liability, and recovery can be had against the delegator if the delegate does not perform, unless the other party to the contract agrees to release that party and substitute a new one (i.e., a novation occurs). Therefore, the first painter (the delegator) is still liable under the contract, and the homeowner has a valid cause of action against him. Answer choice A is incorrect because the homeowner need not exhaust any remedies against the second painter before bringing a cause of action against the first painter. Answer choice B is incorrect. The fact that the second painter accepted consideration in exchange for accepting the delegation does not affect the first painter's liability under the contract. Answer choice D is incorrect because, as stated above, the homeowner is not required to bring an action against the second painter. Therefore, his unavailability is irrelevant to the first painter's liability under the contract.

A boutique hotel contracted with a seamstress to hand make 500 pillows. The signed contract specified that the pillows should be filled with down, and that the pillow covers be made with white, 1000 thread count cotton fabric. Before the seamstress began making the pillows for the boutique hotel, she secured another commission for work that would prevent her from making the hotel's pillows. As a result, the seamstress informed the boutique hotel that she was passing on the hotel's contract to her former business partner, who was comparable in talent and skill at making high-quality pillows. The boutique hotel did not object to the substitution. The former partner diligently worked on making the pillows, using white, 1000 thread count fabric to make the pillow covers. However, instead of using down to fill the pillows, she used a comparably priced synthetic microfiber. The boutique hotel subsequently filed a breach of contract action against the seamstress. Will it succeed? (a) No, because the former partner's use of a synthetic microfiber instead of down did not reduce the value of the pillows. (b) No, because the boutique hotel was aware of, and did not object to the delegation of the seamstress's duties to her former partner. (c) Yes, because the boutique hotel had not released the seamstress from liability under the contract. (d) Yes, because the seamstress did not give consideration for delegating the contract to the former partner.

Answer choice C is correct. When obligations are delegated, the delegator is not released from liability, and recovery can be had against the delegator if the delegatee does not perform, unless the other party to the contract agrees to release that party and substitute a new one (a novation). Here, the former partner filled the pillows with a microfiber instead of down. Thus, the seamstress is liable under the contract for her former partner's failure to comply with the terms of the contract. Answer choice A is incorrect. Even if the former partner's breach constitutes a minor breach, the contract called for the use of down rather than a microfiber. Because this is a contract for goods (i.e., pillows), perfect tender is required. Consequently, the boutique hotel would be entitled to damages and the seamstress remains contractually liable for those damages. Answer choice B is incorrect. Although the boutique hotel was aware of, and did not object to the delegation, this awareness and failure to object, while constituting an acceptance of the delegation, does not constitute a novation. While a novation may be implied as well as expressed as a consequence of a delegation, the obligee awareness of the delegation and subsequent acceptance of performance of the original agreement from the delegatee does not constitute a novation unless the original obligor has repudiated liability to the original promisee and the obligee accepts the delegate's performance without reserving rights against the obligor. Here, there is no indication the seamstress repudiated her liability to the hotel, so a novation did not occur. Answer choice D is incorrect because consideration is not necessary for the delegation to be effective.

A baker and a bride-to-be entered into a contract in which the baker agreed to bake the wedding cake for the bride's wedding at a cost of $2,500. The contract contained a clause that read: "An express condition of Bride's performance under the Contract is Baker's satisfaction of Bride's aesthetic expectations in the design of her wedding cake." In keeping with the wedding's butterfly theme, the baker constructed an elegant cake accented with colorful butterflies, flowers, and caterpillars. At the wedding reception, the guests were enthralled by the cake. The bride, however, upset over the inclusion of the caterpillars, to which she had a genuine aversion, refused to pay the baker. The baker sued the bride for $2,500. Should the court require the bride to pay the baker? (a) Yes, because the cake was aesthetically pleasing to the wedding guests. (b) Yes, because the baker substantially performed. (c) No, because the bride was personally and honestly dissatisfied with the cake. (d) No, because no contract was formed between the parties.

Answer choice C is correct. When parties expressly agree to a condition, they are generally held strictly to that condition; a party must fully comply with that condition before the other party's performance is due. When the aesthetic taste of a party determines whether the other party's performance is satisfactory (e.g., painting a family portrait), satisfaction is determined under a subjective standard. Under this standard, if the party is honestly dissatisfied, even if the dissatisfaction is unreasonable, the condition has not been met. However, the party's dissatisfaction must be in good faith, or a claim of dissatisfaction can be a breach, such as when a party is asserting dissatisfaction merely to avoid its own contractual obligation. Here, the bride's dissatisfaction, although subjective, was based on her aversion to caterpillars rather than a bad faith expression of dissatisfaction in order to avoid paying the baker. Therefore, the bride is discharged from her contractual obligation to pay the baker for his work. Answer choice A is incorrect because the reasonable person standard only applies to non-aesthetic conditions of satisfaction. Here, subjective aesthetics are at issue. Answer choice B is incorrect because substantial performance is not an appropriate standard in a condition based on aesthetic taste. Answer choice D is incorrect because the inclusion of an aesthetic satisfaction clause in a sale of goods contract does not make an otherwise valid contract void for vagueness or lack of consideration; there is an implied requirement that the party will act in good faith.

After the death of a farmer, the executor of his estate held an auction sale of his farm equipment. The executor specified that she reserved the right to withdraw any item from the sale. A neighbor placed a bid on a tractor. The bid was acknowledged by the auctioneer. Before another bid was placed or the auctioneer announced the completion of the sale, the neighbor informed the auctioneer that he was withdrawing his bid. Must the auctioneer permit the neighbor to withdraw his bid? (a) No, the auctioneer may, but is not required to, accept the withdrawal of a bid. (b) No, because there is no right to withdraw a bid. (c) Yes, because the seller retained the right to withdraw an item from the sale. (d) Yes, because the auctioneer had not announced the completion of the sale.

Answer choice D is correct. A bidder may retract his bid until the auctioneer announces the completion of the sale. Here, the neighbor informed the auctioneer that he was withdrawing his bid before the auctioneer announced the completion of his sale. Therefore, the auctioneer must permit the neighbor to withdraw his bid on the tractor. Answer choice A is incorrect because the option of withdrawing a bid lies with the bidder, not the auctioneer. As with any other revocation of an offer, the bidder must convey his withdrawal to the auctioneer. So long as the withdrawal is communicated to the auctioneer prior to the auctioneer announcing the completion of the sale, the auctioneer must allow the bidder to withdraw his bid. Accordingly, answer choice B is also incorrect. Answer choice C is incorrect. Regardless of whether the seller does or does not retain the right to withdraw an item from sale at the auction (i.e., the sale is with or without reserve), the bidder retains the right to withdraw a bid.

An independent trucker and a manufacturer entered a contract for the delivery of a farming implement from the manufacturer to a farmer. Under the terms of the written contract, the trucker promised "to deliver a farming implement from the manufacturer to the farmer," and the manufacturer promised "to pay the trucker if the trucker delivers the implement directly to the farmer after picking it up." The trucker picked up the implement, but instead of driving directly to the farmer, drove 100 miles out of his way to pick up another item from a third party before delivering the implement to the farmer. The manufacturer, unaware that the trucker failed to deliver the implement directly to the farmer, refused to pay the trucker. Who has breached this contract? (a) Both the trucker and the manufacturer. (b) The trucker only. (c) The manufacturer only. (d) Neither the trucker nor the manufacturer.

Answer choice D is correct. A breach of contract can only occur when a party fails to perform a duty that has become due. Because the trucker performed his contractual duty, and because the manufacturer's duty was subject to an unsatisfied express condition precedent, neither party here is in breach of the contract. Answer choices A and B are incorrect. A "condition" is a future event that must take place before a party's contractual rights or obligations are created, destroyed, or enlarged. By contrast, a "promise" constitutes a party's obligation to act or refrain from acting. Non-occurrence of a condition is not a breach by a party unless the party has promised that the condition will occur. When parties expressly agree to a condition precedent, the express condition precedent must be satisfied or excused before the other party's performance is due. The trucker promised only "to deliver the farming implement from the manufacturer to the farmer." Therefore, the trucker is not in breach because he substantially performed this duty by delivering the goods, even if he did not satisfy the condition that would trigger the manufacturer's duty to pay. Answer choice C is incorrect. The manufacturer's duty to pay the trucker was expressly conditioned on the trucker delivering the farm implement directly to the farmer. The facts provide that the trucker failed to satisfy this condition precedent. Therefore, the manufacturer is under no contractual duty to pay. Because a breach of contract can only occur when a party fails to perform a duty that has become due, the manufacturer is not in breach for refusing to pay the trucker. Even though the manufacturer is not in breach, note that the trucker is not without remedy. Since the trucker conferred a benefit on the manufacturer (i.e., the value of having the farm implement delivered to the farmer), the trucker may be able to seek restitution.

A nature magazine advertised a photography contest in its January issue, offering "$1,000 to any subscriber who sends us a photograph of the rare Florida Grasshopper Sparrow that we use for the cover of our May issue. Only submissions meeting our technical specifications and received by April 1 will be considered." The only subscriber to respond to the advertised contest sent the magazine a photograph of the sparrow that met the magazine's technical specifications. The photograph arrived on March 15. However, due to an ecological disaster that occurred in early April, the magazine used a different picture on the cover of its May issue. The magazine used the picture on the cover of its June issue, and has refused to pay $1,000 to the subscriber on the grounds that it was not used on the May cover. Is the subscriber likely to prevail in a breach of contract action against the nature magazine? (a) No, because the subscriber's photo was not used on the cover of the May issue. (b) No, because the subscriber failed to adequately notify the magazine of his acceptance. (c) Yes, because all of the express conditions of the offer have been satisfied. (d) Yes, because the nature magazine prevented the publication of the photograph.

Answer choice D is correct. A performance that is subject to an express condition cannot become due unless the condition occurs or its nonoccurrence is excused. The subscriber's entitlement to the reward was subject to three conditions—the provision of a satisfactory photograph, the provision of that photograph by April 1, and the use of the photograph on the cover of the magazine's May issue. The first two conditions were satisfied when the subscriber delivered the photograph to the magazine. The third condition did not occur, but its nonoccurrence is excused under the doctrine of prevention, which requires that a party refrain from conduct that wrongfully prevents or interferes with the occurrence of a condition. In this case, the magazine itself prevented the May 1 publication condition from occurring. Answer choice A is incorrect because the failure of this condition is excused under the doctrine of prevention. Answer choice B is incorrect. In a unilateral contract, an offeree is not required to give notice after performance is complete, unless he has reason to know that the offeror would not learn of performance within a reasonable time, or the offer requires notice. Such is not the case here. Answer choice C is incorrect because all of the express conditions of the offer have not been satisfied. The photograph of the sparrow did not appear on the cover of the May issue of the magazine. However, his nonperformance of the publication condition is excused by the doctrine of prevention.

An owner of a nail salon started a new promotion to boost sales. The owner claimed that she would pay $100 to any customer who purchased a new type of long-lasting manicure from her nail salon over the next four months if the customer's manicure did not stay intact for an entire month. A customer purchased a manicure during the promotional period per the terms set forth by the owner, but the customer's manicure only stayed intact for three weeks. The customer tried to collect the $100 from the owner, but the owner refused to pay the customer. Can the customer collect the $100? (a) No, because the customer did not notify the owner that she was accepting the offer prior to purchasing the manicure. (b) No, because there was no bargained-for exchange. (c) Yes, because this was an enforceable bilateral contract. (d) Yes, because the customer accepted the offer by purchasing the long-lasting manicure during the promotional period.

Answer choice D is correct. A unilateral contract is one in which one party promises to do something in return for an act of the other party. Unlike in a bilateral contract, in a unilateral contract, the offeree's promise to perform is insufficient to constitute acceptance. Acceptance of an offer for a unilateral contract requires complete performance. A unilateral contract is not formed until performance is complete. Here, the customer accepted the offer and completed performance by purchasing a long-lasting manicure during the promotional period. Thus, an enforceable contract was formed. Because the manicure did not stay intact for a month, the customer can collect the $100 from the owner. Answer choice A is incorrect. In a unilateral contract, an offeree is not required to give notice after performance is complete, unless he has reason to know that the offeror would not learn of performance within a reasonable time, or the offer requires notice. Here, the owner would know about the manicure because the customers were coming into her nail salon. Additionally, notice was not requested by the owner. Therefore, the customer was not required to notify the owner. Answer choice B is incorrect because there was consideration (bargained-for exchange) in this case. In exchange for the customer purchasing a long-lasting manicure during the promotional period, the owner promised to pay her $100 if the manicure did not stay intact for one month. Answer choice C is incorrect because, as stated above, this fact pattern involved a unilateral contract. A bilateral contract is one in which a promise by one party is exchanged for a promise by the other. Here, only the owner is making a promise, whereas the customer's acceptance is through performance of an act.

A mining company contracted with a railroad to transport 10,000 tons of coal from the company's mines to a power company at a cost of $100,000. The railroad told the mining company that the coal would arrive at the power company on June 1, but the contract contained a clause that the railroad would not be liable for any losses suffered by the mining company as a result of a late shipment. The railroad was aware that the mining company had contracted with the power company to deliver the coal on June 1, and pursuant to standard industry custom, the price to be paid by the power company decreased by $1 per ton for each day that the coal was late. The shipment of coal did not reach the power company until June 11, and the railroad had no justification for the 10-day delay. Because of the delay, the mining company lost $100,000 in revenue from the sale. The mining company filed suit against the railroad for breach of contract, claiming $100,000 in damages. Is the mining company likely to succeed in its claim? (a) Yes, because the damages that the mining company would suffer from the railroad's delay were known to the railroad prior to shipment of the coal. (b) Yes, because consequential damages cannot be excluded by a merchant. (c) No, because the claimed damages are disproportionate to the original contract price between the railroad and the mining company. (d) No, because the contract between the mining company and the railroad protected the railroad from losses suffered by the mining company due to a late shipment.

Answer choice D is correct. Although a party may be liable for consequential damages of the other party to a contract where those damages are foreseeable, a party may eliminate that liability through an agreement with the other party. Here, while the railroad was aware of the mining company's liability for failing to supply the power company with coal on June 1st, the contract between the railroad and the mining company eliminated the railroad's liability for any losses suffered by the mining company due to late delivery. Answer choice A is incorrect because the parties to a contract are generally free to allocate responsibility for consequential damages as they see fit. So even though the mining company's liability due to a late shipment was known to the railroad, the railroad expressly disavowed responsibility for that liability. Answer choice B is incorrect because, although the UCC does prohibit the limitation or exclusion of liability for consequential damages, this prohibition applies only where limitation or exclusion is unconscionable, and a commercial loss is not prima facia unconscionable. In addition, the contract at issue (i.e., between the railroad and the mining company) is not a sale of goods and hence not governed by the UCC. Answer choice C is incorrect because, while the lack of proportionality between the contract price and the amount of consequential damages may be an underlying factor in determining whether such damages are foreseeable, proportionality alone is not determinative as to whether such damages are foreseeable. In any case, the parties may generally allocate responsibility for consequential damages as they see fit.

The owner of a retail clothing store regularly displayed for-sale works by local artists on a wall in the store. An art collector who came into the store inquired about purchasing a particular work for display at his home. The two agreed upon a price, but the collector was not ready to commit to purchasing it immediately. Confident that the collector would purchase the work, the owner promised in a signed writing to sell the work to the collector at the agreed-upon price at any time before the end of the month. On the last day of the month, the collector sent the owner a check for the agreed upon price, which the owner received on the following day. If the owner returns the collector's check and refuses to sell the artwork to the collector, which of the following best supports the owner's position that a contract had not been formed? (a) The firm-offer rule is not applicable because the owner was not a merchant with respect to the work of art. (b) The firm-offer rule is not applicable because the collector was not a merchant with respect to the work of art. (c) The collector could not accept the owner's offer by mailing a check. (d) The collector's acceptance of the owner's offer was not timely.

Answer choice D is correct. Although under the mailbox rule an acceptance is effective upon dispatch, the mailbox rule does not apply when the offer is irrevocable. Instead, in such case, the acceptance must be received before the offer expires. Here, the owner promised to hold her offer open until the end of the month. In order to be effective, the acceptance had to be received by the owner before the end of the month. Therefore, the best argument in favor of the owner's position is that the collector failed to accept the offer by the last day of the month, because the check arrived a day later. Answer choice A is incorrect because the owner's signed written promise did create an irrevocable offer under the UCC firm-offer rule. While the rule only applies to offers made by merchants, the definition of merchant under the firm offer rule includes not only a person who regularly deals in the type of goods that are the subject of the offer (here, artwork) or otherwise by his occupation holds himself out as having knowledge or skill peculiar to the practices or goods involved in the transaction, but also any businessperson when the transaction is of a commercial nature. Answer choice A and B both indicate that the firm offer rule is inapplicable, which is not true under this definition. Answer choice B is also incorrect because the firm offer rule looks at the status of the offeror as a merchant, not the offeree; it does not require that the offeree be a merchant. Consequently, the firm-offer rule applied due to the owner's status as a merchant. Answer choice C is incorrect because, unless the offer specifies otherwise, an offer may generally be accepted by either a promise or performance. Consequently, the collector's payment of the purchase price could constitute an acceptance.

A wedding planner contracted with a local bakery to make cupcakes for an upcoming wedding reception. The bakery was very experienced in making cupcakes, and had a great reputation in the community. Although there were other comparably skilled cupcake makers in the area, the wedding planner eventually chose the bakery due to the price it quoted for the cupcakes. A few months before the wedding reception, the bakery's head baker unexpectedly had to take a leave of absence to deal with a medical issue. The bakery subsequently assigned the contract to a pastry chef in the same community. The pastry chef also had an excellent reputation in the community, and was at least equally as skilled in making cupcakes as the bakery. The bakery told the wedding planner about this assignment, and the wedding planner did not object. When the pastry chef fully performed on the contract and delivered the cupcakes, which conformed to the contract requirements, the wedding planner refused to accept or pay for the cupcakes. On these facts, has the wedding planner breached the contract? (a) No, because the bakery breached the contract first by assigning the contract to another party. (b) No, because the wedding planner only had a duty to accept performance by the bakery. (c) Yes, because the bakery told the wedding planner about the assignment of the contract and the wedding planner did not object. (d) Yes, because the assignment of the contract was permitted and the pastry chef properly and fully performed.

Answer choice D is correct. An assignment of a contract that is not limited to contractual rights is typically treated as both an assignment of rights and a delegation of duties. Generally, obligations under a contract can be delegated unless the other party to the contract has a substantial interest in having the delegating individual perform (for example, in a personal services contract involving taste or a special skill) or the delegation is prohibited by the contract. Here, it is unlikely that the wedding planner had a substantial interest in having the bakery make the cupcakes. The wedding planner's decision to use the bakery was based on the price it quoted, rather than the skill of the bakery, as there were other comparably skilled bakers in the area. Additionally, the delegation is not prohibited by the contract. Thus, the delegation was proper and permitted, and the pastry chef properly performed. The assignment also properly assigned the bakery's right to payment to the pastry chef. Therefore, the wedding planner has breached the contract by refusing to accept or pay for the cupcakes. Answer choice A is incorrect for the reasons stated above, because the contract did not prohibit assignment, and the assignment did not materially increase the duty or risk of the obligor or materially reduce the obligor's chance of obtaining performance. Answer choice B is incorrect because the entire contract, including the bakery's duties and obligations, has been properly assigned to the pastry chef. Therefore, the wedding planner has breached the contract by refusing the pastry chef's performance and refusing to pay him. Answer choice C is incorrect because the wedding planner's consent or lack of consent to the assignment of the contract is only relevant as to whether there was a novation that would limit his ability to recover against the bakery. It is not relevant to the enforceability of the assignment of rights and delegation of duties under this contract.

A produce wholesaler sent a written offer to a farmer to purchase all of the corn produced by the farmer for a period of two years. Excited at the prospect of having a guaranteed sale for all of his corn, the farmer immediately communicated his acceptance to the wholesaler. The wholesaler and farmer entered into a written contract reflecting the basic terms set forth in the wholesaler's offer. Six months after the contract was executed, the wholesaler determined that, while the farmer's corn was returning a profit, the farmer's corn was not selling as well as corn that the wholesaler could acquire from other sources. Nevertheless, the wholesaler contacted the farmer and informed him that he would no longer purchase the farmer's corn and would be buying his corn from another source. If the farmer sues the wholesaler for breach of contract, is he likely to prevail? (a) No, because the contract did not contain a specific quantity term. (b) No, because the wholesaler no longer needed the farmer's goods. (c) Yes, because the farmer relied on the wholesaler's promise. (d) Yes, because the wholesaler did not act in good faith.

Answer choice D is correct. An output contract is a contract under which a seller agrees to sell all that she manufactures of a product to the buyer. Here, the wholesaler still needed corn, and was choosing instead to purchase it from another party. There is no indication that this situation would allow the wholesaler to cancel the contract without violating the good-faith requirement of the contract. Therefore, the farmer will likely succeed in a breach of contract action against the wholesaler. Answer choice A is incorrect because output contracts satisfy UCC formation requirements even without naming specific quantities or a specific price. Answer choice B is incorrect. The wholesaler here still needed the farmer's goods; he just chose to buy them elsewhere. Therefore, he could not cancel the contract in good faith on these grounds. Answer choice C is incorrect. The farmer and wholesaler entered into a valid and enforceable output contract supported by consideration. Accordingly, the farmer does not need to show reliance on the wholesaler's promise (i.e., promissory estoppel) to prevail.

A private port authority contracted with a crane operating company to assist with loading and unloading containers from ships docked at the port. One of the company's cranes was defectively manufactured. Due to this defect, a container was dropped. The container tumbled down a hill, crashed through a fence, and struck a passerby. The passerby sued the port authority alleging negligence. Neither the passerby nor the port authority notified the crane operating company of this lawsuit. The port authority settled its claim with the passerby before trial for a reasonable amount. The port authority seeks to recover the cost of the settlement from the crane operating company under a breach of contract action. Of the following, which would be the crane operating company's best defense? (a) The crane operating company was not notified of the lawsuit. (b) Damages for personal injury cannot be recovered in a breach of contract action. (c) The port authority settled the lawsuit rather than litigating the matter to a final judgment. (d) The settlement was not reasonably foreseeable at the time the contract was formed.

Answer choice D is correct. Consequential damages are recoverable in a breach of contract actions only if they were reasonably foreseeable at the time that the contract was entered into. While it is arguable that, although the actual means by which the passerby was injured was not foreseeable, the fact that a person could be injured by a dropped container and therefore recover in a negligence action was foreseeable, the other answer choices present arguments that would not protect the crane operating company from contractual liability regardless of whether the passerby's injuries and the subsequent lawsuit and its settlement were foreseeable. Answer choice A is incorrect because, while the failure to provide the crane operating company with notice of the lawsuit would protect the crane operating company from liability directly imposed on a party to the lawsuit, this lack of notification would not protect the crane operating company from contractual liability to the port authority if the settlement was reasonably foreseeable at the time that the contract was formed. Answer choice B is incorrect because, while generally damages attributable to a personal injury are recoverable in a tort action rather than a breach of contract action, at issue in the action initiated by the port authority is the foreseeability of those damages and the subsequent settlement by the port authority of a claim from those damages. Answer choice C is incorrect because the fact that the port authority elected to settle the negligence action brought by the passerby for reasonable amount rather than litigate the matter to a final judgment does not prevent the port authority from seeking to recover the cost of the settlement as consequential damages.

A homeowner entered into a written agreement with a company to build a swimming pool in the homeowner's backyard for $40,000, to be paid upon completion of the pool. The company delegated its duty to an independent contractor. The independent contractor began the excavation for the pool, but after realizing that the costs would be higher than anticipated, it abandoned the project. The homeowner hired a partnership to complete the pool for $50,000. Can the homeowner sue the company for its expectation damages of $10,000? (a) No, because the high costs rendered performance under the contract impracticable. (b) No, because the company was released from liability. (c) Yes, because the homeowner did not consent to the delegation. (d) Yes, because the independent contractor did not perform.

Answer choice D is correct. Generally, obligations under a contract can be delegated. When obligations are delegated, the delegator is not released from liability, and recovery can be had against the delegator if the delegate does not perform, unless the other party to the contract agrees to release that party and substitute a new one (a novation). Merely consenting to a delegation does not create a novation. In this case, the independent contractor abandoned the project. Therefore, the homeowner can sue the company, the delegator, for its expectation damages. Answer choice A is incorrect because impracticability is not available merely when the costs of performance under the contract are higher than expected. Answer choice B is incorrect. The facts do not indicate that a novation occurred that would release the company from liability under the contract with the homeowner. Answer choice C is incorrect. The homeowner's consent was not necessary for a proper delegation to the independent contractor under these circumstances.

On November 1, the owner of a yacht posted a flier at a local coffee shop reading "Yacht for Sale: Make me an offer!" The flier also included the owner's phone number. A buyer called the owner on November 3 to ask how much the owner wanted for the yacht. The owner said, "Well, I'd hate to part with it for less than $55,000, but if you can pay me $50,000 by November 20, I suppose I'd sell it to you. I'll hold onto the yacht for you until then." Elated, the buyer took steps to obtain a loan by November 20. On November 15, a second buyer called the owner and offered to buy the yacht for $60,000. The owner immediately accepted, and the second buyer picked up the yacht the next day. On November 20, having obtained a loan, the first buyer visited the owner with a check for $50,000. The first buyer then learned the owner had already sold the yacht. Can the first buyer bring a successful suit against the owner for breach of contract? (a) No, because the owner's statement to the first buyer was only an invitation to deal. (b) No, because the second buyer offered more money for the yacht than the first buyer agreed to pay. (c) Yes, because the owner promised to keep the offer open for a specific period of time. (d) Yes, because the owner's offer to the first buyer was still outstanding on November 20.

Answer choice D is correct. In general, an offer can be revoked by the offeror at any time prior to acceptance, and an offer is revoked when the offeror makes a manifestation of an intention not to enter into the proposed contract. Offers are open for a reasonable time, and an offer can be accepted at any time before the offer is terminated. Here, because the offeror said that he would sell the yacht to the buyer if he could pay by November 20, acceptance by November 20 would be acceptance within a reasonable amount of time. The owner could have revoked the offer before November 20 (the contract did not qualify as an option, as explained below), but the owner never actually revoked the offer to the first buyer. Therefore, the first buyer could still accept the offer on November 20, and the owner can be liable for breaching the contract. Note that this would not qualify as a constructive revocation because the buyer did not learn of the prior sale before he accepted the offer. Answer choice A is incorrect because the owner's statement to the first buyer was sufficiently specific (as to the terms, his intent to make an offer, and who may accept it) to constitute an offer. Answer choice B is incorrect because the fact that the second buyer offered more money is irrelevant to whether a contract was formed between the first buyer and the owner. Answer choice C is incorrect. If the option is a promise not to revoke an offer to enter a new contract, the offeree must generally give separate consideration for the option to be enforceable. Here, the first buyer did not offer consideration to keep the offer open. Even if the owner were considered a merchant in this transaction, the offer to keep the option open was not in a signed writing, and thus the firm offer rule would not apply. Therefore, the offer remained revocable. The reason that the buyer will succeed in this action is not because the offer was irrevocable as an option, but because the owner failed to effectively revoke the offer.

On April 1, a buyer and a seller executed a written contract for the sale of an antique car for $40,000, delivery on May 1. As they each signed the contract, the buyer orally reminded the seller that the buyer's duty to purchase the car was conditioned on his ability to get approval for a loan by April 20 to fund the purchase. The seller orally agreed, though the condition was not noted in the written contract. When the seller contacted the buyer to execute the sale on May 1, he discovered that the buyer attempted but failed to get a loan and cannot afford to purchase the car. The buyer refused to honor the contract. The contract contains a clause indicating that it is a total integration of the parties' agreement. If the seller sues the buyer for breach of contract, will the court admit the evidence of the oral condition regarding the buyer's approval for a loan? (a) No, because the oral agreement contradicts the terms of the written contract. (b) No, because the written contract is a complete integration of the agreement between the parties. (c) Yes, because the oral agreement was a distinct and separate contract. (d) Yes, as proof of a condition precedent to the buyer's obligation under the contract.

Answer choice D is correct. Parol evidence may be admitted to prove a condition precedent to the existence of the contract. Therefore, the oral agreement between the buyer and seller made when they signed the contract is admissible. Answer choice A is incorrect. The parol evidence rule generally prevents a party to a written contract from presenting prior extrinsic evidence that contradicts the terms of the contract as written. However, the parol evidence rule does not apply to evidence of a condition precedent to a party's obligation to perform. Additionally, it does not appear that the additional term evidenced by the oral agreement contradicts the written contract. Answer choice B is incorrect because evidence of a condition precedent to an obligation under a contract is admissible even if the written contract is a complete integration. Answer choice C is incorrect because the oral agreement was merely a condition precedent to an obligation under the written contract, not a separate contract.

A theater owner wanted to renovate the interior of the classic theater that he owned. The theater owner contacted a light vendor and the parties began negotiating a deal for unique stage and overhead lighting for the theater. After a couple of phone calls, the light vendor sent a letter to the theater owner that stated: "20 overhead fixtures and 14 stage lights to be delivered on the first of the month." The theater owner, intending to form a contract, responded with a signed letter containing the following: "Delivery, fixtures, and stage lights sound good. Installation would also be helpful. "After the light vendor received the theater owner's letter, was there an enforceable contract between the theater owner and the light vendor? (a) No, because the light vendor is a merchant and the acceptance materially alters the original contract. (b) No, because the price of the lighting fixtures was not specified in the written offer or acceptance. (c) No, because the theater owner's counteroffer acts as a rejection of the original offer. (d) Yes, because the theater owner and the light vendor intended to create a contract.

Answer choice D is correct. The UCC allows for more liberal contract formation than at common law. Under the UCC, a contract is formed if both parties intend to contract and there is a reasonably certain basis for giving a remedy. Here, the light vendor and theater owner intended to create a contract for the lights and fixtures as demonstrated by their letters. In addition, there is a basis for giving a remedy. Therefore, an enforceable contract was formed when the theater owned sent his signed letter. Answer choice A is incorrect. An additional term in the acceptance is automatically included in the contract when both parties are merchants, unless (i) the term materially alters the original contract; (ii) the offer expressly limits acceptance to the terms of the offer; or (iii) the offeror has already objected to the additional terms, or objects within a reasonable time after notice of them was received. If any one of these three exceptions is met, the term will not become part of the contract. However, a contract would still be formed under the offeror's original terms. Here, even if the theater owner is a merchant with regard to this transaction, the theater owner's letter constituted an acceptance of the light vendor's offer, even though it included an additional term regarding installation not included in the offer. Answer choice B is incorrect. The only essential term is quantity, and as long as the parties intend to create a contract, the U.C.C. "fills the gap" if other terms are missing, such as the time or place for delivery, or even the price for the goods. Answer choice C is incorrect. The UCC does not follow the mirror-image rule. Additional or different terms included in an acceptance of an offer do not automatically constitute a rejection of the original offer. Generally, for a sale of goods, an acceptance that contains additional or different terms with respect to the terms in the offer is nevertheless treated as an acceptance rather than a rejection and a counteroffer. Although the theater owner's letter contained an additional term regarding installation, it is treated as an acceptance under the UCC, not a rejection and a counteroffer.

The owner of a high-rise building entered into a written contract with a company to maintain and service the elevators in the building. The written contract contained the following provision: "This contract is the entire and final agreement of the parties regarding the maintenance and servicing of the elevators in Building. It supersedes any prior agreements, understandings, or negotiations." On the starting date of the contract, the company discovered that the elevators were significantly older than the owner had orally represented to the company during the negotiations prior to the signing of the contract. The company refused to maintain and service the building's elevators unless the owner agreed to a sizeable increase in the monthly payments called for in the contract. The owner refused and found another entity to maintain and service the elevators at a cost below what the company wanted, but above the original contract price. The owner then sued the company for breach of contract, seeking the difference between the contract price and the amount paid to the entity that was currently providing elevator maintenance and service. At trial, the company sought to introduce evidence of the owner's oral statement as to the age of the elevators during contract negotiations. Should the court permit the introduction of this statement? (a) No, because of the parol-evidence rule. (b) No, because the contract is governed by common law rather than the Uniform Commercial Code. (c) Yes, because the statement was oral, not written. (d) Yes, because the statement relates to a contract defense.

Answer choice D is correct. The contract at issue contains a merger clause, which is strong evidence that the writing constitutes a total integration of the parties' agreement and, consequently, that the parol-evidence rule applies to prevent the introduction of prior or contemporaneous evidence that contradicts the terms of the contract. However, the owner's statement regarding the age of the elevators constitutes evidence of misrepresentation, which, if established, would serve as a defense to the formation of the contract itself. Even if the parol-evidence rule does apply to this contract, it does not prevent the company from introducing the statement in its defense. Answer choice A is incorrect. Even though the parol-evidence rule would most likely apply to this contract because the contract contains a merger clause, the rule does not prevent the introduction of evidence that is relevant to establishing a defense to the formation of the contract. Here, the owner's statement made in negotiations prior to the execution of the written agreement is evidence of misrepresentation, which would provide the company with a defense to the formation of the contract. Answer choice B is incorrect. Although the contract as a contract for services is governed by the common law rather than the Uniform Commercial Code (UCC), and the applicable law may have a bearing on the determination of whether or not the parol-evidence rule applies, evidence that serves as a defense to the formation of a contract is admissible even if the parol-evidence rule applies. Answer choice C is incorrect because the parol-evidence rule applies to oral as well as written evidence that predates the execution of a contract. However, that rule does not apply to evidence, whether written or oral, that is relevant to the establishment of a defense to the formation of the contract itself.

A collector agreed to sell his collection of authentic extras' costumes from a cult classic 80's show to a costume store for $10,000, payable one month after the collection was delivered to the store via a third-party carrier. Due to the time and expense that went into accumulating and repairing the costumes, the collector expected a $2,000 profit. The costumes suffered minor water damage in transit, and the store immediately notified the collector that it was rejecting the collection and would hold the collection until the collector picked them up. The collector told the store that he would look for a new buyer and would pick up the collection in a few weeks. The collector quickly found another buyer willing to pay the original contract price. However, before the collector retrieved the costume collection, the store sold and delivered the costumes to a theater company who knowingly accepted the costumes despite the water damage. The theater company paid the store $15,000 for the collection, which the store retained. If the store's sale of the costume was NOT an acceptance, what is highest value remedy available to the collector? (a) $2,000, the collector's lost profit. (b) $5,000, the difference between market price and contract price. (c) $10,000, the collector-store contract price. (d) $15,000, damages for conversion.

Answer choice D is correct. The store's rejection of the collection was proper under the perfect tender rule, but the store's selling the collection to the theater company constituted conversion. The remedy for conversion is the fair market value of the goods at the time of the conversion of the collection. The $15,000 received by the store from the sale of the collection reflects the collection's fair market value at the time of the conversion. Answer choice A is incorrect because the collector is not a volume seller; consequently, lost profit damages are not available to him. Answer choice B is incorrect because the difference between market and contract prices is an improper measure of damages in cases of conversion. Answer choice C is incorrect because the collector is not limited to the contract price in ascertaining the value of the collection; instead, he is entitled to fair market price of the costumes.

A student inherited a large tract of land from an eccentric uncle. The student had no present need for the land, and because he had numerous student loans, he decided to sell the land. He advertised a proposed sale of the property, and he was soon contacted by a rancher who owned property adjacent to the offered land. The rancher wanted to purchase the student's property to expand his ranch and to build facilities for dairy production. The student told the rancher that his car had just broken down, and that he was eager to sell the property quickly so that he could repair his car for his commute to class. Although the rancher was fully aware of the fair market value of the property, he offered the student a cash price 80 percent lower than the property was worth. The student, disappointed with the low price but desperate to repair his car, accepted the rancher's offer. On these facts, which of the following legal concepts would give the student the best chance of canceling the contract with the rancher? (a) Bad faith (b) Duress (c) Equitable estoppel (d) Unconscionability

Answer choice D is correct. The student's best chance of canceling the contract is that the rancher took advantage of the student's inexperience in order to knowingly negotiate an unfair deal for the property. A contract is unconscionable when it is so unfair to one party that no reasonable person would agree to it. Answer choice A is incorrect because, although a duty of good faith is implied into all contracts, a good-faith argument would only come into play if the rancher used fraud or other dishonest actions to induce the student to agree to the deal, which is not the case here. Answer choice B is incorrect because only a wrongful act or threat that deprives a party of a meaningful choice constitutes duress, and taking advantage of the student's economic need is not duress. Answer choice C is incorrect because it is inapplicable in this situation. Equitable estoppel generally applies when one party has misrepresented a fact, and the other party has injuriously relied on that misrepresentation. Here, the student does not seem to have relied on any misrepresentation—he only accepted an unfair offer.

The owner of a beauty products store mentioned to a long-time customer that she was selling her car. The storeowner showed the customer, who was a sales representative at a clothing store, a picture of the vehicle and told her its year, make, model, and mileage. When the customer expressed an interest, the storeowner gave her the keys and told her to check it out for herself. The customer took the keys, looked over the inside and the outside of the car, and drove it around the block. When the customer returned to the store, the storeowner honestly stated that she knew little about cars and was selling the car with all its faults. The customer indicated that she also knew little about cars. The storeowner and the customer agreed upon a price of several thousand dollars for the car. Several days after the customer paid for the car and took ownership of it, she learned that the car required a costly engine overhaul that neither the storeowner nor the customer was aware of at the time of the sale. The customer has filed a lawsuit against the storeowner for breach of the warranty of merchantability. Is the customer likely to be successful? (a) Yes, because the warranty of merchantability cannot be orally disclaimed. (b)Yes, because the customer knew little about cars. (c) No, because the storeowner was unaware of the problem with the car's engine. (d) No, because the storeowner was not a merchant.

Answer choice D is correct. The warranty of merchantability is an implied warranty that the goods sold are fit for their ordinary purpose. Here, the fact that the car's engine required a costly overhaul indicates that the car was not fit for use as a means of transportation. However, this warranty is only implied when the seller is a merchant with respect to the goods sold. Here, although the storeowner was clearly a merchant with respect to beauty products, she was merely a casual seller of her own vehicle. Consequently, although the customer may be entitled to rescind the sale based upon a mutual mistake, the customer cannot successfully pursue a claim against the owner upon the basis of a breach of the implied warranty of merchantability. Answer choice A is incorrect because the warranty of merchantability may be disclaimed orally. Answer choice B is incorrect because the warranty of merchantability does not generally hinge upon the knowledge of the buyer, but rather it hinges upon what is typically expected when a good is sold—here, that a car can be used as a functioning means of transportation. If a buyer, before entering into the contract, has examined the goods as fully as the buyer desired, or has refused to examine the goods, then there is no implied warranty with respect to defects that an examination ought to have revealed to the buyer. Here, it is unclear whether the defect in question was one that the customer's inspection should have revealed. In any case, there was no warranty of merchantability with regard to this sale because the storeowner was only a casual seller, not a merchant with respect to the car. Answer choice C is incorrect. Although the seller's knowledge about a defect in a good being sold can give rise to an action for misrepresentation, the absence of such knowledge does not automatically operate to protect the seller from liability for the breach of the implied warranty of merchantability.

A jeweler who specialized in engagement rings assisted a man who was trying to pick out the perfect engagement ring. The man was inexperienced with the various cuts of diamonds and types of ring settings. Over the course of a few weeks, the jeweler and the man looked at all of the ring styles and discussed pricing based on the man's budget of $5,000. The man finally settled upon a square cut diamond with a prong setting that was priced at $5,500. The man initially offered the jeweler $4,500 for the ring. While the man and the jeweler were negotiating the price, the jeweler received a phone call regarding a family emergency. The jeweler told the man that he would email him an offer in the evening, and if they could "meet halfway," the jeweler would sell the ring to the man. The man agreed. That evening, both the jeweler and the man received emails from one another at the same time. The jeweler's email contained an offer to sell the ring for $5,000, and the man's email contained an offer to buy the ring for $5,000. Both emails (i) specified the same style of ring that the two parties had discussed earlier that day, (ii) required payment upon receipt of the ring in two weeks, and (iii) were signed with an electronic signature. Based upon their earlier discussions and the jeweler's email offer to sell the ring to him for $5,000, the man did not look for an engagement ring at any other jewelry store. When the man showed up two weeks later to pick up and pay for the ring, the jeweler denied that they had a binding contract and would not sell the ring. If the man sues the jeweler for breach of contract, will he likely succeed? (a) No, because the two emails do not qualify as a writing which is required for the sale of goods over $500 under the Statute of Frauds. (b) No, because two emails received at the same time with the same information do not act as an offer and acceptance. (c) Yes, because the man detrimentally relied upon the jeweler's offer to "meet halfway" and the email offer to sell the ring to him. (d) Yes, because both parties conveyed an intent to contract with one another through prior negotiations and the simultaneous emails.

Answer choice D is correct. Under the UCC, a contract is formed if parties intend to contract, and there is a reasonably certain basis for giving a remedy. A sale-of-goods contract may be made in any manner sufficient to show agreement, even though the moment of its making is undetermined. Answer choice A is incorrect because emails are considered signed writings that satisfy the Statute of Frauds, and electronic signatures are legally valid. Answer choice B is incorrect, as generally with cross-offers, neither offer accepts the other, and there is no contract unless one of the parties accepts the offer he has received. In this case, though, the parties had previously agreed that if they could "meet halfway" that evening, they would have a contract. Answer choice C is incorrect. Promissory estoppel is referred to as a "consideration substitute." The doctrine of promissory estoppel (detrimental reliance) can be used under certain circumstances to enforce a promise that is not supported by consideration. Here, there was an enforceable contract and each party provided adequate consideration. Thus, the doctrine of promissory estoppel does not apply.

A homeowner called a septic cleaning company and made arrangements for the company to remove the waste from the septic tank on the homeowner's property. After completing the job, the company mailed the homeowner a bill for $500, the fair market value of the services rendered by the company. The bill indicated that payment was due in 60 days. Upon receiving the bill, the homeowner called the company and informed it that, since he had lost his job due to an accident, he would not be paying the company's bill. The following day the company filed suit for breach of contract. Ten days later, the homeowner moved to dismiss the suit. The court granted the motion, dismissing the suit without prejudice. Is the court's dismissal proper? (a) No, because the parties' dealings created an implied-in-fact contract. (b) No, because the homeowner has repudiated the contract. (c) Yes, because the vendor failed to demand assurances. (d) Yes, because the vendor's complaint is premature.

Answer choice D is correct. Under the doctrine of anticipatory repudiation, which is applicable when a promisor repudiates a promise before the time for performance is due, the promisee may treat the repudiation as a breach of the contract and sue immediately. However, in a situation in which the date of performance has not passed and the only performance left is payment, the aggrieved party must wait until performance is due before filing suit. Here, despite the fact that the customer unequivocally stated that he would not pay for the services rendered by the vendor, the customer's payment is the only performance left on the contract, and the vendor must therefore wait until the 60-day period expires, as dictated by the contract terms, before filing suit. Thus the court's dismissal without prejudice is proper, as the vendor may re-file the complaint after the 60-day period has expired. Answer choice A is incorrect because, although the dealings between the homeowner and the septic cleaning company did give rise to an implied-in-fact contract, the company cannot seek immediate enforcement of the homeowner's implied promise to pay for the services rendered despite the homeowner's anticipatory repudiation of its contractual obligation. Answer choice B is incorrect because, although the homeowner has anticipatorily repudiated his obligation to the pay the company, the obligation cannot be enforced until the payment is due. Answer choice C is incorrect because a party to a contract need not demand assurances prior to bringing suit under a theory of anticipatory repudiation. First, assurances are only applicable in sales of goods contracts governed by the UCC. Additionally, under such sales of goods contracts, a party who is insecure about the other party's performance may demand assurances of the party causing the insecurity, but demanding assurances is not a prerequisite to bringing suit.

A man was moving to another state and decided that he wanted to give away some of his belongings. He knew his brother had always expressed interest in an antique desk. The man called his brother and said, "I'm going to be moving in two weeks. I would like to give you the antique desk as a gift. I'll drop it off at your house on my way out of town." The brother told the man that he was very grateful for the gift and was looking forward to having the desk in his home office. The brother immediately disposed of his old desk and made room for the antique one. A couple of days later, an appraiser, who was a friend of the man, visited the man's house for dinner. While at his house, he saw the antique desk and informed the man that it was worth well over $20,000. The man decided to keep the desk and did not drop it off at the brother's house on his way out of town. The brother brought suit against the man to recover the antique desk. If the court finds in favor of the man on these facts, what is the most likely reason? (a) A promise to make a gift in the future cannot be enforced. (b) The brother did not rely to his detriment on the man's promise. (c) The man's promise was not in writing. (d) The man's refusal to give the piece of furniture did not cause injustice.

Answer choice D is correct. Under the doctrine of promissory estoppel, a party's promise to make a gift is enforceable if the donor should reasonably expect the promise to induce detrimental reliance by the donee, the promise actually induces such reliance, and the failure to enforce the promise will cause injustice. In this case, the man's promise to give the piece of furniture to his brother will only be enforced if enforcement is necessary to avoid injustice. Because the facts indicate that the brother did rely on the promise by disposing of his desk, and that the court finds in favor of the man anyway, the court must have concluded that the man's failure to give the desk as promised did not cause injustice. Answer choice A is incorrect because a promise to make a gift in the future can be enforced if the requirements for promissory estoppel are met. Answer choice B is incorrect because the brother arguably did rely to his detriment on the man's promise by immediately disposing of his old desk. Therefore, on these facts, this would not be a likely reason to find in favor of the man. Accordingly, answer choice D is a better answer. Answer choice C is incorrect because a writing would not be required to enforce a promise to make a gift under the doctrine of promissory estoppel. Therefore, the lack of a writing would not be dispositive here, and answer choice D is the better choice.

A party-planning company specialized in creating and selling nine different kits for themed parties. A store that sells party related items entered into a written agreement with the company. Under this agreement, the company was to deliver 500 kits to the buyer by November 1. The agreement stated that selections regarding the type of kit and the number of each were to be made by October 15, but did not specify who was to make the selections. Neither the store nor the company selected any assortment of the kits by October 15. On October 16, the company notified the store that due to its breach, the company would not be shipping the party kits. At that time, the company had a surplus of all of their merchandise and could have filled the store's order with any combination of themed kits. On October 17, after receiving the company's notification, the store informed the company of its selections, but the company refused to send the kits that the store selected. If the store sues the company for breach of contract on November 1, is the store likely to prevail? (a) No, because the failure in the agreement to specify the party responsible for making the selection of type and number of each kit renders the contract unenforceable due to the indefiniteness of its terms. (b) No, because the company had no duty to perform since an assortment was not selected by October 15. (c) Yes, because the company was required to make a reasonable selection of available merchandise to fill the order. (d) Yes, because the store's two-day delay in making its selection did not have a material effect on the company's ability to perform the contract.

Answer choice D is correct. When a contract fails to specify the assortment of goods, the duty to select the assortment falls on the buyer. If the buyer fails to specify the assortment of goods, then the seller can treat the failure as a breach by failure to accept the contracted-for goods only if the buyer's failure materially impacts the seller's performance. Since there is no indication that the store's failure to select the assortment of party kits by October 15 would affect the company's ability to satisfy the store's order, the company could not treat that failure as a breach. Consequently, the company's own failure to ship the store's selections constituted a breach of contract for which the store is likely to recover damages. Answer choice A is incorrect because, under the UCC, a contract that is otherwise sufficiently definite is not invalid merely because the agreement fails to specify the assortment of goods to be delivered to the buyer. Answer choice B is incorrect because, even though the selection of the assortment was not made by October 15, the UCC provides that seller can treat the failure as a breach by failure to accept the contracted-for goods only if the buyer's failure materially impacts the seller's performance. Here, that is not the case. Answer choice C is incorrect because, while the company is required to act in a commercially reasonable manner when the buyer fails to make a selection of the goods to be shipped, the company as the seller is not required to make the selection.

The owner of a bed and breakfast hired an artist to paint nature-themed murals in each of the five bedrooms. The contract provided that payment was due upon the satisfactory completion of all five rooms. The owner told the artist that each mural should relate to the name of the bedroom, but otherwise gave the artist broad discretion in designing each mural. When the owner checked the artist's progress a few weeks later, she found that although the murals in the three completed rooms related to the theme of the rooms, the color choices clashed with the overall décor of the bed and breakfast. The owner told the artist that she would accept his performance on the first three rooms, but asked the artist to incorporate a different color palette in the remaining rooms. The artist, unwilling to compromise his artistic autonomy, refused to paint the remaining two rooms and immediately terminated the contract. What is the artist entitled to recover from the owner of the bed and breakfast? (a) Nothing, because the contract expressly provided that payment would be due upon the completion of all five rooms. (b) Nothing, because the murals in the three completed rooms clash with the overall décor of the bed and breakfast. (c) The artist's expenditures in painting the first three rooms, and the artist's anticipated profit for painting the last two rooms. (d) The reasonable value of the artist's services in painting the first three rooms, less any damages the owner may suffer from the artist's failure to paint the last two rooms.

Answer choice D is correct. When a defendant is unjustly enriched by the plaintiff, restitution generally allows the plaintiff to recover the value of the benefit conferred by the plaintiff upon the defendant. Even if the plaintiff has breached the contract, if the defendant has benefited from the plaintiff's performance of the contract, the plaintiff can generally recover in restitution for the benefit conferred on the defendant, less the defendant's damages for the breach. Here, because the owner has accepted the artist's performance with respect to the first three rooms, the artist is entitled to recover the reasonable value of his services, less any damages the owner may suffer due to his breach. Answer choice A is incorrect because a breaching party who has partially performed under a contract may recover restitution for the benefit conferred to the non-breaching party, regardless of when payment was due under the contract. Answer choice B is incorrect. If a party intentionally furnishes services that are materially different from what she promised, then she cannot recover anything in restitution unless the nonbreaching party has accepted or agreed to accept the substitute performance. Here, despite the problematic color scheme, the owner told the artist that she would accept his performance regarding the first three rooms. Therefore, regardless of whether the clashing colors constituted a breach of the contract's "satisfaction" requirement, the owner has accepted the performance, and the artist is entitled to restitution for the reasonable value of the murals. Answer choice C is incorrect because it does not describe a calculation of damages available to a breaching party to a contract. Compensatory damages are meant to compensate the nonbreaching party for actual economic losses. Because the artist has breached this contract, this is not the proper measure of his damages.

A tenant rented a small cabin from a landlord. The lease provided that the tenant was permitted to make structural improvements to the cabin, but that the tenant must pay for such improvements. Relying on this clause in the contract, the tenant contacted a contractor to install a loft in the cabin for $10,000. The tenant and contractor agreed in a writing signed by both parties that payment would be due 30 days after the loft was completed. The contractor knew that the tenant was renting the cabin and sent the landlord a letter informing him of the impending construction on his property. The landlord received the letter and did not reply. The contractor completed the loft, which increased the market value of the cabin by $6,000. Ten days later and three months before the end of her lease, the tenant vacated the cabin and disappeared. Thirty days after the loft was completed, the contractor's bill remained unpaid. The contractor has no remedy quasi in rem under the relevant jurisdiction's mechanic's lien statute. Which of the following will give the contractor the best chance of recovery against the landlord? (a) An action as a third-party beneficiary to the lease. (b) An action based on an implied-in-fact contract. (c) An action based on promissory estoppel. (d) An action in quasi-contract for the benefit conferred to the landlord.

Answer choice D is correct. When a plaintiff confers a benefit on a defendant and the plaintiff has a reasonable expectation of compensation, allowing the defendant to retain the benefit without compensating the plaintiff would be unjust. In this case, the court can permit the contractor to recover the value of the benefit to prevent unjust enrichment. A court may allow restitutionary recovery if the plaintiff has conferred a measurable benefit on the defendant, the plaintiff acted without gratuitous intent, and it would be unfair to let the defendant retain the benefit because either (i) the defendant had an opportunity to decline the benefit but knowingly accepted it, or (ii) the plaintiff had a reasonable excuse for not giving the defendant such opportunity. Here, the contractor conferred a measureable benefit on the landlord by increasing the value of his cabin, and the contractor expected compensation for the work. Additionally, the landlord knew about the construction and did nothing to stop it. Therefore, the contractor should be able to collect restitution from the landlord based on an implied-in-law contract or a quasi-contract. Answer choice A is incorrect because the contractor is not an intended beneficiary of the lease. Answer choice B is incorrect. If one party requests that another party perform a service but does not indicate a price, and the service is performed, this generally creates an "implied-in-fact" contract; agreement is indicated by conduct. This is a contract implied in law to prevent unjust enrichment. Answer choice C is incorrect because the landlord never made any representations to the contractor that would reasonably induce reliance.

On January 8, a liquor company sent a signed, written offer to a retailer containing the following: "We will sell you our last 500 cases of our caffeine-infused vodka for $100 per case." Upon receiving the liquor company's offer, the retailer decided to take a couple of days to contemplate the offer. On January 15, after not hearing from the retailer, the liquor company decided it was no longer willing to sell the vodka to the retailer, but it did not contact the retailer. On January 17, the retailer decided that it was not interested in the vodka and placed a letter rejecting the offer in the mail. However, after hearing about the popularity of the vodka from a number of patrons that same day, the retailer immediately mailed a signed acceptance to the liquor company. The liquor company received the retailer's acceptance on January 20. Later that same day but unknown to either party, the state enacted a law prohibiting the sale or distribution of the caffeine-infused vodka due to a number of serious health risks connected to the product. Due to an error at the post office, the retailer's initial rejection was not received until January 21. The retailer and the liquor company both learned of the state law on January 22.Which of the following is an accurate statement regarding the relationship between the retailer and the liquor company as of January 23? (a) No contract was formed because the liquor company's offer was revoked on January 15. (b) No contract was formed because the retailer's rejection was effective on January 17. (c) An enforceable contract was formed on January 20. (d) A contract was formed on January 20, but the duty to perform under the contract is discharged due to illegality.

Answer choice D is correct. When a rejection is sent first, and then an acceptance is sent, the mailbox rule does not apply, and first to be received by the offeror will control. Here, the retailer placed a letter in the mail rejecting the offer on January 17. However, later that same day, it mailed an acceptance of the offer to the liquor company. Accordingly, the first to be received will control. Because the liquor company received the retailer's acceptance first, a contract was formed on January 20. However, the state law made the sale or distribution of the particular vodka illegal. If a contract becomes illegal after it is formed, the duty to perform under the contract is discharged. As a result, the duty to perform under the contract between the liquor company and the retailer is discharged. Answer choice A is incorrect. A revocation may be made in any reasonable manner and by any reasonable means, and it is not effective until communicated. In this case, although the liquor company determined that it would not sell the vodka to the retailer, it did not communicate that decision to the retailer. Accordingly, the liquor company's offer was not revoked on January 15. Answer choice B is incorrect. A rejection is usually effective upon receipt, not mailing. Thus, the retailer's rejection was not effective on January 17 when it was mailed. Answer choice C is incorrect. Although a contract was formed on January 20, the contract is not enforceable due to illegality. Therefore, this is an incorrect statement of the relationship between these parties.


Set pelajaran terkait

Kin 313 aquatic therapy mid term

View Set

Political Geography: Indian Tribal areas and Union territories

View Set

𝘊𝘏𝘈𝘗𝘛𝘌𝘙 8 ~ 𝘉𝘜𝘚𝘐𝘕𝘌𝘚𝘚

View Set

Security+ Updated Exam Questions 401-500

View Set

Insurance Regulation- Practice Questions

View Set